Текст
                    БИБЛИОТЕЧКА-КВАНТ-
ВЫПУСК 60
ЗАДАЧИ
МОСКОВСКИХ
ФИЗИЧЕСКИХ
ОЛИМПИАД
я>.ж.

БИБЛИОТЕЧКА -КВАНТ выпуск 60 А.И. БУЗДИН В.А. ИЛЬИН И. В. КРИВЧЕНКОВ С.С. КРОТОВ Н.А. СВЕШНИКОВ ЗАДАЧИ МОСКОВСКИХ ФИЗИЧЕСКИХ ОЛИМПИАД Под редакцией С. С. КРОТОВА МОСКВА «НАУКА» ГЛАВНАЯ РЕДАКЦИЯ ФИЗИКО-МАТЕМАТИЧЕСКОЙ ЛИТЕРАТУРЫ 1988 Scan AAW
ББК 22.3 3-15 УДК 53 (023) РЕДАКЦИОННАЯ КОЛЛЕГИЯ Академик Ю. А. Осипьян (председатель), академик А. Н. Кол- могоров (заместитель председателя), кандидат физ .-мат. наук А. И. Буз- дин (ученый секретарь), член-корреспондент АН СССР А. А. Аб- рикосов, академик А. С. Боровик-Романов, академик Б. К. Вайнштейн, заслуженный учитель РСФСР Б. В. Воздвиженский, академик В. Л. Гинз- бург, академик Ю. В. Гуляев, академик А. П. Ершов, профессор С. П. Капица, академик А. Б. Мигдал, академик С. П. Новиков, академик АПН СССР В. Г. Разумовский, академик Р. 3. Сагдеев, профессор Я. А. Смородинский, академик С. Л. Соболев, член- корреспондент АН СССР Д. К. Фадеев. Рецензент кандидат физико-математических наук Г. В. Меледин 315 Задачи московских физических олимпиад / Под ред. С. С. Кротова. — М.: Наука. Гл. ред. физ.-мат. лит., 1988. — 192 с. — (Б-чка «Квант». Вып. 60.) — 40 к. Сборник, в котором содержится около 250 задач по физике, предлагавшихся школьникам на московских физических олимпиадах с 1968 по 1985 г. В книгу вошли наиболее интересные и оригинальные задачи с решениями. Для школьников и преподавателей физики. Может быть использован для подготовки к конкурсным экзаменам в вузы, где предъявляются повышенные требования к знаниям по физике. 1704010000 — 055 <R,_g7 ББК 22.3 053(02)-88 ISBN 5-02-014231-Х © Издательство «Наука». Главная редакция физико-математической литературы, 1988
ПРЕДИСЛОВИЕ Современный. этап развития науки и техники требует подготовки большого числа высококвалифициро- ванных специалистов в области естественных и технических наук. Надлежащее решение этой задачи невозможно без суще- ственного повышения уровня преподавания физики, усиления индивидуального подхода, раннего выявления и развития твор- ческих способностей будущих специалистов. При этом важную роль играют олимпиады по физике для учащихся средних школ и профессионально-технических училищ, которые способ- ствуют выявлению наиболее одаренных учащихся, позволяют правильно сориентировать их в выборе будущей профессии, пропагандируют научно-технические знания среди молодежи. Предлагаемая читателям книга «Задачи московских физиче- ских олимпиад» подытоживает опыт проведения московских олимпиад школьников по физике за последние 15 лет. Москов- ская олимпиада включает три теоретических тура — районный, городской, отборочный — и заключительный, эксперимен- тальный тур, по результатам проведения которого формирует- ся команда школьников г. Москвы для участия во Всесоюзной олимпиаде. Уровень сложности задач нарастает, начиная с про- стейших задач районного тура, решение которых требует лишь уверенного владения основными законами и понятиями физи- ки, и кончая довольно трудными задачами отборочного тура. Некоторые из задач последнего, отборочного тура представ- ляют собой небольшое научное исследование (как правило, эти задачи являются украшением олимпиады). В настоящую книгу включены только задачи теоретических туров. Структура книги полностью отражает представитель- ность различных разделов курса физики на олимпиаде. При этом, естественно, чем раньше тот или иной раздел физики изучается в школе, тем большее число задач этого раздела встречается на олимпиаде, тем больший удельный вес соответ- ствующего раздела в предлагаемой книге. Поскольку от- дельные темы оказались представленными одной-двумя зада- чами, а некоторые — десятком задач, мы не стали вводить подробную рубрикацию. 1* 3
Большинство задач являются оригинальными, значительная их часть предложена авторами книги. Наиболее слож- ные задачи отмечены звездочкой. Представляя произведение замкнутого коллектива авторов, они отражают определен- ные традиции, опыт проведения именно московских олимпиад, передают научные вкусы и стиль работы коллектива еди- номышленников — в этом состоит специфика предлагаемой книги. Учитывая все сказанное, книгу можно рекомендовать очень широкому кругу читателей. Наиболее целесообразно использо- вать ее в качестве дополнения к имеющимся сборникам задач по элементарной физике. Она будет полезна тем, кто после подробного изучения курса физики хочет углубить свои знания, попробовать силы при решении нестандартных задач, развить физическую интуицию. Предназначая ее в первую очередь школьникам старших классов, мы считаем, что и студенты первых курсов найдут в ней кое-что интересное, а руководители физических кружков, преподаватели вечерних и заочных школ, учителя, ведущие факультативные занятия, тоже почерпнут для себя много полезного. Настоящая книга никогда не увидела бы свет, если бы не вдохновляющая поддержка идеи ее создания академиком И. К. Кикоиным, который на протяжении многих лет возглав- лял Центральный оргкомитет Всесоюзной олимпиады школь- ников, редколлегию журнала «Квант» и серии «Библиотечка «Квант». Данная книга является данью нашего уважения и па- мяти этому крупнейшему советскому ученому, мудрому и чут- кому учителю. Нам бы хотелось высказать слова благодарности своим старшим товарищам-коллегам по олимпиадной деятельно- сти — В. К. Петерсону, Г. Е. Пустовалову, Г. Я. Мякишеву, А. В. Ткачуку, В. И. Григорьеву, Б. Б. Буховцеву, совместная работа с которыми способствовала формированию нашего по- нимания того, что такое физическая задача. Мы признательны также всем членам жюри московских олимпиад последних лет. Идеи ряда задач принадлежат им. Наконец, нам очень приятно поблагодарить Г. В. Меледина, самым тщательным образом прочитавшего книгу в рукописи и сделавшего ряд полезных замечаний как по содержанию, так и по стилю книги.
ЗАДАЧИ 1. МЕХАНИКА а.х.
В задачах этого раздела, там, где необходимо, считать ускорение свободного падения д = 10 м/с2. 1.1. Какое время больше -спуска тела по наклонной пло- скости с высоты h (при нулевой начальной скорости) или подъема с такой начальной скоростью, что тело останавливает- ся на той же высоте А? 1.2. Локомотив находился на расстоянии L = 400 м от све- тофора и имел скорость v = 54 км/ч, когда началось тормо- жение. Определите положение локомотива относительно светофора через 1 минуту после начала торможения, если он двигался с ускорением а = 0,3 м/с2. 1.3. Вертолет взлетает с аэродрома по вертикали с ускоре- нием а = 3 м/с2 и начальной скоростью, равной нулю. Через некоторое время tY пилот выключил двигатель. Звук на земле в месте взлета перестал быть слышен спустя время t2 = 30 с. Определите скорость вертолета v в момент прекращения ра- боты двигателя. Считать скорость звука с = 320 м/с. 1.4. Материальная точка начинает двигаться по прямой с постоянным ускорением а. Спустя время после начала ее движения ускорение меняет знак на противоположный, оста- ваясь неизменным по модулю. Определите, через какое время t после начала движения точ- ка окажется в исходном положении. 1.5. Два тела движутся по прямой навстречу друг другу с начальными скоростями и v2 и постоянными ускорениями аг и а2, направленными противоположно соответствующим скоростям в начальный момент времени. При каком максимальном начальном расстоянии /тах между телами они встретятся в процессе движения? 1.6. На упругую плиту свободно падают два стальных ша- рика: 1-й с высоты hr =44 см, 2-й с высоты h2 = 11 см спустя т секунд после 1-го. Через некоторое время т скорости шариков совпадают по модулю и направлению. Определите время т и интервал времени, в течение которого скорости обоих шариков будут равными. Считать, что шарики между собой не соударяются. 1.7* . На вогнутую сферическую поверхность радиуса R с высоты Н = К/8 вблизи вертикальной оси симметрии падают с нулевой начальной скоростью маленькие шарики. Считая удары шариков о поверхность абсолютно упругими, покажите, что после 1-го соударения каждый шарик попадает в низшую точку сферической поверхности. Считать, что шари- ки между собой не соударяются. 6
1.8. Маленький шарик, брошенный с начальной скоростью v0 под углом а к горизонту, ударился о вертикальную стенку, движущуюся ему навстречу с горизонтально направленной ско- ростью v, и отскочил в точку, из которой был брошен. Определите, через какое время t после броска произошло столкновение шарика со стенкой? Потерями на трение прене- бречь. 1.9* . Небольшой шарик движется с постоянной скоростью v по горизонтальной поверхности и попадает в точке А в вер- тикальный цилиндрический колодец глубины Н и радиуса г. Скорость шарика v составляет угол а с диаметром колодца, проведенным в точку А (рис. 1, вид сверху). При каком соотношении между v, Н, г и а шарик после упру- гих соударений со стенками и дном сможет «выбраться» из ко- лодца? Потерями на трение пренебречь. 1.10. Артиллерийское орудие стреляет из-под укрытия, на- клоненного под углом а к горизонту (рис. 2). Орудие находится в точке А на расстоянии I от основания укрытия (точка В). На- чальная скорость снаряда равна v0, траектория снаряда лежит в плоскости рисунка. Определите максимальную дальность полета снаряда Lmax. 1.11. В одной автомашине лобовое стекло имеет угол наклона к горизонту, равный рх = 30°, а в другой р2 = 15°. При каком отношении скоростей двух автомашин vjv2 водители увидят градины, отскакивающими от лобовых стекол своих машин в вертикальном направле- нии? Считать, что относительно земли градины падают вертикально. 1.12. По гладкой горизонтальной по- верхности движется лист фанеры. Ско- рости точек А и В равны v и лежат в плоскости листа фанеры (рис. 3). Рис. 3 Определите скорость точки С. 1.13. Необходимо поставить в небольшой просвет между вереницей автомашин, стоящих вдоль тротуара, еще одну машину. 7
Как следует заезжать в просвет: передним ходом или за- дним, если у машины поворачиваются только передние колеса. 1.14* . Самолет, летящий горизонтально со скоростью и0, начинает подниматься вверх, описывая окружность, лежащую в вертикальной плоскости. Скорость самолета при этом ме- няется с высотой h над первоначальным уровнем движения по закону v2 = Vq — 2aQh. В верхней точке траектории его скорость оказывается равной vr = г0/2. Определите ускорение а самолета в тот момент, когда его скорость направлена вертикально вверх. 1.15. Открытая карусель вращается с-угловой скоростью со. На карусели на расстоянии г от оси вращения стоит человек. Идет дождь и капли дождя падают вертикально вниз со скоростью г0. Как человек должен держать зонт, чтобы наилучшим обра- зом укрыться от дождя? 1.16* . Катушка катится без проскальзывания по горизон- тальной поверхности, причем скорость конца нити (точка А) го- ризонтальна и равна v. На катушку опирается шарнирно за- крепленная в точке В доска (рис. 4). Внутренний и внешний радиусы катушки равны г и R соответственно. Определите угловую скорость со доски в зависимости от угла а. 1.17. На пустую катушку магнитофона, вращающуюся с по- стоянной угловой скоростью, перематывается магнитная лента. После перемотки конечный радиус гк намотки оказался в три раза больше начального радиуса гн (рис. 5). Время перемотки ленты равно За какое время t2 на такую же катушку перемотается лента, вдвое более тонкая? 1.18. При прослушивании магнитофонной записи было за- мечено, что радиус намотки пленки на кассете уменьшился, вдвое за время tr = 20 мин. 8
За какое время t2 после этого радиус еще раз уменьшится вдвое? 1.19. Два колечка О и О' надеты на вертикальные непо- движные стержни АВ и А'В' соответственно. Нерастяжимая нить закреплена в точке А' и на колечке О и продета через ко- лечко О' (рис. 6). Считая, что колечко О' движется вниз с постоянной ско- ростью vl9 определите скорость v2 колечка О, если zl АО О' = а. 1.20. На неподвижном клине, образующем угол а с гори- зонтом, лежит нерастяжимая невесомая веревка (рис. 7). Один из концов веревки прикреплен к стене в точке А. В точке В к веревке прикреплен небольшой грузик. В некоторый момент времени клин начинает двигаться вправо с постоянным ускоре- нием а. Определите ускорение грузика, пока он находится на клине. 1.21. Муравей бежит от муравейника по прямой так, что его скорость обратно пропорциональна расстоянию до центра муравейника. В тот момент, когда муравей находится в точке А на расстоянии lr = 1 м от центра муравейника, его скорость равна = 2 см/с. За какое время t муравей до- бежит от точки А до точки В, которая находится на расстоянии 12 = 2 м от центра муравейника? 1.22. Во время движения паро- воза по закруглению радиуса R Рис. 8 в горизонтальном направлении дует ветер. Форма дымного следа изображена на рис. 8 (вид сверху). Определите, используя рисунок, скорость ветра vB, если из- вестно, что она постоянна, а скорость паровоза vn = 36 км/ч. 9
1.23* . Трое школьников — Слава, Игорь и Никита — реши- ли покататься на каруселях. Слава и Игорь заняли диаметраль- но противоположные точки карусели радиуса г. Никита выбрал карусель радиуса R. Расположение мальчиков в начальный мо- мент времени показано на рис. 9. Учитывая, что карусели касаются друг друга и вращаются в одну и ту же сторону с одной и той же угловой скоростью о, определите характер движения: Никиты с точки зрения Игоря; Славы с точки зрения Никиты. 1.24. На горизонтальной поверхности стоит обруч радиуса R. Мимо него движется со скоростью v такой же обруч. Найдите зависимость скорости vA верхней точки «пересече- ния» обручей от расстояния d между их центрами. Считать, что обручи тонкие и второй обруч «проезжает» вплотную к первому. 1.25. Шарнирная конструкция состоит из трех ромбов, сто- роны которых относятся-как 3:2:1 (рис. 10). Вершина А3 пере- мещается в горизонтальном направлении со скоростью v. Определите скорости вершин А19 Л2, В2 в тот момент, когда все углы конструкции прямые. Рис. 12 1.26. Конец нити, намотанной на катушку, внешний радиус которой равен К, внутренний — г, перекинут через вбитый в стену гвоздь А (рис. 11). Нить тянут с постоянной ско- ростью V. 10
Найдите скорость v0 движения центра катушки в тот мо- мент, когда нить составляет угол а с вертикалью. Считать, что катушка катится по горизонтальной поверхности без про- скальзывания. 1.27. Жесткая заготовка зажата между двумя параллельны- ми направляющими, движущимися горизонтально в противо- положные стороны со скоростями vr и г2. В некоторый момент времени точки касания заготовки и направляющих лежат на прямой, перпендикулярной направлениям скоростей vr и v2 (рис. 12). Какие точки заготовки имеют в этот момент скорости, равные по модулю vr и г2? 1.28. На длинную горизонтальную ленту транспортера, дви- жущуюся с постоянной скоростью, положили брусок и сообщи- ли ему скорость относительно земли v0 = 5 м/с, направленную против движения ленты. Спустя время t = 4 с скорость бруска сравнялась со скоростью ленты. Коэффициент трения между бруском и лентой равен ц = 0,2. Определите скорость v ленты транспортера. 1.29. Тело соскальзывает без начальной скорости с вер- шины наклонной плоскости, образующей угол а с горизонтом. Коэффициент трения ц между телом и наклонной плоскостью изменяется с увеличением расстояния I от вершины наклонной плоскости по закону ц = Ы. Тело останавливается, не дойдя до конца наклонной плоскости. Найдите время t, прошедшее с начала движения тела до его остановки. 1.30. Сани с грузом, едущие по льду, попадают на участок, посыпанный песком, и, не пройдя и половины своей длины, останавливаются не разворачиваясь. После этого им резким толчком сообщают первоначальную скорость. Найдите отношение путей и времен торможения до первой остановки и после резкого толчка. 1.31. Через неподвижное, горизонтально расположенное на некоторой высоте бревно переброшена веревка. Чтобы удер- жать груз массой т = 6 кг, подвешенный на одном конце ве- ревки, необходимо тянуть второй конец с минимальной силой 7\ = 40 Н. Определите минимальную силу Т2, с которой необходимо тянуть веревку, чтобы груз начал подниматься. 1.32. Почему трудно повернуть руль, когда автомобиль стоит, и легко, когда автомобиль движется? 1.33. На тело, движущееся с постоянной скоростью г, на- чинает действовать некоторая постоянная сила. Спустя проме- жуток времени At скорость тела уменьшается в два раза
Спустя еще такой же интервал времени скорость уменьшается еще в два раза. Определите скорость гк тела через интервал времени 3Af с начала действия постоянной силы. 1.34. В закрытый вагон, стоящий на рельсах, проложенных по горизонтальной поверхности, сел человек, у которого имеются пружинные весы и секундомер. Сидя лицом по напра- влению движения вагона (вдоль рельсов) и прицепив к пру- жинным весам грузик массой т, человек стал наблюдать за на- правлением отклонения грузика и показаниями весов, фиксируя те моменты времени, когда показания изменялись. Вагон пришел в движение и первые = 4 с грузик был от- клонен к человеку, а весы показывали l,25mg; затем время, рав- ное t2 = 3 с, грузик висел вертикально, а весы показывали тд. После этого грузик отклонился влево (поперек вагона) и при этом в течение времени г3 = 25,12 с весы опять показывали 1,25тд. Наконец, еще на протяжении t4 = 4 с грузик был откло- нен вперед при тех же показаниях весов. Определите, где относительно своего первоначального по- ложения оказался вагон и с какой скоростью он к этому мо- менту двигался. Считать, что при изменении направления от- клонения и показаний весов человек сразу гасит рукой возникающие колебания. 1.35. Два невесомых одинаковых стержня соединены шар- нирно друг с другом и подвешены на шарнирах к горизонталь- VHofi балке (рис. 13). Жесткость каж- дого из стержней равна к0, угол между ними 2а. Найдите жесткость к системы стержней по отношению к верти- кальному смещению шарнира А под действием некоторой силы F. Считать РИС< 1з смещение малым по сравнению с длиной стержней. 1.36. Из двух расположенных в одной горизонтальной пло- скости на расстоянии s = 10 м одна от другой пружинных пу- шечек одновременно вылетают тяжелые шарики, один из ко- торых (1-й) имеет начальную вертикальную скорость vr = = 10 м/с, а 2-й вылетает под углом а к горизонту со скоростью v2 = 20 м/с. На каждый из шариков действуют сила тяжести и сила сопротивления воздуха F = цг, ц = 0,1 г/с. Найдите, каким должен быть угол а, чтобы шарики столк- нулись в воздухе. 1.37. Легкая пружина длины /, жесткости к установлена на столе вертикально. На нее падает небольшой шарик массой т. 12
Определите, на какой высоте h от поверхности стола шарик будет иметь максимальную скорость. 1.38* . На невесомой нити длиной I привязан тяжелый ша- рик массой т; сила трения шарика о воздух пропорциональна скорости его движения относительно воздуха: Frp = jw. Дует горизонтальный сильный ветер с постоянной скоростью v. Определите период Т малых колебаний. Считать, что коле- бания шарика затухают за время, много большее периода колебаний. 1.39. Резиновую нить массой m и жесткости к подвешивают за один из концов. Определите общее удлинение нити А/. 1.40. Для покоящейся системы, изображенной на рис. 14, найдите ускорения всех грузов сразу после того, как была пере- резана удерживающая их нижняя нить. Считать, что нити неве- сомы и нерастяжимы, пружины неве- сомы, масса блока пренебрежимо мала, трение в подвесе отсутствует. Рис. 14 1.41. Человек.поднимал один из двух грузов равной массы с постоянной скоростью v (рис. 15). В момент, когда оба груза оказались на одинаковой высоте h, верхний блок растормозил- ся (получил возможность вращаться без трения, как и нижний блок). Определите, какой из грузов и спустя какое время t первым окажется на полу, считая, что человек продолжает выбирать ве- ревку с прежней постоянной скоростью v. Массой блоков, вере- вок и растяжением веревок пренебречь. 1.42. Имеются наклонная плоскость и брусок, который мо- жет скользить по ней в различных направлениях (рис. 16). Если бруску сообщить некоторую начальную скорость и, направлен- ную вдоль наклонной плоскости вниз, то он будет двигаться равнозамедленно и пройдет до остановки расстояние Если 13
бруску сообщить такую же по модулю скорость, но направлен- ную вверх, то он, двигаясь вверх, пройдет до остановки рас- Рис. 16 стояние /2- В нижней части на- клонной плоскости установлена идеально гладкая горизонталь- ная направляющая планка. Какое расстояние I пройдет брусок по наклонной плоскости вдоль горизонтальной направ- ляющей планки, если ему сооб- щить в горизонтальном направ- лении ту же по модулю начальную скорость, что и в преды- дущих случаях? 1.43. Брусок толкнули резко вверх вдоль крыши, образую- щей угол а с горизонтом. Время подъема бруска до высшей точки оказалось в два раза меньше, чем время спуска до исход- ной точки. Определите коэффициент трения ц между бруском и кры- шей. 1.44. На «невесомой подставке», образованной двумя глад- кими наклонными плоскостями, каждая из которых составляет угол а с горизонтом, находятся два шарика, расположенные, как показано на рис. 17. Подставка может без трения скользить по горизонтальной плоскости. Верхний шарик, масса кото- рого mlt отпускают. Определите, при каком условии нижний шарик, масса кото- рого т2, начнет после этого «забираться» на подставку. 1.45. Цилиндр массой т и радиуса г опирается на две под- ставки одинаковой высоты (рис. 18). Одна подставка неподвиж- на, а другая выезжает из-под цилиндра со скоростью v. Определите силу давления N цилиндра на неподвижную подставку в тот момент, когда расстояние между точками опоры равно АВ = г]/2. Считать, что в начальный момент под- ставки располагались очень близко друг к другу; трением ме- жду цилиндром и подставками пренебречь. 14
1.46. По двум гладким наклонным плоскостям, образую- щим одинаковые углы а с горизонтом, движутся, касаясь друг друга, цилиндр и клин, одна из плоскостей которого вертикаль- на (рис. 19). Масса цилиндра т19 масса клина т2. Найдите силу давления N клина на цилиндр. Трением ме- жду цилиндром и клином пренебречь. Рис. 19 Рис. 20 1.47. Невесомый стержень длиной I с небольшим грузом массой т на конце шарнирно закреплен в точке А (рис. 20) положении, толч- касаясь при Рис. 21 стер- и находится в строго вертикальном этом тела массой М. От небольшого ка система приходит в движение. При каком отношении масс М/т жень в момент отрыва от тела будет со- ставлять с горизонтом угол а = л/6? Чему будет равна в этот момент скорость и тела? Трением пренебречь. 1.48. Однородный стержень АВ массой т длины I нижним концом опирается о стену и с помощью нити DC удерживается в наклонном наложении (рис. 21). Нить привязана к стене в точке С, а к стерж- ню в точке D, такой, что AD = АВ/3. Углы, составляемые нитью и стержнем со сте- ной, равны аир соответственно. Найдите возможные значения коэффи- циента трения ц между стержнем и стеной. 1.49* . Массивный диск вращается во- круг вертикальной оси с угловой скоро- стью Q. На него сверху опускают диск массой ось которого направлена строго вертикально (рис. 22). Рас- стояние между осями дисков равно d (d > г), коэффициент трения между поверхностями равен ц. Определите установившуюся угловую скорость <о малого диска. Какой момент сил Л необходимо приложить к оси большого диска, чтобы скорость его вращения оставалась не- т радиуса г, 15
изменной? Радиус массивного диска R > d + г. Трением в осях дисков пренебречь. 1.50. Две жестко связанные однородные палочки одинако- вой длины массами m/и т2 образуют угол л/2 и лежат на ше- роховатой горизонтальной поверхности (рис. 23). Систему рав- номерно тянут с помощью нити, прикрепленной к вершине угла и параллельной поверхности. Определите угол а, который составляет нить с палочкой массой 1.51. Мяч, движущийся со скоростью, равной v = 10 м/с, ударяется о ногу футболиста. Определите скорость и, с которой должна двигаться нога футболиста для того, чтобы ударившийся о ногу мяч остано- вился. Считать массу мяча много меньшей массы ноги футбо- листа, а удар абсолютно упругим. 1.52. Вблизи поверхности земли свободно падает тело мас- сой т. В некоторый момент времени в него попадает (и застре- вает) горизонтально летящая тяжелая пуля массой М. Как изменится время падения тела на землю? Определите время падения t тела, если известно, что пуля попала в тело на половине пути, а время свободного падения тела с той же вы- соты равно t0. Считать, что масса пули много больше массы тела (М » т). Сопротивлением воздуха пренебречь. » * v Рис. 24 1.53. Два тела массами т1 = 1 кг, т2 = 2 кг движутся на- встречу друг другу во взаимно перпендикулярных направле- 16
ниях со скоростями vt = 3 м/с, v2 = 2 м/с (рис. 24). В результате соударения тела слипаются. Определите, какое количество теплоты Q выделится в ре- зультате соударения. 1.54. Два подвижных клина одинаковой массы М имеют плавные переходы на горизонтальную плоскость (рис. 25). С ле- вого клина соскальзывает шайба массой т с высоты h. На какую максимальную высоту hmax шайба поднимется на правом клине? Трением пренебречь. 1.55. На гладкой горизонтальной поверхности около стенки покоится симметричный брусок массой с углублением полу- сферической формы радиуса г (рис. 26). Из начального положе- ния без трения соскальзывает маленькая шайба массой т2. Найти максимальную скорость бруска при его последую- щем движении. Рис. 27 1.56. На столе лежит круглая коробка с внутренним диа- метром D, а в ней шайба радиусом г (рис. 27). Коробку как це- лое начинают двигать с постоянной скоростью v, направленной вдоль линии центров коробки и шайбы. В момент времени t0 шайба ударяется о коробку. Найдите временные зависимости перемещения центра шайбы хш и ее скорости относительно стола, начиная с мо- мента t0 и считая все удары шайбы о коробку абсолютно упру- гими. Постройте графики хш (t) и (t). Трением между короб- кой и шайбой пренебречь. 1.57. Тонкий обруч массой М и ради- уса г поставлен на горизонтальную плоскость. В начальный момент обруч покоится. По гладкому каналу, проходя- щему внутри обруча, соскальзывает из верхней точки без начальной скорости небольшая шайба массой т. Определите скорость и центра обруча в тот момент, когда шайба 17
находится в некоторой точке обруча А, радиус-вектор кото- рой образует угол ф с вертикалью (рис. 28). Трением между обручем и плоскостью пренебречь. 1.58. Горизонтально расположенный невесомый стержень длиной 3/ с закрепленными на нем на равных расстояниях друг от друга и от концов удерживающих его вертикальных нитей грузами массами т1 и т2 находится в положении равновесия (рис. 29). Определите силу натяжения Т левой нити в тот момент времени, когда правую нить перерезают. 1.59. Колечко массой т, свободно скрепляющее два тонких одинаковых обруча массой М каждый, начинает соскальзывать вниз; обручи при этом разъезжаются в разные стороны по ше- роховатой горизонтальной поверхности. Определите ускорение а колечка в начальный момент вре- мени, если = a (рис. 30). Трением между колечком и обручами пренебречь. 1.60. Гибкий трубопровод длиной I соединяет в простран- стве точки А и В, разность высот между которыми равна h (рис. 31). Внутри трубопровода по всей его длине лежит ве- ревка, которую удерживают в точке А. Определите ускорение а, с которым начнет двигаться верев- ка в начальный момент времени после того, как ее отпустят. д Трением между веревкой и стен- Рис. 31 Рис. 32 1.61. На группу из трех гладких одинаковых кубиков, лежа- щих на гладкой горизонтальной поверхности, как показано на 18
рис. 32, налетает со скоростью v гладкая шайба. Масса каждого кубика равна массе шайбы. Диаметр шайбы и ее высота равны ребру кубика. Определите скорости всех тел после соударения. 1.62. В гладкой неподвижной круговой горизонтальной трубе покоятся несколько одинаковых шариков. Один из шариков взрывается, распадаясь на две части неравной массы. Определите окончательную скорость образовавшегося по- сле всех соударений тела. Считать все соударения абсолютно неупругими. 1.63. Три небольших тела, массы которых относятся, как 3:4:5 (масса самого легкого тела равна т), удерживаются в трех различных точках на внутренней поверхности гладкой полусферической чаши радиуса г. В нижней точке чаша закреп- лена на горизонтальной поверхности. В некоторый момент времени тела отпускают и предоставляют самим себе. Определите максимальное количество теплоты Q, которое может выделиться в такой системе. При каком начальном по- ложении тел это осуществится? Считать все соударения тел аб- солютно неупругими. 1.64. Покажите, что максимальная скорость, которую при столкновении может сообщить протону а-частица, составляет 1,6 ее начальной скорости. 1.65. При движении по песку рекомендуют давление возду- ха в шинах автомобиля уменьшить. Зачем? 1.66. Длинная гладкая цилиндрическая труба радиуса г наклонена под углом а к горизонту (рис. 33). Из точки А по внутренней поверхности трубы пускают вверх небольшое тело, направление начальной скорости которого составляет угол ф с прямой АВ. Определите минимальную начальную скорость v0, при ко- торой тело начнет двигаться вверх без отрыва от поверхности трубы. 1.67* . К оси колеса массой т ра- диуса г привязана нерастяжимая ве- ревка, которую тянут в горизонталь- ном направлении в плоскости колеса. Колесо катится без подскоков по ре- ( д \ шетке, состоящей из параллельных \ \ горизонтальных прутьев, расстоя- \ ние между которыми равно /, при- "7Z > 00 чем I г. Определите, какой должна быть средняя сила натяжения Т веревки, чтобы колесо двигалось в среднем с постоянной ско- ростью v? Считать массу колеса сосредоточенной в его оси. 19
1.68. По шероховатой горизонтальной плоскости, переходя- щей в наклонную, составляющую угол а с горизонтом, катится без проскальзывания с некоторой скоростью v, перпендикуляр- ной границе раздела плоскостей, колесная пара, состоящая из двух легких колес радиуса г, насаженных на тонкую тяжелую ось (рис. 34). Определите, при каком значении v колесная пара перекатит- ся с горизонтальной плоскости на наклонную без отрыва. 1.69. Тонкий обод массой т и радиуса г скатывается с на- клонной плоскости, образующей угол а с горизонтом, наматы- вая при этом на себя тонкую ленту, линейная плотность которой равна р (рис. 35). В начальный момент обод находится на высоте h над горизонтальной поверхностью. Определите, на каком расстоянии 5 от основания наклонной плоскости обод остановится. Считать переход от наклонной плоскости к горизонтальной поверхности плавным. 1.70. С аэростата, парящего в воздухе, сбросили два одина- ковых по размерам маленьких шарика массой и т2 (гп1> > т2), связанных тонкой невесо- мой нитью. Найдите силу натяжения Т ни- ти в полете после того, как дви- жение шариков установится. 1.71*. Шарик невесомой нера- стяжимой нитью прикреплен к неподвижному цилиндру радиуса г. Первоначально нить была на- мотана так, что шарик касался ци- линдра. В некоторый момент вре- мени шарику была сообщена скорость v в радиальном направ- лении и нить начала разматываться (рис. 36). Найдите длину / размотанного участка нити к моменту времени г. Силой тяжести пренебречь. 1.72. Три маленьких шарика одинаковой массы — белый (Б), зеленый (3), голубой (Г) — закреплены невесомыми стержнями 20
в вершинах равностороннего треугольника со стороной I. Си- стема шариков положена на гладкую горизонтальную поверх- ность и приведена во вращение вокруг центра масс с перио- дом Т. В некоторый момент времени голубой шарик отрывается от каркаса. Определите расстояние L, на котором окажется голубой ша- рик от зеленого спустя время Т. 1.73. На расстоянии / слева от края стола лежит брусок, со- единенный с другим бруском такой же массы через невесомый блок невесомой нерастяжимой нитью длины 21, перекинутой через блок (рис. 37). Правый брусок удерживают на одном уровне с левым так, что нить не натянута и не провисает, затем его отпускают. Что произойдет раньше: левый брусок доедет до края (до- стигнет блока) стола или правый брусок ударится о стол? Рис. 37 1.74. На концах невесомой нерастяжимой нити, перебро- шенной через невесомый блок, находятся два груза одной и той же массы (рис. 38). Сначала система покоится и грузы на- ходятся на одном уровне. За- тем правому грузу толчком со- общается горизонтальная ско- рость v в плоскости рисунка. Какой из грузов окажется ниже через некоторое время? 1.75. К концу свободно вися- щего стержня на двух нитях длиной /1 = 7 см и 12 = И см подвешены шарики массами mr = 56 г и т2 = 28 г (рис. 39). Определите угловую ско- рость со, с которой надо вра- щать стержень вокруг вертикальной оси, чтобы он не отклонялся от вертикального положения. 21
1.76. Вокруг вертикальной оси вращается с постоянной угловой скоростью со невесомый горизонтальный жесткий стер- жень, по которому без трения могут двигаться два шарика одной и той же массы т. Шарики соединены между собой неве- сомой пружиной жесткости к, длина которой в ненапряженном состоянии равна /0. Ближайший к вертикальной оси шарик со- единен с ней такой же пружиной. Найдите длину каждой из пружин. При каких условиях ша- рики будут двигаться по окружностям? 1.77. Зависимость кинетической энергии WK тела от переме- щения s при движении тела по прямой Изображена на рис. 40. Известно, что в точке А на тело действовала сила FA = 2 Н. Определите, какие силы действовали на тело в точках В и С. Рис. 41 Рис. 42 Рис. 43 1.78. Лента транспортера длиной /, на которой лежит бру- сок массой т, движется со скоростью v (рис. 41). Определите скорость v0, с которой нужно толкнуть брусок против движения транспортера, чтобы количество теплоты, вы- делившееся при торможении бруска лентой транспортера, было максимальным. Чему равно это максимальное количество теплоты Q, если коэффициент трения равен ц и выполняется условие v < ]/2ц/^? 22
1.79. Массивная труба скатывается с одинаковой высоты с горок разного профиля (рис. 42, 43). В первом случае про- скальзывания нет, а во втором случае труба проскальзывает на некотором участке пути. Определите, в каком случае скорость трубы в конце горки будет меньше. 1.80. На легкой пружине подвешен тяжелый груз. Пружину медленно оттягивают вниз за середину, совершая при этом не- которую работу А, затем пружину отпускают. Определите максимальную4 кинетическую энергию WK груза при последующем движении. 1.81. Массы двух звезд равны тг и т2, расстояние между ними равно /. Найдите период Т обращения этих звезд по круговым орби- там вокруг их общего центра. 1.82. Метеорит, летящий на планету массой М (по прямой, проходящей через центр планеты), попадает в автоматическую космическую станцию, вращавшуюся вокруг планеты по круго- вой орбите радиуса R. Масса станции в 10 раз превосходит массу метеорита. В результате столкновения метеорит застре- вает в станции, которая переходит на новую орбиту с мини- мальным расстоянием до планеты R/2. Определите скорость и метеорита перед столкновением. 1.83. Высадившись на полюсе некоторой планеты, космо- навты обнаружили, что сила тяжести там составляет 0,01 зем- ной, а продолжительность суток такая же, как и на Земле. При Рис. 44 исследовании планеты оказалось, что на ее экваторе тела неве- 7'л сомы. Определите радиус R этой пла- неты. 1.84. Радиус орбиты Нептуна в 30 раз больше радиуса орбиты Земли. Определите период обращения Нептуна 7н вокруг Солнца. 1.85. На нити, перекинутой че- рез два блока, подвешены три гру- за, массы которых равны ть т2 и М (рис. 44). Блоки находятся на одинаковой высоте от точек подвеса. Найдите соотношения между массами грузов, при которых вся система будет находиться в состоянии равновесия. Всегда ли эти условия осуществимы? Трением пренебречь. 1.86. Определите, при каком минимальном коэффициенте трения gmin однородного тонкого стержня о пол человек может 23
медленно без проскальзывания поднять его с пола до верти- кального положения, прилагая к концу стержня силу, перпенди- кулярную ему. 1.87. Три невесомых шарнирно связанных в точках С и D стержня длины I каждый закреплены также шарнирно в точ- ках А и В, лежащих на одной горизонтали (рис. 45); длина АВ = 21. К шарниру С подвесили груз массой т. Определите минимальную силу Fmin, приложенную к шар- ниру D, при которой средний стержень сохраняет горизонталь- ное положение. 1.88. Если шестигранный карандаш поместить на наклон- ную плоскость, составляющую угол а с горизонтальной по- верхностью, перпендикулярно ее образующей (линии пересече- ния плоскости с горизонтальной поверхностью), карандаш будет покоиться. Если его положить параллельно образующей, он будет скатываться вниз. Определите угол ф между осью карандаша и образующей наклонной плоскости (рис. 46), при котором карандаш еще бу- дет находиться в равновесии. 1.89. Однородный стержень длины 21 опирается одним кон- цом о вертикальную стену, а другим концом о гладкую непо- движную поверхность. Какой функцией у(х) должно описываться сечение этой по- верхности, чтобы стержень в любом положении оставался в равновесии даже в отсутствие трения? Считать, что стержень все время находит- ся в фиксированной вертикальной плоско- сти, перпендикулярной плоскости стены. 1.90. Тонкий абсолютно жесткий неве- сомый стержень, на конце которого за- креплен точечный шарик, отклонили на небольшой угол а от положения равновесия и отпустили. В момент, когда стержень со- ставлял угол Р < а с вертикалью, про- Рис. 47 24
изошло абсолютно упругое соударение шарика с наклонной стенкой (рис. 47). Определите отношение 1\/Т периода колебаний такого маятника к периоду математического маятника той же длины. 1.91* . На чашку пружинных весов массой М с некоторой высоты падает шарик массой m (М » т). Жесткость пружины равна к. Определите смещение Ах точки, около которой будет совер- шать колебания стрелка весов. Считать удары шарика о чашку абсолютно упругими. 1.92. По длинной проволоке, изогнутой в вертикальной плоскости в виде графика некоторой функции, может двигаться без трения бусинка массой ш. Пусть 1А — длина участка прово- локи от начала координат до некоторой точки А. Известно, что если отпустить бусинку в точке А, такой, что lA < 1Aq9 то ее дви- жение будет строго гармоническим: I(t) = lAcosot. Докажите, что существует такая точка В (1Ао в которой условие гармоничности колебаний будет нарушено. 1.93. На горизонтальной плоскости лежат два бруска мас- сами m и 2m, соединенные пружиной жесткости к. Найдите период Т малых продольных колебаний системы. Трением пренебречь. 1.94. Тяжелое круглое бревно подвесили за концы на двух канатах так, что расстояние между точками подвеса каждой веревки равно диаметру бревна. Длина каждого вертикального участка канатов равна /. Определите период Т малых колебаний системы в верти- кальной плоскости, перпендикулярной бревну. //////////, 1.95. На горизонтальных рельсах находится агС груз массой М. К нему прикреплен маятник — шарик массой m на невесомой нерастяжимой у у нити. Груз может двигаться только вдоль ^Zaaaa/A>j рельсов. V у Найдите отношение периодов Т211\ малых А Д колебаний маятника в параллельной и перпен- \с/ дикулярной рельсам вертикальных плоскостях. у 1.96. Четыре невесомых стержня длины I каж- f~i дый соединены шарнирно и образуют ромб (рис. 48). Шарнир А закреплен, а к шарниру С ис‘ подвешен груз. Шарниры D и В соединены неве- сомой пружиной, имеющей в недеформированном состоянии длину 1,5 I. В положении равновесия стержни образуют с верти- калью углы а0 = 30°. Найдите период Т малых колебаний груза. 25
1.97. Тонкий обруч, шарнирно закрепленный в точке А, рас- полагают в начальный момент так, что его центр масс нахо- дится почти прямо над точкой А (рис. 49). После этого обруч Рис. 49 Рис. 50 отпускают без толчка, и спустя время т = 0,5 с центр масс обруча занимает крайнее ниж- нее положение. Определите время t, за ко- торое вернется в нижнее поло- жение равновесия маятник, представляющий собой массив- ный шарик В, закрепленный на невесомом жестком стержне, длина которого равна* радиусу обруча, если в начальный момент шарик занимал почти крайнее верхнее положение (рис. 50) и был отпущен без толчка. 1.98. Невесомый жесткий стержень с грузом на конце при- креплен шарнирно в точке А к стене так, что может поворачи- ваться во все стороны (рис. 51). Стержень удерживается в гори- зонтальном положении вертикальной нерастяжимой нитью длины /, прикрепленной к его середине. Грузу сообщили им- пульс, перпендикулярный плоскости рисунка. Найдите период Т малых колебаний системы. 1.99. Одна веревка качелей закреплена выше другой на ве- личину Ь. Расстояние между столбами качелей равно а. Длины веревок равны и 12, причем /1 4-12 = а2 4- Ь2 (рис. 52). Найдите период Т малых качаний на таких качелях. Раз- мерами человека пренебречь по* сравнению со всеми указанными выше длинами. Рис. 51 1.100. Лифтер высотного здания, будучи человеком пунк- туальным, повесил на стену лифта точные маятниковые часы, чтобы знать, когда кончается рабочий день. Время движения лифта с ускорением, направленным вверх и направленным 26
вниз, одинаково (по неподвижным часам); одинаковы также модули ускорений. Как вы думаете, закончит ли лифтер работу вовремя, пере- работает или недоработает? 1.101. Как известно, атмосферное давление уменьшается с высотой. Поэтому в самых верхних этажах высотного здания МГУ атмосферное давление должно быть меньше, чем в самых нижних. Чтобы проверить это, учащийся соединил трубами одно из колен манометра с верхней аудиторией, а другое с нижней. Что показал манометр? 1.102. Две тонкостенные трубки, закрытые с одного конца, • вставлены одна в другую и целиком заполнены ртутью. Пло- щади поперечных сечений трубок равны 5 и 25. Атмосферное давление равно р0 = pprgh, где ррт — плотность ртути, д — уско- рение свободного падения, h — высота. Длина каждой трубки равна I > h. Рис. 53 Какую работу А нужно произвести внешними силами, чтобы медленно вытащить внутреннюю трубку? Давлением па- ров ртути и силами сцепления ртути с материалом трубок пренебречь. 1.103. На горизонтальной поверхности расположены два цилиндра. Ось одного из них горизонтальна, а другого верти- кальна, и в нижней части ци- линдры соединены трубкой не- большого сечения. «Г оризон- тальный» цилиндр радиуса г с одной стороны открыт, и в него вставлен поршень (рис. 53). «Вер- тикальный» цилиндр открыт с верхней стороны. В цилиндры налита вода, причем в «горизонтальном» цилиндре она заполняет все отделяемое поршнем ‘пространство, а в «вертикальном» стоит на некотором уровне. Определите уровень h воды в вертикальном цилиндре, при котором поршень находится в равновесии. Трением прене- бречь. 1.104. На пробку массой тпр намотана проволока из алю- миния. Плотность пробки равна рпр = 0,5 • 103 кг/м3, алюминия рал = 2,7 • 103 кг/м3, воды рв = 1 • 103 кг/м3. Определите, какую минимальную массу тал проволоки надо намотать на пробку, чтобы пробка вместе с проволокой пол- ностью погрузилась в воду. 1.105. К шару массой М = 10 кг и диаметром D = 0,3 м (объем такого шара V = 0,0141 м3) прикреплена одним концом 27
железная цепь, другой конец цепи свободен. Длина цепи равна I = 3 м, масса тп = 9 кг. Шар с цепью находится в водоеме, глубина которого равна Н = 3 м. Определите глубину, на которой будет плавать шар. Счи- тать, что железо тяжелее воды в 7,85 раза. 1.106. На рычаге уравновешены два различных по весу, но одинаковых по объему тела. Нарушится ли равновесие, если погрузить рычаг в воду так, чтобы вода покрыла тела целиком? 1.107. В двух одинаковых сосудах с водой плавают плоская широкая и высокая узкая коробочки. Коробочки не утонули, когда в каждую из них положили по одинаковому тяжелому предмету массой тп. В каком из сосудов уровень воды при этом поднялся выше? . 1.108. В сосуде с ртутью плавает стальной шарик. Как изменится объем части шарика, погруженной в ртуть, если поверх ртути налить слой воды, полностью закрывающей шарик? 1.109. В сосуде с водой, поверх которой налит слой более легкого масла, плавает кусок льда. Как изменится уровень границы раздела воды и масла, если лед растает? Как при этом изменится общий уровень жидкости в сосуде? 1.110. К концу однородной палочки массой М = 4,4 г под- вешен на невесомой нити однородный алюминиевый шарик ра- диуса г = 0,5 см. Палочку кладут на край стакана с водой, до- биваясь такого положения равновесия, при котором погружен- ной в воду окажется половина шарика (рис. 54). Плотность алюминия равна рал = 2,7 • 103 кг/м3, плотность воды рв = = 1 • 103 кг/м3. Определите, в каком отношении у/х делится длина палочки в этом случае. Поверхностным натяжением на границе шарика и воды пренебречь. Рис. 54 Рис. 55 1.111. До какого деления заполнит трубку ртуть свободно падающего барометра длиной 105 см при атмосферном давле- нии 760 мм рт. ст.? 28
1.112. Простейший акселерометр (прибор для измерения ускорений) может быть сделан в виде трубки, заполненной жидкостью и изогнутой, как показано на рис. 55. При движении уровень жидкости в левом колене установился на высоте hl9 а в правом — на высоте h2. Найдите ускорение а вагона, в котором находится акселеро- метр. Считать диаметр трубки много меньше, чем hr и h2. 1.113. Реактивный лайнер, длина салона которого равна / = = 50 м, летит горизонтально с ускорением а = 1 м/с2. Плот- ность воздуха в салоне равна р = 1,2 • 10" 3 г/см3. На сколько отличается от атмосферного давление на уши пассажиров, сидящих в начале, середине и конце салона? 1.114. Закрытая с обоих кон- цов трубка, полностью запол- ненная водой, равномерно вра- щается в горизонтальной пло- скости вокруг оси 00'. На боко- вой стенке трубки на расстояниях гг и г2 от оси вращения установлены манометры, которые показывают давления pY и р2 соответственно (рис. 56). Определите угловую скорость со вращения трубки. Плот- ность воды Рв считать известной. 1.115. Предположим, что в некоторой среде сила сопроти- вления F движению тела зависит от скорости v последнего как F = |wa, где а > 0. При каких значениях показателя степени а тело после при- дания ему начального импульса проходит бесконечно большое расстояние? 1.116. Известно, что атмосферное давление на Марсе соста- вляет 1/200 часть от атмосферного давления на Земле. Диаметр Марса примерно вдвое меньше земного, и если средняя плот- ность Земли равна Рз = 5,5 • 103 кг/м3, то Марса рм = = 4103 кг/м3. Определите, во сколько раз масса марсианской атмосферы меньше, чем земной.
2. ТЕПЛОТА И МОЛЕКУЛЯРНАЯ ФИЗИКА
В задачах этого раздела, там, где необходимо, принять универ- сальную газовую постоянную R = 8,3 ДжДмоль • К). 2.1. В двух вертикальных цилиндрах различного попереч- ного сечения под поршнями, массы которых т1 = 1 кг и т2 = = 2 кг, находится газ при постоянной температуре, а над поршнями — вакуум. Цилиндры соединены внизу трубкой, а поршни располагаются на одинаковой высоте h0 = 0,2 м. Какова будет разность их высот h, если увеличить массу 1-го поршня до массы 2-го? 2.2. Стенки сосуда, в котором находится газ температуры Т, имеют температуру Тс. В каком случае давление газа нач стенки сосуда больше: когда стенки сосуда холоднее газа (Тс < Т) или когда теплее (Тс > Т)? 2.3. С п молями идеального газа совершен круговой про- цесс (цикл) 1 — 2 — 3 — 4 — 1, состоящий из двух изобар 2 — 3 и 4 — 1, изохоры 1 — 2 и некоторого процесса 3 — 4, изображен- ного на рКдиаграмме прямой линией (рис. 57). Температуры газа в состояниях 1, 2, 3 равны Т19 Т2, Т3 соответственно, точки 2 и 4 лежат на одной изотерме. Определите работу А газа за цикл. Рис. 58 2.4. С 3 молями идеального одноатомного газа совершен цикл, изображенный на рис. 58. Температуры газа в различных состояниях равны: 7\ = 400 К, Т2 = 800 К, Т3 = 2400 К и Т4 = = 1200 К. Найдите работу А газа за цикл. 2.5. Определите работу А, которую совершает идеальный газ в замкнутом цикле /->4->3->2->1, изображенном на рис. 59, если рх = 105 Па, р0 = 3• 105 Па, р2 = 4-105 Па, V2 — — V\ = 10 л и участки цикла 4 — 3 и 2 — 1 параллельны оси V. 2.6. Над газом совершают два тепловых процесса, нагревая его из одного и того же начального состояния до одинаковой конечной температуры. На рК-диаграмме процессы изобра- жаются прямыми линиями 1—3 и 1 — 2 (рис. 60). 31
Определите, при каком из процессов газу сообщается боль- шее количество теплоты. Рис. 59 2.7. Сосуд вместимостью V = 30 л разделен на три равные части неподвижными полупроницаемыми тонкими перегородка- ми (рис. 61). В левую часть вводят тН2 = 30 г водорода, в среднюю тО2 = 160 г кислорода и в правую mN2 = 70 г азота. Через ле- вую перегородку может диффундировать только водород, че- рез правую — водород и азот. Какое давление будет в каж- дой из трех частей сосуда после установления равновесия, если он поддерживается при посто- янной температуре Т= 300 К? 02 <м Z Рис. 61 2.8* . Спускаемый аппарат космического корабля прибли- жается к поверхности планеты по вертикали с постоянной ско- ростью, передавая на борт корабля данные о наружном давле- нии. График зависимости давления (в условных единицах) от времени приведен на рис. 62. Опустившись на поверхность пла- неты, аппарат измерил и передал на борт данные о температу- ре: Г = 700 К и ускорении свободного падения: /7=10 м/с2. Определите: скорость v спуска аппарата, если известно, что атмосфера планеты состоит из углекислого газа СО2; температуру Th на высоте h = 15 км над поверхностью планеты. 2.9. В вертикальном цилиндре вместимостью V под невесо- мым поршнем находится п молей идеального одноатомного газа. 32
Газ под поршнем теплоизолирован. На поршень положили груз массой М, в результате чего поршень переместился на расстояние h. Определите конечную температуру газа Тк, установившую- ся после перемещения поршня, если площадь поршня равна S, атмосферное давление р0. 2.10. В вертикальном цилиндре с площадью поперечного сечения S под поршнем, масса которого равна М, находится 1 моль идеального одноатомного газа. В некоторый момент времени под поршнем включается нагреватель, передающий газу за единицу времени количество теплоты q. Определите установившуюся скорость v движения поршня при условии, что давление таза под поршнем постоянно и рав- но р0, газ под поршнем теплоизолирован. 2.11* . Произведение давления газа на его объем (pF) не ме- няется с изменением объема при постоянной температуре, только если предположить, что газы, с которыми мы имеем де- ло, являются идеальными. Определите, будет уменьшаться или увеличиваться произве- дение pV при очень сильном сжатии газа, если не делать пред- положения об идеальности последнего. 2.12* . Горизонтальный цилиндрический сосуд длиной 21 разделен тонким нетеплопроводящим поршнем на две равные части, в каждой из которых находится по п молей идеального одноатомного газа при температуре Т. Поршень прикреплен к торцам сосуда недеформированными пружинами жесткости к каждая (рис. 63). Газу в правой части сообщили количество теплоты Q, в результате чего поршень сместился влево на рас- стояние х = //2. Определите количество теплоты Q', отданное при темпера-, туре Т термостату, с которым газ в левой части все время на- ходился в тепловом контакте. Рис. 64 Рис. 63 2.13. Теплоизолированный сосуд разделен на две части не- теплопроводящим поршнем, который может перемещаться в сосуде без трения. В левой части сосуда содержится 1 моль 2 А. И. Буздин и др. 33
идеального одноатомного газа, в правой — вакуум. Поршень соединен с правой стенкой сосуда пружиной, длина которой в свободном состоянии равна длине сосуда (рис. 64). Определите теплоемкость С системы. Теплоемкостью сосу- да, поршня и пружины пренебречь. 2.14. Докажите, что к. п. д. тепловой машины, использую- щей цикл, состоящий из двух изотерм и двух изохор, меньше к. п. д. идеальной тепловой машины Карно, работающей с те- ми же нагревателем и холодильником. 2.15* . Предположим, что планету массой М и радиуса г окружает атмосфера постоянной плотйости, состоящая из га- за с молярной массой ц. Определите температуру Т атмосферы на поверхности пла- неты, если толщина атмосферы равна h (h « г). 2.16. Известно, что если температура на улице равна — 20 °C, то в комнате температура равна -1-20 °C, а если на улице температура равна —40 °C, то в комнате устанавливается температура -I-10 °C. Найдите температуру Т батареи, отапливающей комнату. 2.17. Космический объект имеет форму шара радиусом R. По всему его объему равномерно распределены источники, обеспечивающие выделение теплоты с постоянной скоростью. Количество теплоты, выделяемое единицей площади поверхно- сти, пропорционально 4-й степени термодинамической температуры. , Во сколько раз измени- лась бы температура объекта, если его радиус уменьшился бы вдвое? 2.18*. Теплообменник длины I состоит из трубы площадью поперечного сече- ния 2S, внутри которой про- ходит другая труба площадью поперечного сечения S (рис. 65). Трубы тонкостенные. Вся конструкция теплоизолирована от внешней среды. В трубах со скоростью v прокачивается жид- кость плотностью р и удельной теплоемкостью с. Температуры жидкости при входе в теплообменник равны Tni и Тк2 соответ- ственно. Определите температуры и Т^2 жидкости при выходе из теплообменника, если потоки жидкости по трубам текут на- встречу друг другу. Считать, что теплота, переданная в единицу времени через единичную площадку, пропорциональна раз- ности температур с коэффициентом пропорциональности к. Т«1 —►и — Рис. 65 34
Теплопроводностью жидкости в направлении ее течения прене- бречь. 2.19* . В закрытом цилиндрическом сосуде с площадью ос- нования S находится вещество в газообразном состоянии вне поля тяготения Земли. Масса газа равна М, давление р. причем р « Рнас> где РиасДавление насыщенных паров вещества яри данной температуре. Сосуд начинают разгонять с ускорением а, направленным по оси цилиндра. Температура поддерживает- ся постоянной. Определите, какая масса жидкости тж образуется в резуль- тате движения в сосуде. 2.20. На некоторой планете давление насыщенною во дяно- го пара равно р0 = 760 мм рт. ст. Определите его плотность р. 2.21. В холодную погоду изо рта при дыхании идет ’<пар». Если приоткрыть дверь в теплую избу в морозный тень, то в комнату тоже врывается «пар». Объясните эти явления. 2.22* . Сосуд объемом V = 2 л содержит тИ2 = 2 г водорода и немного воды. Давление в сосуде равно рн = 17 - 10s Па. Со- суд нагревают так, что давление в нем увеличивается до рк = = 26-105 Па, и часть воды испаряется. Молярная масса во- дяных паров равна ц=18-10“3 кг/моль. Определите начальную Тн и конечную Тк температуры воды и ее массу Am. Указание. Воспользуйтесь следующей температурной за- висимостью давления насыщенных паров воды: Г, °C 100 120 133 152 180 рнас, 105 Па 1 2 3 5 10 2.23. Нижний конец капилляра радиусом г = 0.2 мм и длины I = 8 см погружен в воду, температура которой по- стоянна и равна Тн = 0 °C. Температура верхнего конца капил- ляра равна Тв= 100 °C. На какую высоту h поднимется вода в капилляре? Считать, что теплопроводность капилляра намного превосходи! тепло- проводность воды в нем. Теплообменом с окружающим возду- хом пренебречь. Указание. Воспользуйтесь следующей температурной за- висимостью поверхностного натяжения воды: Г, °C 0 20 50 90 с, мН/м 76 73 67 60 2.24. В цилиндре с подвижным поршнем находится воздух под давлением рг и мыльный пузырь радиуса п Поверхностное 2* 35
натяжение равно ст, температура Т поддерживается постоян- ной. Определите давление р2, до которого нужно сжать воздух медленным вдвиганием поршня, чтобы мыльный пузырь уменьшил свои размеры вдвое. 2.25. Почему при кладке кирпичных печей используют гли- няный раствор для скрепления кирпичей, а не, например, це- ментный (более прочный)? Учесть, что для кладки печей ис- пользуют красный кирпич, сделанный из глины. 2.26. В теплоизолированном сосуде имеются две жидкости с начальными температурами и Т2 и удельными теплоемко- стями с1 и с2, разделенные нетеплопроводящей перегородкой. Перегородку убирают, и после установления теплового рав- новесия разность между начальной температурой одной из жидкостей и установившейся в сосуде температурой Т оказы- вается в два раза меньше разности начальных температур жидкостей. Найдите отношение масс жидкостей m1/m2. 2.27. В первый раз в пробирку налили воду при температу- ре 20 °C. Дно пробирки погрузили в большое количество воды при температуре 80 °C. Вода в пробирке нагрелась за время tr до 80 °C. Во второй раз в пробирку налили воду при темпера- туре 80 °C. Дно пробирки погрузили в большое количество во- ды при температуре 20 °C. Вода в пробирке охладилась за вре- мя t2 до 20 °C. Какое время больше: tt или t2? 2.28. В два одинаковых легких металлических сосуда нали- ли одну и ту же массу воды. Тяжелый шарик (масса которого равна массе воды, а его плотность много больше плотности воды) опустили на тонкой нетеплопроводящей нити в один из сосудов так, что он находится в центре объема налитой воды. Сосуды нагрели до температуры кипения воды и поставили остывать. Известно, что время остывания сосуда с шариком до температуры окружающей среды в к раз больше времени осты- вания сосуда без шарика. Определите отношение удельных теплоемкостей материала шарика и воды сш/св. 2.29. Два одинаковых теплоизолированных цилиндрических калориметра высоты h = 75 см заполнены на 1/3. Первый — льдом, образовавшимся в результате замерзания налитой в не- го воды, второй — водой при Тъ = 10 °C. Воду из второго кало- риметра переливают в первый, в результате чего он оказывает- ся заполненным на 2/3. После того как температура в первом калориметре установилась, уровень заполнения его увеличился на А/i = 0,5 см. Плотность льда равна рл = 0,9 рв, удельная 36
теплота плавления льда X = 340 кДж/кг, удельная теплоем- кость льда сл = 2,1 кДжДкг • К), удельная теплоемкость воды св = 4,2 кДжДкг • К). Найти начальную температуру Тл льда в первом калори- метре. 2.30* . В теплоизолированном цилиндрическом сосуде под легким поршнем находится смесь равных масс воды и льда: m = mB = тл = 1 кг. Давление на поршень медленно увеличивают от начального значения р0 = Ю5 Па до = 2,5 • 106 Па. Удель- ные теплоемкости воды и льда равны св = 4,2 кДжДкг • К), сл = 2,1 кДжДкг • К), удельная теплота плавления льда X = 340 кДж/кг, плотность льда рл = 0,9 рв (рв — плотность воды). Определите, какая масса Ат льда при этом растает и какую работу А совершит внешняя сила. Известно, что для уменьше- ния температуры плавления льда на 1 °C нужно довести давле- ние до значения р = 14-106 Па, а для уменьшения объема неко- торой массы воды на 1% давление нужно поднять до р'= = 20 106 Па. 1) Решите задачу, считая воду и лед несжимаемыми. 2) Оцените поправку, которую дает учет сжимаемости, счи- тая сжимаемость льда равной половине сжимаемости воды. 2.31. Известно, что если обычную воду подсолить, то тем- пература ее кипения станет выше. Определите, как при этом изменится плотность насы- щенных водяных паров при температуре кипения. 2.32. Для многих веществ существует такое значение темпе- ратуры Ттр и давления ртр, при котором все три фазы вещества (газообразная, жидкая и твердая) находятся в равновесии друг с другом — так называемая тройная точка вещества. Например, для воды Ттр = 4- 0,0075 °C, ртр = 4,58 мм рт. ст. Удельная те- плота испарения воды в тройной точке равна q = = 2,48 • 103 кДж/кг, удельная теплота плавления льда X = = 0,34 • 103 кДж/кг. Найдите удельную теплоту сублимации воды v (т. е. прямо- го перехода из твердого в газообразное состояние) в тройной точке. 2.33. Известно, что давление насыщенного пара над водным раствором сахара меньше, чем над чистой водой, где оно равно Рнас> на величину Ар = 0,05рнасс, где с — молярная концентрация раствора. Цилиндрический сосуд, наполненный до высоты hv = = 10 см раствором сахара с концентрацией с± = 2-10"3, поме- щают под широкий колпак. На горизонтальную поверхность под колпаком налит тот же раствор с концентрацией с2 = 10”3, его уровень h2«hi (рис. 66). 37
Определите уровень h раствора в цилиндрическом сосуде после установления равновесия. Температура поддерживается постоянной и равной 20 °C. Пар над поверхностью раствора со- держит только молекулы воды, молярная масса водяных паров равна ц = 18 • 10"3 кг/моль. 2.34. Вертикальная длинная кирпичная труба заполнена чу- гуном. Нижний конец трубы поддерживается при температуре > Тпл .(Тпл — температура плавления чугуна), верхний — при температуре Т2 < Тпл. Теплопроводность у расплавленного (жидкого) чугуна в к раз больше, чем у твердого. Определите, какая часть трубы занята расплавленным металлом. 2.35* . Оболочка космической станции представляет собой зачерненную сферу, температура которой в результате работы аппаратуры внутри станции поддерживается равной Т = 500 К. Количество теплоты, выделяемое единицей площади поверхно- сти, пропорционально 4-й степени термодинамической темпе- ратуры. Определите температуру оболочки Тх, если станцию окру- жить тонким черным сферическим экраном почти такого же радиуса, как и радиус ее оболочки. 2.36. В ведре находится смесь воды со льдом массой ш = = 10 кг. Ведро внесли в комнату и сразу же начали измерять температуру смеси. Получившаяся зависимость темпера- туры от времени Т(т) изображена на рис. 67. Удельная теплоемкость воды равна св = 4,2 Дж/ (кг • К), удельная теплота плавления льда X = 340 кДж/кг. Определите массу тл льда в ведре, когда его внесли в ком- нату. Теплоемкостью ведра пренебречь. 2.37* . Для исследования свойств нелинейного резистора был произведен ряд экспериментов. Сначала была исследована 38
зависимость сопротивления резистора от температуры. При повышении температуры до 7\ = 100 °C мгновенно происходил скачок сопротивления от значения Rt = 50 до R2 = 100 Ом; при охлаждении обратный скачок происходил при температуре Т2 = 99 °C. Затем к резистору приложили постоянное напряже- ние Ux = 60 В, при котором его температура оказалась равной Т3 = 80 °C. Наконец, когда к резистору приложили постоянное напряжение U2 = 80 В, то в цепи возникли самопроизвольные колебания тока. Температура воздуха в лаборатории постоянна и равна То = 20 °C. Теплоотдача от резистора пропорциональна разно- сти температур резистора и окружающего воздуха, теплоем- кость резистора С = 3 Дж/К. Определите период Т этих колебаний, а также максималь- ное и минимальное значения силы тока. 2.38. Когда летом после сухой и жаркой погоды идет дождь и капли попадают на стену из красного кирпича, слышится шипение. Объясните, почему? 2.39. Тонкая U-образная, запаянная с одного конца трубка состоит из трех колен длиной по / = 250 мм каждое, согнутых под прямыми углами. Вертикальные части трубки заполнены ртутью до половины (рис. 68). Медленно нагревая в запаянной трубке газ, отделенный от атмосферы ртутью, можно вытес- нить из трубки всю ртуть. Определите, какую работу А совершит при этом газ в труб- ке, полностью вытеснив ртуть. Атмосферное давление равно р0 = 105 Па, плотность ртути ррт=13,6-103 кг/м3, поперечное сечение трубки 5=1 см2. 2.40. Остаточную деформацию упругого стержня можно грубо описать в рамках следующей модели. Если растяжение 39
стержня Л/ < х0 (гДе хо — заданная для данного стержня вели- чина), то сила, необходимая для того, чтобы вызвать растяжение Л/, определяется законом Гука F = /cA/, где /с — жесткость стержня. Если Д/ > х0, то сила перестает зависеть от растяже- ния (вещество стержня начинает «течь»). Если теперь начать снимать нагрузку, то удлинение стержня будет спадать по пути CD, который для простоты считаем прямым и параллельным участку АВ (рис. 69). Поэтому при полном снятии нагрузки стержень остается деформированным (точка D на рисунке). । Пусть стержень вначале растянут : на Д/ = х > xQ, а затем нагрузку • I * | убирают. ___** . ••• jp §- Определите максимальное из- / ** J EZ менение ДТ температуры стерж- — — — —!-------— — ня, если его теплоемкость равна - — — — — - q Стержень теплоизолирован. Рис. 70 2.41. Тонкостенный заполнен- ный газом цилиндр массой т, высоты h и площадью основания S плавает в воде (рис. 70). В результате потери герметичности в нижней части цилиндра его глубина погружения увеличилась на ДЛ. Определите начальное давление рг газа в цилиндре. Атмо- сферное давление равно р0, температура не меняется. 2.42. Ударная волна представляет собой область повышен- ного давления, распространяющуюся в положительном напра- влении оси х с большой скоростью v. В момент прихода вол- ны давление резко повышается. Эта зависимость изображена на рис. 71. Определите, какую скорость и приобретает клин сразу по- сле прохождения через него фронта ударной волны. Масса клина равна т, размеры указаны на рис. 72. Трением прене- бречь. Считать, что приобретаемая клином скорость много меньше скорости волны (и « v).
3. ЭЛЕКТРИЧЕСТВО И МАГНЕТИЗМ
В задачах этого раздела, там, где необходимо, считать электри- ческую постоянную £0 заданной. 3.1. Между двумя разноименными точечными зарядами + и ~^2 помещают тонкий диэлектрический стержень (рис. 73). Как изменятся силы, дейст- вующие на заряды? Рис. 74 © ?2 © 7/ Рис. 73 3.2. Линия напряженности выходит из положительного то- чечного заряда + qx под углом а к прямой, соединяющей его с отрицательным точечным зарядом — q2 (рис. 74). Под каким углом р линия напряженности войдет в заряд ~«2? 3.3. Найдите напряженность Е электрического поля в цен- тре полусферы, создаваемую зарядами, распределенными рав- номерно с плотностью и по всей поверхности этой полусферы. 3.4. Напряженность электрического поля, создаваемая за- рядами, равномерно распределенными по поверхности полу- сферы, в центре О этой полусферы равна Ео. Двумя плоскостями, проходящими через один и тот же диаметр и со- ставляющими друг с другом угол а, от этой полусферы отделе- на часть поверхности (рис. 75). Найдите напряженность Е электрического поля в той же точке О, создаваемую зарядами, находящимися на отделенной части поверхности (на «дольке»). Рис. 76 3.5* . Два плоских конденсатора расположены перпендику- лярно общей оси. Расстояние между конденсаторами d много больше размеров их пластин и расстояния I между ними. Оба конденсатора заряжены — заряд первого конденсатора qu а второго q2 (рис. 76). Найдите силу взаимодействия F конденсаторов. 42
3.6. Найдите силу взаимодействия F двух соприкасающихся по всей поверхности полусфер радиуса R, если одна из них рав- номерно заряжена с поверхностной плотностью а другая п2. 3.7. Известно, что минимальная напряженность однородно- го электрического поля, которое разрывает на две части прово- дящую незаряженную тонкостенную сферу, равна Ео Определите минимальную напряженность Et поля, которое разорвет сферу вдвое большего радиуса, если толщина ее сте- нок остается постоянной. 3.8. Три небольших одинаковых незаряженных металличе- ских шарика находятся в вершинах равностороннего треуголь- ника. Шарики поочередно по одному разу соединяют с уда- ленным большим заряженным про- —------ водящим шаром, центр которого находится на перпендикуляре, вое- / ставленном к плоскости треуголь- / ^<7 \ ника и проходящем через центр по- / / | след него. В результате на первом 1 I шарике оказался заряд </ь а на вто- \ ( р) ] ром - заряд q2. \ / Определите заряд q3 третьего 'Ч \ / шарика. _____ 3.9. Металлический шар радиуса рис 77 Гр заряженный до потенциала <рь окружают проводящей тонкостенной сферической оболочкой радиуса г2 (рис. 77). Определите потенциал <р2 шара после того, как шар будет на некоторое время соединен проводником с оболочкой? 3.10. Очень маленькая заземленная проводящая сфера нахо- дится на расстоянии а от точечного заряда и на расстоянии b от точечного заряда q2 (а < Ь). В некоторый момент сфера начинает расширяться гак. что ее радиус растет по закону R — vt. Определите зависимость силы тока от времени I(t) в про- воднике, осуществляющем заземление. Считать, что точечные заряды и центр сферы неподвижны и в соответствующие мо- менты времени исходные точечные заряды попадают внутрь расширяющейся сферы, не касаясь ее (через небольшие отвер- стия). 3.11. Три незаряженных конденсатора, емкости которых равны С19 С2 и С3, соединены, как показано на рис. 78, и под- ключены к точкам Л, В и D. Потенциалы этих точек равны фл, Фв и <pD. Определите потенциал ф0 общей точки О. 4?
3.12* . Толщина плоского листка металлической фольги рав- на d, площадь листка S. Расстояние отпекоторого заряда q до центра листка равно /, причем d «]/S « I. Определите силу F, с которой листок притягивается к заря- ду q. Считать, что прямая, соединяющая заряд с центром лист- ка, перпендикулярна поверхности листка. 3.13. К какой паре точек схемы, изображенной на рис. 79, надо подключить источник тока, чтобы зарядить все шесть конденсаторов, емкости которых равны? 3.14. Плоский конденсатор заполнен диэлектриком, прони- цаемость которого зависит от напряжения на конденсаторе по закону 8 = аГ7, где а = 1 В-1. Параллельно этому «нелинейно- му» конденсатору, который не заряжен, подключают такой же конденсатор, но без диэлектрика, который заряжен до напряже- ния Uq = 156 В. Определите напряжение U, которое установится на конден- саторах. 3.15. Два небольших шарика массой т, несущие одина- ковый заряд q каждый, соединены непроводящей нитью длины 21, В некоторый момент времени середина нити начинает дви- гаться с постоянной скоростью v, перпендикулярной направле- нию нити в начальный момент времени. Определите, на какое минимальное расстояние d сблизятся шарики. 3.16. Два шарика с зарядами qr и q2 имели вначале одина- ковые по модулю и направлению скорости. После того как на некоторое время было включено однородное электрическое по- ле, направление скорости 1-го шарика повернулось на 60°, а модуль скорости уменьшился вдвое. Направление скорости 2-го шарика повернулось на 90°. Во сколько раз изменилась скорость 2-го шарика? Опреде- лите модуль отношения заряда к массе для 2-го шарика, если для 1-го он равен kt. Электростатическим взаимодействием шариков пренебречь. 44
3.17. В вершинах правильного 1977-угольника со стороной а были закреплены небольшие одинаковые шарики с равными зарядами. В некоторый момент времени один из шариков был освобожден, а через достаточно большой промежуток времени был освобожден шарик, соседний с 1-ым освобожденным. Ока- залось, что на достаточно большом расстоянии от многоуголь- ника кинетические энергии отпущенных шариков различаются на величину К. Найдите заряд q каждого шарика. 3.18. Почему ударную ионизацию (ионизация в результате соударения) атомов производят электроны, а не ионы, хотя те и другие приобретают в ускоряющем поле одинаковую кинети- ческую энергию nw2/2 = е Аф (е — заряд частиц, Аф — разность потенциалов ускоряющего поля). Считать, что после соударе- ния ионизируемый атом и налетевшая на него частица имеют приблизительно одинаковые скорости. 3.19. Два небольших одинаковых шарика, лежащие на гори- зонтальной плоскости, соединены невесомой пружиной. Один из шариков закреплен в точке О, другой свободен. Каждый из шариков одинаково зарядили, в результате чего пружина рас- тянулась в два раза. Определите, во сколько раз изменилась частота гармониче- ских колебаний системы. 3.20. Два небольших шарика, имеющих одинаковые массы и заряды и находящихся на одной вертикали на высотах ht и h2, бросили в одну сторону в горизонтальном направлении с одинаковыми скоростями v. Шарик 1-й коснулся земли на расстоянии I от вертикали бросания. На какой высоте Н2 в этот момент будет 2-й шарик? Со- противлением воздуха и влиянием индуцированных на земле зарядов пренебречь. 3.21. Моток голой проволоки, состоящей из семи с полови- ной витков, растянут между двумя вбитыми в доску гвоздями, к которым прикреплены концы прово- локи. Подключив к гвоздям приборы, измерили сопротивление цепи между гвоздями. Определите, во сколько раз изменит- ся это сопротивление, если моток раз- мотать, оставив концы присоединен- ными к гвоздям. 3.22. Пять одинаковых сопротивле- ний (спиралей для электрических пли- ток) включены по схеме, указанной на рис. 80. 45
Как изменится накал правой верхней спирали, если замк- нуть ключ К? 3.23. Как изменится сопротивление цепи, состоящей из пяти одинаковых проводников, если добавить еще два таких же про- водника. как показано штриховой линией на рис. 81? 3.24. Проволочный каркас в виде тетраэдра ADCB подклю- чен к источнику постоянного тока (рис. 82). Сопротивления всех ребер 1страэдра одинаковы. Определите, исключение какого из ребер каркаса приведет к наибольшему изменению тока I в цепи. Чему равно это мак- симальное изменение тока Д1тах? Сопротивлением подводящих проводов пренебречь. 3.25. В схеме, изображенной на рис. 83, сопротивления всех резисторов одинаковы и равны R. Напряжение на клеммах рав- но L. Определите силу тока I в подводящих проводах, если их со- противлением можно пренебречь. 3.26* . Определите сопротивление цепи RAB между точками А и В каркаса составленного из девяти одинаковых проволо- чек сопротивлением R каждая (рис. 84). 3.27. Найдите сопротивление RAB между точками А и В кар- каса. изготовленного из тонкой однородной проволоки (рис. 85) Число последовательно вложенных равносторонних треугольников (стороны каждого последующего уменьшаются в два раза) считать стремящимся к бесконечности. Сторону АВ принять равной а, сопротивление единицы длины проволоки р. 3.28. Схема из резисторов состоит из очень большого (бес- конечною,1 числа звеньев (рис. 86). Сопротивления резисторов каждою последующего звена в к раз отличаются от сопроти- вления резисторов в предыдущем звене. Найдите сопротивление RAB между точками А и В, если со- противления в первом звене равны и R2. 3.29 Для регулирования напряжения на нагрузке собрана схема, изображенная на рис. 87.Сопротивления нагрузки и регу- лировочного реостата равны R. Нагрузка подключена к поло- вине реостата. Входное напряжение неизменно и равно U. Определите, как изменится напряжение на нагрузке, если ее сопротивление увеличить в два раза. 3.30. Имеются два разных амперметра. Отклонение стрелки в каждом амперметре пропорционально силе тока, а шкалы имеют равномерные деления. К первому амперметру подклю- чен резистор с сопротивлением Rl9 ко второму амперметру — резистор с неизвестным сопротивлением Rx. Сначала ампер- метры соединяют последовательно и подключают к точкам А и В (как показано на рис. 88). Отклонения стрелок в ампер- 46
Рис. 81 R R В Рис. 84 Рис. 86 Рис. 87 47
метрах в этом случае равны пг и п2. Затем эти амперметры со- единяют параллельно и подключают к точкам А и В (как пока- зано на рис. 89). Отклонения стрелок во втором случае равны п[ и п2. Рис. 88 Рис. 89 Определите неизвестное сопротивление второго рези- стора. 3.31. Имеются две проволоки одинаковой длины, но разно- го квадратного сечения, сделанные из одного и того же мате- риала. Сторона сечения первой проволоки dr = 1 мм, второй d2 = 4 мм. Для того чтобы расплавить первую проволоку, че- рез нее нужно пропустить ток Ц = 10 А. Определите силу тока 12, который нужно пропустить через вторую проволоку, чтобы она расплавилась. Считать, что коли- чество теплоты, уходящее в окружающую среду за 1 секунду, подчиняется закону Q = kS(T— Тср), где S' - площадь поверхно- сти проволоки, Т— ее температура, Тср — температура окру- жающей среды вдали от проволоки, к — коэффициент пропор- циональности, одинаковый для обоих образцов. 3.32. В схеме, изображенной на рис. 90, переключатель К может находиться в положениях 1 и 2. В цепь включены два источника постоянного тока, два резистора и амперметр. Э. д. с. одного источника тока равна э. д. с. другого не- известна. Внутреннее сопротив- ление источников тока равно нулю. Сопротивление резисто- ров также неизвестно. Сопро- тивление одного резистора яв- ляется переменным, и его под- бирают так, чтобы ток через амперметр при двух положениях переключателя был одинаковым; этот ток измеряют, и он оказывается равным I. Определите сопротивление, обозначенное на схеме через Rx. 3.33. Может ли сила тока, протекающего через резистор, увеличиться, если замкнуть накоротко один из источников то- 48
ка, например с э. д. с. $2, как показано на рис. 91? Параметры элементов схемы считать заданными. 3.34* . Скрытая схема, состоящая из резисторов («черный ящик»), имеет четыре вывода (рис. 92). Если к зажимам 1 и 2 подвести напряжение, то при разомкнутых клеммах 3 и 4 вну- три схемы выделяется мощность = 40 Вт, а при замкнутых клеммах 3 и 4 N2 = 80 Вт. Если к тому же источнику подклю- чить клеммы 3 и 4, то при разомкнутых клеммах 1 и 2 в схеме выделяется мощность N3 = 20 Вт. Определите мощность N4, которую будет потреблять схема при замкнутых клеммах 1 и 2, когда то же напряжение будет приложено к клеммам 3 и 4. 3.35. В схеме, изображенной на рис. 93-» определите силу протекающего через батарею тока:, в первый момент времени после замыкания ключа К; спустя большой промежуток вре- мени. Параметры элементов схемы считать заданными. 3.36. Ключ К (рис. 94) замыкают поочередно с каждым из контактов на малые одинаковые промежутки времени, так что изменение заряда конденсатора за время каждого замыкания мало. Какой заряд qyci установится на конденсаторе? 3.37. В электрическую цепь включены источник тока с э. д. с. % и внутренним сопротивлением г, конденсаторы ем- костью Сг и С2 и резисторы сопротивлением и R2 (рис. 95). Найдите напряжения U t и U2 на каждом конденсаторе. 3.38* . Между точками Е и F схемы, изображенной на рис. 96, включают сначала идеальный вольтметр, а затем идеальный амперметр; их показания соответственно равны Uo и /0- Определите силу тока I, который будет течь через резистор сопротивлением R, включенный между точками Е и F. 3.39. Пластина А плоского конденсатора неподвижна, пластина 15 прикреплена к стенке пружиной и может двигаться, оставаясь параллельной пластине А (рис. 97). После замыкания ключа К пластина 15 начала двигаться и остановилась в новом положении равновесия. При этом начальное равновесное рас- стояние d между пластинами конденсатора (когда пружина не растянута) уменьшилось на 10%. На сколько изменилось бы равновесное расстояние между пластинами, если бы ключ К замкнули на короткое время? Считать, что за это время пластина Б не успевает заметно сдвинуться. 3.40. Из однородной проволоки постоянного сечения соста- влена показанная на рис. 98 цепь. 49
Рис. 91 Рис. 92 Рис. 94 50
Найдите отношение количества теплоты 612/634» выделяю- щееся в единицу времени на участках 1—2 и 3 — 4. 3.41. Напряжение между анодом и катодом вакуумного диода равно U, анодный ток равен I. Найти среднее давление рср электронов на анод пло- щадью S. 3.42. На клеммы АВ (рис. 99) подается такое меняющееся во времени напряжение, что напряжение на обкладках конден- сатора меняется по закону, представленному на рис. 100. Нарисуйте график зависимости напряжения от времени на клеммах CD. 3.43. Две батареи с э. д. с. и F2, конденсатор емкостью С и резистор сопротивлением R соединены, как показано на рис. 101. Определите количество теплоты Q, выделяющееся на рези- сторе после переключения ключа К. 3.44. Электрическая цепь составлена из источника тока с э. д. с. F и внутренним сопротивлением г и двух подклю- ченных параллельно к источнику тока резисторов (рис. 102). Сопротивление одного из резисторов R{ неизменно, а сопроти- вление другого R2 можно подобрать так, чтобы выделяемая в этом резисторе мощность была максимальной. Найдите значение R2, соответствующее этой максимальной мощности. 3.45. Конденсатор емкостью Сг разряжается через резистор сопротивлением R. Когда сила тока разряда достигает значе- ния 10, ключ К размыкают (рис. 103). Найдите количество теплоты Q, которое выделится на рези- сторе, начиная с этого момента времени. 3.46. Батарея с э. д. с., равной F, конденсаторы емкостями и С2 и резистор сопротивлением R соединены, как показано на рис. 104. Найдите количество теплоты Q, выделяющееся на резисторе после переключения ключа К. 3.47. В схеме, показанной на рис. 105, перед замыканием ключа К конденсатор емкостью С не был заряжен. Ключ за- мыкают на некоторое время, в течение которого конденсатор зарядился до напряжения U. Определите, какое количество теплоты 62 выделится за это время на резисторе сопротивлением R2. Э. д. с. источника тока равна F, его внутренним сопротивлением пренебречь. 3.48. По горизонтальным параллельным рельсам, расстоя- ние между которыми равно d, может скользить без трения перемычка, масса которой равна т. Рельсы соединены ре- зистором сопротивлением R и помещены в вертикальное 51
Ло----1[ So----- <>С ой Рис. 99 К R Рис. 101 Рис. 104 Рис. 105 52
однородное магнитное поле, индукция которого равна В. Перемычке сообщают скорость vQ (рис. 106). Найдите путь s, пройденный перемычкой до остановки. Как зависит ответ от направления индукции В? 3.49. Как будут зависеть от времени показания гальвано- метра, включенного в цепь расположенного горизонтально кругового контура, если вдоль оси этого контура будет падать заряженный шарик? 3.50. В однородном постоянном во времени магнитном по- ле, индукция которого В направлена вверх, движется подве- шенный на нерастяжимой нити длины / маленький заряженный шарик. Масса шарика равна т, заряд q, период обращения Т. Найдите радиус г окружности, по которой движется шарик, ес- ли нить все время натянута. 3.51. В однородном магнитном поле с индукцией В с по- стоянной скоростью v движется металлический шарик радиу- са г. Укажите точки шарика, разность потенциалов Дфтах между которыми будет максимальна, и определите эту разность по- тенциалов. Считать, что направление скорости составляет с на- правлением магнитной индукции угол а. 3.52. По обмотке длинного цилиндрического соленоида ра- диуса R протекает постоянный ток, создающий внутри соле- ноида однородное магнитное поле с индукцией В. Между вит- ками соленоида в него влетает по радиусу (перпендикулярно оси соленоида) электрон со скоростью v (рис. 107). Отклоняясь в магнитном поле, электрон спустя некоторое время покинул соленоид. Определите время t движения электрона внутри соленоида. 3.53. По двум параллельным металлическим направляю- щим, наклоненным под углом а к горизонту и расположенным на расстоянии b друг от друга, может скользить без трения ме- таллическая перемычка массой т. Направляющие замкнуты снизу на незаряженный конденсатор емкостью С, и вся кон- струкция находится в магнитном поле, индукция которого 53
В направлена вертикально. В начальный момент перемычку удерживают на расстоянии / от основания «горки» (рис. 108). Определите время г, за которое перемычка достигнет осно- вания «горки» после того, как ее отпустят. Какую скорость v¥ она будет иметь у основания? Сопротивлением направляющих и перемычки пренебречь. 3.54* . Квадратная недеформируемая сверхпроводящая рам- ка массой т со стороной а расположена горизонтально и нахо- дится в неоднородном магнитном поле, индукция которого ме- няется в пространстве по закону Вх = — ах, Ву = 0, Bz = az -I- Во (рис. 109). Индуктивность. рамки равна L. В начальный момент време- ни центр рамки совпадает с началом координат О, а стороны параллель- ны осям х и у. Ток в рамке в этот момент равен нулю. Рамку отпускают. Как она будет двигаться и где окажется спустя время t после начала движения? 3.55. Длинная цилиндрическая ка- тушка, намотанная на каркас диамет- ром Di, имела индуктивность При подключении катушки к источнику тока внутри нее создавалось магнитное поле с индукцией Потом катушку решили переделать. Ее размотали и тот же провод намотали на каркас диаметром В2- Индук- тивность катушки стала L2. Определите индукцию В2 магнитного поля внутри новой катушки при подключении ее к тому же источнику тока. Счи- тать, что длина провода намного больше длины катушек. 3.56* . Две длинные цилиндрические катушки с равномерной намоткой одинаковой длины и почти одинакового радиуса имеют индуктивности и L2. Их вставили друг в друга (соос- но) и присоединили к цепи так, как показано на рис. ПО. На- правление тока в цепи и в витках катушек показано стрелками. Найдите индуктивность L такой составной катушки. 3.57. Лебедка приводится в движение электродвигателем с независимым возбуждением, питающимся от батареи с э. д. с. = 300 В. Без груза конец троса с крюком поднимается со скоростью Vi =4 м/с, с грузом массой т='10 кг со скоростью v2 = 1 м/с. Определите, с какой скоростью v' будет двигаться груз и какова должна быть его масса т', чтобы лебедка развивала максимальную мощность. Массой троса с крюком пренебречь. 3.58. Схема с идеальным диодом включена в сеть перемен- ного тока (рис. 111). 54
Определите, в каких пределах изменяется напряжение ме- жду точками А и К. 3.59. Конденсатор неизвестной емкости, катушка индуктив- ностью L и резистор сопротивлением R подключены к источ- нику переменного напряжения % = ^Ocos(ot (рис. 112). Сила то- ка в цепи равна I = (^0/К) cos art. Определите амплитуду напряжения Uo между обкладками конденсатора. Рис. 113 3.60. Под действием постоянного напряжения U конденса- тор емкостью С=10-11 Ф, входящий в схему, указанную на рис. ИЗ, заряжается до заряда qx = 10"9 Кл. Индуктивность катушки равна L= 10”5 Гн, сопротивление - резистора R = 100 Ом. Определите амплитуду установившихся колебаний заряда q0 конденсатора при резонансе, если амплитуда внешнего сину- соидального напряжения равна Uo = U. 3.61. Батарея из двух последовательно - соединенных конденсаторов емкостью С каждый заряжена до напряжения U и в начальный момент времени подключена к катушке индуктив- ностью Ц так что образовался колебательный контур (рис. 114). Спустя интервал времени т один из конденсаторов пробивается и сопротивление между его обкладками становит- ся равным нулю. Найдите амплитуду колебаний заряда qQ на непробитом конденсаторе. 3.62. Как можно избежать аварии, связанной с перегора- нием обмотки сверхпроводящего соленоида? 55
4. ОПТИКА
4.1. Точечный источник S находится на оси полого конуса с зеркальной внутренней поверхностью (рис. 115). С помощью собирающей линзы на экране Э получают изображение источ- ника, создаваемое лучами, однократно отраженными от зер- кальной поверхности конуса (прямые лучи от источника на линзу не попадают). Рис. 117 Что произойдет с изображением, если линзу закрыть диа- фрагмами, такими, как на рис. 116 и 117? 4.2. На достаточно удаленные предметы смотрят через со- бирающую линзу с фокусным расстоянием F = 9 см, распола- гая глаз на расстоянии а = 36 см от линзы. Оцените минимальный размер экрана, который нужно рас- положить за линзой так, чтобы он перекрыл все поле изобра- жения. Где следует расположить экран? Считать, что зрачок глаза равен г « 1,5 мм. 4.3* . Цилиндрический прозрачный сосуд высоты I Rc — радиус сосуда) заполнен идеальным газом с молярной массой ц, температурой Т под давлением р0. Зависимость пока- зателя преломления п газа от его плотности р удовлетворяет соотношению п = 1 4- ар. Сосуд привели во вращение с угловой скоростью со вокруг оси. Вдоль оси на сосуд падает узкий па- раллельный световой пучок радиуса гп. Определите радиус R пятна на экране, расположенном пер- пендикулярно оси сосуда за ним на расстоянии L. Считать, что изменение давления газа в каждой точке сосуда вследствие вра- щения мало по сравнению с р0. Влиянием торцов сосуда на ход световых лучей пренебречь. 4.4. Подзорная труба, имеющая угловое увеличение к = 20, состоит из двух собирающих тонких линз — объектива с фо- кусным расстоянием F = 0,5 ми окуляра, который можно под- страивать по глазу в пределах от D_ = —7 до D+ = +10 дптр 57
(при подстройке окуляр перемещается относительно объек- тива). Начиная с какого минимального расстояния а от объектива можно рассматривать удаленные предметы ненапряженным нормальным глазом при помощи этой трубы? 4.5. Можно ли с помощью рассеивающей линзы увеличить освещенность некоторых участков поверхности экрана? 4.6. Точно над карандашом, расположенным вертикально над водой, находится точечный источник света. На дне сосуда с водой видна тень карандаша. Если карандаш опускают в во- ду, то, когда он входит в нее, размер темного пятна увеличи- вается. Если затем карандаш вытаскивают из воды, то на месте темного появляется светлое пятно. Объясните описанные явления. 4.7. Если смотреть на освещенную поверхность через широ- кое отверстие корпуса шариковой ручки, то вокруг узкого от- верстия в корпусе видно несколь- ко концентрических темных и свет- лых колец. _____________________I Объясните, почему наблю- ^1___________________J даются эти кольца. I___________________11 4.8. Точечный источник света 5 ________Z_________находится на расстоянии I = 1 м от экрана. В экране напротив источ- ника сделано отверстие диаметром Рис. 118 d = 1 см, в которое проходит свет. Между источником и экраном помещен прозрачный цилиндр (рис. 118), показатель прелом- ления которого равен п = 1,5, длина / = 1 м, а диаметр тот же, что и у отверстия. Как изменится световой поток через отверстие? Поглоще- нием света в веществе пренебречь. 4.9. Объектив и окуляр оптической трубы представляют со- бой двояковыпуклые симметричные линзы, изготовленные из стекла с показателем преломления ист = 1,5. Труба настроена на бесконечность, при этом расстояние между объективом и оку- ляром равно Lq = 16 см. Определите расстояние Ц на котором должны находиться объектив и окуляр трубы, настроенной на бесконечность, если между окуляром и объективом будет налита вода (ив = 1,3). 4.10. На поверхности стеклянного шара находятся паук и муха. Где на поверхности шара должна находиться муха, чтобы паук смог ее увидеть? Считать, что радиус шара много больше размеров паука и мухи. Показатель преломления для стекла равен мст = 1,43. 58
4.11. Точечный источник света S' расположен вне цилиндра на его оси вблизи торца (основания). Найдите минимальный показатель преломления п материа- ла цилиндра, при котором ни один луч, вошедший через осно- вание, не выйдет через боковую поверхность наружу. 4.12. В концы трубы, внутренняя боковая поверхность кото- рой зачернена, вставлены две собирающие линзы. Диаметры линз равны диаметру трубы. Фокусное расстояние одной линзы в два раза больше фокусного расстояния другой. Линзы нахо- дятся на таком расстоянии друг от друга, что параллельные лучи света, падающие вдоль оси трубы на одну линзу, выходят из второй линзы также параллельным пучком. Когда широкий пучок света падает на линзу с большим фокусным расстоя- нием, то на экране получается светлое пятно с освещенностью Теперь перевернем трубу так, чтобы свет падал на линзу с меньшим фокусным расстоянием. На экране получается свет- лое пятно с освещенностью Е2. Определите, во сколько раз изменится освещенность на экране. 4.13. Начинающий фотограф, знаток геометрической опти- ки, фотографировал с некоторой выдержкой фасад здания с расстояния 100 м. Подойдя к зданию на расстояние 50 м, чтобы получить снимок в большем масштабе, он решил, зная, что площадь изображения увеличится в четыре раза, соответ- ственно увеличить в четыре раза и выдержку. Дома, проявив пленку, он обнаружил, что первый снимок получился хорошо, для второго же выдержка оказалась неправильной. Определите, во сколько раз нужно было на самом деле из- менить выдержку и почему. 4.14* . На одном берегу залива, который образует клин с углом а, живет рыбак; его дом находится в точке А (рис. 119). Расстояние от точки А до ближайшей к ней точки залива С равно h, а расстояние до конца залива, т. е. до точки D, рав- но I. На другом берегу залива в точке В находится дом прияте- ля рыбака. Точка В расположена симметрично (относительно залива) точке А. В распоряжении рыбака имеется лодка. Определите минимальное время t, необходимое рыбаку, чтобы он из своего дома смог добраться до дома приятеля при условии, что рыбак может двигаться по суше со скоростью г и плыть по заливу в лодке со скоростью, в два раза меньшей (И = 2). 4.15. Изображение точечного источника S', находящееся от прозрачного шара на расстоянии Ь, создано благодаря неболь- шой диафрагме только лучами, близкими к оптической оси (рис. 120). 59
Где будет изображение, если распилить шар пополам пер- пендикулярно горизонтальной оси и плоскую поверхность ле- вой половины посеребрить? 4.16. В днище судна сделан стеклянный иллюминатор для наблюдения за морскими животными. Диаметр иллюминатора D = 40 см много больше толщины стекла. Определите площадь S обзора дна из такого иллюминато- ра. Показатель преломления воды равен пв = 1,4, расстояние до дна h = 5 м. 4.17* . Предположим, что ваш собеседник, с которым вы разговариваете, сидя напротив него за столом, носит очки. Сможете ли вы определить, каким дефектом зрения — даль- нозоркостью или близорукостью — он обладает? Естественно, как воспитанный человек, вы при этом не станете просить собе- седника дать вам примерить его очки и вообще не будете заво- дить о них разговор. 4.18. Человек идет по прямой, образующей угол а с пло- скостью зеркала, со скоростью v. Определите, с какой скоростью иотн он приближается к свое- му изображению. Считать предмет и его изображение симме- тричными относительно плоскости зеркала. 4.19. На сферическое зеркало радиуса R = 5 см падают па- раллельно его оптической оси два луча — один проходит от оси на расстоянии hr = 6,5 см, другой на расстоянии h2 = 3 см. Определите расстояние Ах между точками, в которых эти лучи пересекают оптическую ось после отражения от зеркала. 4.20. Внутренняя поверхность конуса, покрытая отражаю- щим слоем, образует коническое зеркало. Вдоль оси конуса внутри него натянута тонкая светящаяся нить. Определите минимальный угол а раствора конуса, при ко- тором лучи, идущие от нити, будут отражаться от поверхности конуса не более одного раза. 60
РЕШЕНИЯ 1. МЕХАНИКА ».Х
1.1. Предположим сначала, что трение отсутствует. Тогда вслед- ствие закона сохранения энергии скорость тела v после спуска по на- клонной плоскости с высоты h равна скорости, которую нужно сооб- щить телу, чтобы оно поднялось на ту же высоту h. Поскольку при подъеме и спуске ускорение по модулю одинаковое, время подъема бу- дет равно времени спуска. При учете силы трения скольжения скорость vr тела после спуска в конце пути меньше скорости v (из-за работы силы трения), а ско- рость v2, которую нужно сообщить телу для подъема, больше v по той же причине. Поскольку спуск и подъем происходят с постоянными (хо- тя и разными) ускорениями, а пройденные пути одинаковы, то время спуска tr и время подъема t2 найдем из уравнений s = fjti/2, s — v2t2/2, где s — длина проходимого вдоль наклонной плоскости пути. Так как выполняется неравенство vt < v2, то легко получим, что > t2. Таким образом, при наличии силы трения скольжения время спуска с высоты h больше времени подъема на ту же высоту. При решении задачи не принималась во внимание сила сопротив- ления воздуха. Тем не менее легко показать, что если наряду с силой тяжести и силой нормальной реакции наклонной плоскости имеется сила сопротивления воздуха, то независимо от конкретного вида по- следней время спуска всегда будет больше времени подъема. Действи- тельно, если тело в процессе подъема оказалось на промежуточной вы- соте h', то, чтобы оно смогло подняться на высоту h, его скорость v' на высоте hf при наличии силы сопротивления должна быть больше, чем при движении без сопротивления — часть кинетической энергии при последующем подъеме перейдет в теплоту. При спуске с высоты h те- ло, оказавшись на высоте h't из-за работы силы сопротивления будет иметь скорость г", которая меньше скорости тела, спускающегося в от- сутствие силы сопротивления. Таким образом, проходя одну и ту же точку наклонной плоскости, на подъеме тело имеет скорость большую, чем на спуске. Поэтому небольшой участок наклонной плоскости вблизи точки h' тело на подъеме пройдет быстрее, чем на спуске. Раз- бивая весь путь тела на небольшие участки, видим, что на подъеме каждый такой участок будет пройден телом быстрее, чем на спуске. Следовательно, и общее время подъема будет меньше времени спуска. 1.2. Поскольку после начала торможения локомотив движется равнозамедленно, то он остановится через время t — v/a = 50 с, за ко- торое он пройдет путь s — v2/(2a) = 375 м. Таким образом, через 1 ми- нуту после начала торможения локомотив будет находиться на рас- стоянии I = L — s = 25 м от светофора. 1.3. В момент выключения пилотом двигателя вертолет находился на высоте h = atJ/2. Учитывая, что звук на земле перестал быть слы- 62
шен спустя время t2, получим уравнение *2 “ G atl ~2с где справа мы учли время подъема вертолета на высоту h и время, ко- торое шел звук с высоты h до земли. Решая полученное квадратное уравнение, найдем величину — + 2 —12--- а а Мы отбросили второй корень уравнения, поскольку он не имеет физи- ческого смысла. Скорость вертолета v в момент прекращения работы двигателя с учетом числовых данных задачи найдем из соотношения с а с + 2 —t2 а = j/c2 + 2act2 — с = 80 м/с. 1.4. Спустя время материальная точка, двигаясь с ускорением а, пройдет путь s = at2/2 и будет иметь скорость v = atv Выберем коор- динатную ось х, как показано на рис. 121. Здесь О — точка, из которой |-« *1 _ О А X Рис. 121 началось движение, А — та точка, где тело оказалось спустя время tv Учитывая смену знака ускорения и применяя формулу для пути при равнопеременном движении, найдем время t2i за которое тело переме- стится из точки А снова в точку О: 0 = at2/2 + atxt2 — at2/2\ отсюда t2 = tr (1 +1/2). Время t, прошедшее от начала движения до возврата в исходное положение, найдем по формуле t = ti + t2 = 11 (2 +1/2). 1.5. Будем рассматривать относительное движение тел с точки зрения 1-го тела. Тогда в начальный момент времени 1-е тело покоит- ся (оно будет находиться в покое и в последующие моменты времени), а 2-е движется ему навстречу со скоростью + v2. Его ускорение по- стоянно, равно по модулю ах + а2 и направлено против направления его начальной скорости. Условие встречи означает, что расстояние, на котором скорость 2-го тела обратится в нуль, должно быть больше, 63
чем расстояние между телами в начале движения; отсюда получаем ^nax = (^1 + 1.6. Поскольку движение шариков происходит по вертикальной прямой, направим координатную ось вертикально вверх. Построим график зависимости проекции скорости шариков от времени на эту ось — рис. 122 для 1-го шарика (v1(t)), рис. 123 для 2-го (v2(t)) (моменты начала движения пока никак не связаны между собой). Эти графики Рис. 122 будут представлять бесконечный набор одинаково наклоненных (уско- рение одинаковое) участков прямых. По оси времени эти отрезки от- стоят друг от друга: для Д-го шарика на величину =2]/2hl/g, для 2-го на величину t2 = 2|/2/i2/^. Так как по условию задачи hr =4h2, то ti = 2t2, т. е. для 2-го шарика движение будет возобновляться вдвое ча- ще. Из соотношения начальных высот следует, что максимально до- стижимые шариками скорости также будут отличаться в два раза (см. рис. 122, 123): и1тах = ^2 max = = v0- Для того чтобы скорости шариков совпали в какой-нибудь мо- мент и по модулю, и по направлению, имеется две возможности. Либо впервые скорости шариков совпали спустя т = ntY (где п = О, 1, 2, ...) от начала движения в течение интервала времени ti/4, а потом они совпали спустя время Згх/4 от начала движения и совпадали в те- чение интервала tJ2. Впоследствии с периодичностью совпадение скоростей будет продолжаться в течение интервала времени tr/2. Дру- 64
гая возможность состоит в том, что 2-й шарик начал движение спустя время т = tjl + nt1 (где п = 0, 1, 2, ...). Спустя интервал времени скорости шариков впервые совпадают, и полное совпадение скоростей длится в течение интервала времени tj/2. Далее картина повторяется с периодичностью tv При других моментах начала движения 2-го шарика из-за кратно- сти периодов повторяемости движений шариков графики скоростей не будут иметь общих точек, если их соответствующим образом «нало- жить», поэтому задача не будет иметь решений. 1.7* . Рассмотрим движение одного шарика, свободно падающего вблизи оси симметрии с высоты Я, начиная с момента его соударения с поверхностью. В момент удара шарик имеет начальную скорость v0 = y2gH (поскольку удар абсолютно упругий), направление скорости v0 составляет угол 2а с вертикалью (рис. 124). Пусть спустя время t после соуда- рения с поверхностью смещение ша- рика по горизонтали равно s, тогда Vo sin 2а • t = s. Отсюда получаем t = = s/(]/lgH sin 2a), где v0 sin 2a — гори- зонтальная составляющая начальной скорости шарика (за время t шарик больше не ударяется о поверхность). Высота, на которой будет находиться шарик спустя время г, равна Ah = h0 + v0 cos 2a • t — gt2/l, где Vq cos 2a — вертикальная составляющая начальной скорости ша- рика. Поскольку шарик начал падать с высоты Я вблизи оси симметрии (угол а мал), можно считать, что h0 « 0, sin 2a « 2a, cos 2a « 1, s « Ra. Учитывая эти и полученные выше соотношения, найдем условия попа- дания шарика в низшую точку сферической поверхности: t = s/ (^2дН sin 2a) = R/ (2 ^2gH), ДА « vot - gt2/2 = R/2 - R2/(16H) = 0. Отсюда получаем, что Я = R/8. 1.8. Так как стенка гладкая, то удар о стенку не изменяет верти- кальную составляющую скорости шарика. Поэтому полное время дви- жения шарика представляет собой полное время подъема и спуска на первоначальную высоту в поле силы тяжести тела, брошенного вверх со скоростью uosina. Следовательно, = 2i>0 sin a/g. Движение шарика по горизонтали складывается из двух участков пути: до соуда- рения со стенкой он двигался со скоростью i?0cosa; после соударения 3 А. И. Буздин и др. 65
шарик пролетел назад такой же путь, но с другой скоростью. Чтобы рассчитать скорость обратного движения шарика, заметим, что ско- рость сближения шарика и стенки (по горизонтали) была равна v0cosa-l- v. Поскольку удар абсолютно упругий, то после удара шарик будет удаляться от стенки со скоростью v0 cos qt + v, поэтому относи- тельно земли он будет иметь горизонтальную скорость (i?0 cos a + v) 4- v = cos a + 2r. Если до удара о стенку шарик летел время г, то, приравнивая пути, пройденные им до и после соударения, получим уравнение v0 cos a • t = (гi — t)(v0 cos a +‘20. Отсюда, учитывая, что полное время движения шарика равно = = 2vosina/0, получим t ~ v0 sin a (v0 cos a + (v0 cos a + v)]. 1.9* . На рис. 125 приведен вид сверху на траекторию движения шарика. Поскольку соударения шарика со стенкой и дном колодца упругие, модуль горизонтальной составляющей скорости шарика остается неизменным и равным v. Расстояния по горизонтали между \ точками, в которых происходят / 'ч \ дна последовательных соударения, | Лк а ' равны АА1 = А ХА2 = А2А3 = ... \ /а/7' ... = 2r cos а. Время между двумя \ / последовательными соударениями шарика со стенкой колодца равно ti ** 2rQostt/v. Рис. 125 Вертикальная составляющая скорости шарика при соударении со стенкой не изменяется, а при соударении с дном меняет знак на проти- воположный. Модуль вертикальной составляющей скорости при пер- вом ударе о дно равен |/2#Н, время движения от верха колодца до дна равно t2 = ^2Н/д. i^rcosaj г*----*1 Рис. 126 66
На рис. 126 представлена плоская вертикальная развертка много- гранника А1А2А3,..'9 участки траектории движения шарика внутри ко- лодца на такой развертке — параболы (целые параболы — участки траектории между последовательными ударами о дно). Шарик сможет «выбраться» из колодца, если момент максимального подъема по па- раболе совпадает с моментом соударения со стенкой (т. е. в момент максимального подъема шарик окажется в точке Ап края колодца). При этом времена и t2 будут связаны соотношением nti = 2kt2, где п и к — целые взаимно простые числа. Подставляя значения и t2, находим соотношение между v, Н, г и а, при котором шарик может «выбраться» из колодца: и г cos a/v = к ]/2Н/д. 1Д0. Из всех возможных траектории снаряда выберем ту, которая касается укрытия. Рассмотрим движение снаряда в системе координат, оси которой направлены так, как показано на рис. 127. В этой системе «горизонтальная» (вдоль оси Ах) составляющая начальной скорости снаряда равна vox = v0 cos (ф — а), а «вертикальная» (вдоль оси Ау) со- ставляющая равна voy — v0 sin (ф — а), где ф — угол, который составляет с горизонтом направление начальной скорости снаряда. Точка С, в которой траектория снаряда касается укрытия, опреде- ляет максимальную высоту h' поднятия снаряда над «горизонтом», равную, как видно из рис. 127, Z sin а. В этой точке составляющая пол- ной скорости v снаряда на ось Ау равна нулю и h' = v20y/(2g'), где д' = д cos а — «ускорение свободного падения» в системе координат хАу. Таким образом, Vq sin2 (ф — а) = 2gl cos а sin а. 3* 67
Отсюда, в частности, вытекает, что если по условию задачи Vq < 2gl cos a sin а = gl sin 2а, то ни одна из траекторий снаряда не коснется укрытия и максималь- ная дальность полета L^x будет у снаряда, пущенного под углом ф = = тс/4 к горизонту; при этом L^x = v%/g. Если по условию задачи выполняется соотношение Vq gl sin 2а, то для того, чтобы траектория касалась укрытия, снаряд должен быть пущен под углом Ф = фкас = а 4- arcsin у gl sin 2а /v0. Если при этом по условию задачи выполняется неравенство vo/(vo + < sin 2а, что в свою очередь означает выполнение условия фкас > тс/4 (покажи- те!), то угол вылета снаряда с максимальной дальностью полета равен Ф = тс/4 и Lmax = v^/g. Если же по условию задачи выполняется обрат- ное неравенство v2q/ (v2q 4- 2gl) > sin 2a, что в свою очередь означает выполнение условия фкас < тс/4, то Vgl sin 2a Ф = фкас = a 4- arcsin-----------, vo ( V 9^ sin 2a Lmax = — sin 2ф = — sin 21 a 4- arcsin------------- g g \ 1.11. Предположим, что град падает вертикально со скоростью v. В системе отсчета, связанной с автомашиной, угол падения града на Рис. 128 лобовое стекло равен углу отражения. Скорость града при этом до попада- ния на стекло будет равна v — (рис. 128). Поскольку после отражения градины (с точки зрения водителя) летят верти- кально вверх, то угол отражения, а значит, и угол падения равны рх (pi — угол на- клона лобового стекла автомашины). Сле- довательно, a 4- 2рх = тс/2, причем tga = = v/vi. Отсюда tg a = tg (тс/2 - 2pJ = = ctg2pb v/v1=ctg2p1. Поэтому для отношения скоростей двух автомашин получим условие Г1/г2 = ctg 2p2/ctg 2р! = 3. 1.12. Перейдем в систему отсчета, движущуюся вместе с точками А и В. В этой системе отсчета скорости точек А и В будут равны нулю. 68
Поскольку расстояния АС и ВС постоянны, точка С, с одной стороны, может двигаться по окружности радиуса АС с центром в точке А, с другой стороны, — по окружности радиуса ВС с центром в точке В. Поэтому направление скорости точки С должно быть перпендикуляр- но как прямой АС, так и прямой ВС. Поскольку точки А, В и С не ле- жат на одной прямой, то направление скорости точки С было бы пер- пендикулярно двум пересекающимся прямым АС и ВС, что невозмож- но. Следовательно, скорость точки С в движущейся системе равна нулю, а в исходной системе (связанной с землей) скорость точки С рав- на скорости точек А и В. Если бы точка С лежала на прямой АВ и ее скорость (в системе отсчета, связанной с листом фанеры) не была бы равна нулю, то спу- стя небольшой интервал времени либо расстояние АС, либо расстоя- ние ВС должно увеличиться, чего не может быть. Следовательно, при рассматриваемом движении листа фанеры скорости всех его точек одинаковы. 1.13. Пусть машина заехала в небольшой просвет между двумя другими автомашинами. При этом она расположена по отношению к тротуару так, как показано на рис. 129. Поставим вопрос: как легче машине теперь выехать — передним или задним ходом? Поскольку по- ворачиваются только передние колеса, то при любом маневрировании машины центр окружности О, по которой машина выезжает (вперед или назад), всегда лежит на прямой, проходящей через центры задних колес. Таким образом, при выезде задним ходом траектория машины скорее «заденет» заднюю машину, чем при выезде передним ходом (центр соответствующей окружности сдвинут назад по отношению к середине машины). Легко понять, что выезд машины — это обра- щенный во времени въезд. Поэтому заезжать в небольшой просвет следует задним ходом. 1.14* . Рассмотрим движение самолета, начиная с момента его перехода на круговую траекторию (рис. 130). По условию задачи 69
в верхней точке траектории В скорость самолета равна 14 = г0/2, по- этому радиус г описываемой самолетом окружности найдем из соотношения «о/4 = »о - 2«о • 2г, которое получается из закона движения самолета при h = 2г. Для точ- ки траектории С, где скорость самолета направлена вверх, общее уско- рение будет складываться из центростремительного ускорения ац = = vc 1Г (»с — vo — 2aor, где vc — скорость самолета в точке С) и танген- циального ускорения at (эго ускорение отвечает за изменение модуля скорости). Чтобы найти тангенциальное ускорение, рассмотрим небольшое перемещение самолета из точки С в точку С. Тогда vc = vj — — 2а0 (г 4- Ай). Поэтому vc — vc=~ 2a0Ah, где Ай — изменение высоты самолета при перемещении его в точку С. Разделим обе части получен- ного соотношения на тот промежуток времени А/, за который это пере- мещение произошло: (vc - ^с)/А1 - -2aQ&h/&t. Тогда, устремив точку С -> С и Af -► 0, получим 2vca,= -2aovc. Отсюда at = —• а0. Общее ускорение самолета в момент, когда его ско- рость направлена вертикально вверх, получим в виде а = |/ао +(»с/»')2 = «о j/109/З. 1.15. Пусть скорость капель над человеком относительно карусели направлена под углом а к вертикали. Этот угол определяется из тре- угольника скоростей, изображен- ного на рис. 131. Рис. 132 Так как, согласно закону сложения скоростей, с0 = »отн + где ®кар “ скорость карусели в месте нахождения человека, то »отн = г0 - — гкар- Скорость карусели равна Гкар = w. Следовательно, ctg а = = 70
Итак, ось зонта должна быть наклонена под углом а = = arcctg [го/(сог)] к вертикали в направлении движения карусели и пер- пендикулярна радиусу карусели. 1.16*. Пусть в некоторый момент времени доска касается катушки в точке С. Скорость точки С складывается из скорости v0 оси катушки О и равной ей по модулю (проскальзывания нет) скорости точки С (от- носительно точки О), касательной к окружности в точке С. Если угло- вая скорость доски в этот момент времени равна ю, то линейная ско- рость той точки доски, которая касается катушки, будет равна coR tg '1 (а/2) (рис. 132). Поскольку доска все время касается катушки, скорость точки С относительно доски будет направлена вдоль доски, откуда (oR tg ~1 (а/2) = г0 sin а. Из отсутствия проскальзывания катушки по горизонтальной поверхности следует, что р0/К = v/(R + г). Поэтому для угловой скорости (о получим выражение v a 2v sin2 (а/2) со =------sin а • tg — =-------------. R + г z ‘ (R + г) cos (а/2) 1.17. После перемотки толстой ленты она будет занимать часть катушки площадью = л (г2 - г2) = 8яг2. Тогда длина намотанной ленты будет I =₽ SJd = 8л(г\2/с/), где d — толщина толстой ленты. После перемотки тонкой ленты она будет занимать часть катушки площадью = п (г^2 — г2), где г* — конечный радиус намотки во втором случае. Так как длины лент одинаковы, а толщина ленты во второго случае вдвое меньше, чем в первом, можно написать I = 2л - ri)/d, г'к2 - = 4г2. Следовательно, конечный радиус г* намотки во втором случае равен ~ V ‘н- Число оборотов Nj и N2 катушки, сделанных при перемотке в первом и втором случаях, можно записать в виде d ’ 2 d/2 ’ откуда t2 *(|/5- l)rv 1.18. Пусть начальный радиус намотки был 4г, тогда при умень- шении радиуса вдвое (до 2г) площадь намотки уменьшится на величину 5 = п (16г2 - 4г2) = 12лг2, что равно произведению длины смотавшейся пленки на ее толщину d. При прослушивании записи скорость движения ленты г постоянна, поэтому и можно написать соотношение 12яг2 = vtxd. (1) 71
Когда радиус намотки на кассете уменьшится еще раз вдвое (с 2г до г), площадь намотки уменьшится на величину п (4г2 — г2) = Злг2, т. е. Злг2 = vt2d, (2) где t2 — время, за которое уменьшился радиус намотки во втором слу- чае. Поделив уравнения (1) и (2) почленно друг на друга, найдем t2 = МИН. 1.19. Перейдем в систему отсчета, связанную с колечком О'. В этой системе отсчета скорость точки О равна vx/cosa и направлена вверх, так как нить нерастяжима и относительно колечка О' веревка выбирается с постоянной скоростью vx. Поэтому относительно прямой АА', связанной с землей, скорость колечка О будет равна 2sin2(a/2) v2 =-------vr = Vi---------- cos a cos a Рис. 134 1.20. К моменту времени t от начала движения клин пройдет рас- стояние s = at2/2 и приобретет скорость гк = at. За это время грузик переместится вдоль клина на такое же расстояние s, а его скорость от- носительно клина будет равна готн = at и направлена вдоль клина вверх. Скорость ггр грузика относительно земли равна iyp = ®отн + ®к, т. е. (рис. 133) ггр = 2гк sin (a/2) = [2a sin (a/2)] t, а угол, который скорость vrp составляет с горизонтом, равен Р = (л — а)/2 = const. Таким образом, грузик движется вдоль прямой, составляющей с горизонтом угол Р = (л — а)/2; ускорение грузика относительно зе- мли, пока он находится на клине, равно arp = 2a sin (a/2). 1.21. Скорость муравья меняется со временем не по линейному за- кону. Поэтому средняя скорость на разных участках пути различна, 72
и пользоваться для решения известными формулами для средней ско- рости нельзя. Разобьем путь муравья от точки А до точки В на малые участки, проходимые за одинаковые промежутки времени Аг. Тогда Аг = — А//гср (А/), где иср (А/) — средняя скорость на данном отрезке А/. Эта формула подсказывает идею решения задачи: нарисуем зависи- мость величины 1/иср (AZ) от I на пути от точки А до точки В. Этот гра- фик — отрезок прямой (рис. 134); заштрихованная на рисунке площадь S под этим отрезком численно равна искомому времени. Вычислим ее: 5 = l/t?l + l/t>2 2 1 2v rK,j- (так как l/v2 = (1/ih) li/li)- Таким образом, муравей добежит от точки А до точки В за время 4 м2 — 1 м2 t = —--------—;-----= 75 с. 2 • 2 м/с -10 2 • 1 м 1.22. Очевидно, все участки дымного следа в горизонтальном на- правлении движутся со скоростью ветра. Рассмотрим траекторию дви- жения паровоза относительно системы отсчета, движущейся вместе с ветром (рис. 135). Точки дымного следа А' и В' соответствуют дыму, Рис. 138 выпущенному паровозом в точках А и В его круговой траектории от- носительно земли. При этом очевидно, что АА' || ВВ'. Легко понять, что траектория паровоза относительно системы отсчета, движущейся вместе с ветром, будет представлять собой траекторию точки колеса радиуса R, которое вращается со скоростью ип и катится против ветра (влево) со скоростью гв. Как известно, эта траектория называется 73
циклоидой. В зависимости от соотношения скоростей vn и гв траекто- рия существенно различна в своей нижней части. Она либо образует петлю — случай vn > vB (рис. 136), либо имеет место плавный переход, когда vn < гв (рис. 137), либо, наконец, образует острый «клюв», когда гп = vB (рис. 138). Последний случай отвечает нашей задаче. Таким образом, скорость ветра равна v8 = ип = Ю м/с, а чтобы найти ее на- правление, проведем из острия «клюва» касательную СС к траектории движения паровоза (окружности) относительно земли. 1.23* . Пусть карусели повернулись на некоторый угол, ср (рис. 139). Построим точку О (ОА = R) такую, что точки О, С. Л, И будут лежать на одной прямой. Тогда нетрудно видеть, что ОН = R г в любой мо- мент времени. Кроме того, точка О относительно Игоря неподвижна (Слава все время находится напротив Игоря). Поэтому с точки зрения Игоря Никита будет двигаться поступательно по окружности радиуса R 4- г с центром в точке О, которая относительно земли движется по окружности радиуса R с центром в точке А. Слава с точки зрения Ни- киты будет поступательно двигаться по окружности радиуса R + г с центром в точке О, неподвижным относительно Никиты. Однако от- носительно земли точка О движется по окружности радиуса г с цен- тром в точке В. Рис. 139 Рис. 140 1.24. Поскольку обруч с центром в точке Oj покоится, скорость vA верхней точки А «пересечения» обручей в любой момент времени дол- жна быть направлена по касательной к окружности с центром О} (рис. 140). Отрезок АВ в любой момент времени делит расстояние d — — 00J между центрами обручей пополам, поэтому горизонтальная проекция скорости vA все время равна р/2. Следовательно, скорость vA составляет с горизонтом угол ср = л/2 — а и равна va = v/(2 cos ф) = v/ (2 sin a). Поскольку sin a = ]/1 — cos2 a = ]/1 — (J/2R)2, то скорость верхней точки «пересечения» обручей равна v ”д =------------- 2\/l—(d/2R)2 74
1.25. Из условия задачи следует, что во время движения конструк- ции между длинами /2, /3 отрезков А0Аи AGA2, А0А3 сохраняется соотношение G :/2 :13 = 3:5: 6. Поэтому скорости точек Аг, А3 относятся как «л,: »л2: % = 3:5:6 и, следовательно (рис. 141), = »/2. VA1 = 5»/6. Рассмотрим теперь движение среднего звена (Л]В2Л2С2) в тот мо- мент, когда углы конструкции прямые. В системе отсчета, движущейся со скоростью v^t, в этот момент скорость vg2 точки В2 направлена Рис. 141 Рис. 142 вдоль стороны В2А2; скорость точки А2 направлена горизонтально и равна vA1 = vA1 - vAt = v/3. Из условия нерастяжимости стержня В2А2 вытекает, что »в2 = »л2 sin (я/4) = |/2 /6. Скорость точки В2 относительно неподвижной системы отсчета най- дем, воспользовавшись теоремой косинусов: »в2 = + <£ + (.2]/2/2)vAiv^ = (17/36) v2, ₽в2 = (/Й/6)р. 1.26. Если нить тянуть так, как показано на рис. 142, катушка бу- дет катиться вправо, вращаясь по часовой стрелке вокруг своей оси. Для точки В сумма проекций скорости v0 поступательного движе- ния и линейной скорости вращательного движения (с угловой ско- ростью о) на направление нити равна v: v — vQ sin а — (or. 75
Так как по условию задачи катушка движется по горизонтальной по- верхности без проскальзывания, то сумма проекций соответствующих скоростей для точки С равна нулю: vQ — oR = 0. Решая полученные уравнения, находим, что скорость v0 равна v0 = vR/. (R sin a — r). Очевидно, что при R sin a = г (это соответствует случаю, когда точки Л, В и С лежат на одной прямой) выражение для vQ теряет смысл. Заме- тим также, что найденное выражение описывает движение катушки как вправо (когда точка В находится выше прямой АС и Rsina>r), так и влево (когда точка В находится ниже прямой АС и R sin a < г). 1.27. Скорости точек заготовки, лежащих в данный момент на от- резке АВ, равномерно меняются от в точке А до v2 в точке В. Сле- довательно, скорость точки О (рис. 143) в дан- ный момент равна нулю. Точка О — мгновенный центр вращения. (Поскольку заготовка объем- ная, точка О лежит на мгновенной оси враще- ния, которая перпендикулярна плоскости рисун- ка.) Понятно, что скорость Vi в данный момент имеют точки заготовки, лежащие на окружно- сти радиуса О А, а скорость v2 — точки, лежа- щей на окружности радиуса ОВ. (В объемной за- готовке точки с такими скоростями лежат на цилиндрических поверхностях, радиусы которых равны соответственно О А и ОВ.) 1.28. Для описания движения бруска выберем систему отсчета, связанную с лентой транспортера. Тогда в начальный момент времени брусок имел скорость vq = v0 + v, а его движение происходило с по- стоянным ускорением а = — цд. Для момента времени Г, когда ско- рость бруска станет равной нулю, получим уравнение 0 = v0 + v — \igt. Отсюда найдем скорость ленты транспортера: v = \igt — vQ = 3 м/с. 1.29. Запишем уравнение движения тела по наклонной плоскости. Пусть его мгновенная координата равна х (смещение от вершины на- клонной плоскости), тогда та = mg sin a — mg cos a • bx, где m — масса тела, a — его ускорение. Полученное уравнение движения по своему виду напоминает уравнение колебаний тела, подвешенного на пружине жесткости к = mg cos a • b, в поле «силы тяжести» mg sin a. Аналогия с колебательным движением и помогает решить задачу. Найдем положение тела х0, для которого сумма действующих на тело сил равна нулю — это будет «положением равновесия» при коле- бательном движении тела. Очевидно, mgsin a — mgcosa-bx0 = 0; отсю- 76 Рис. 143
да находим х0 = (V^) tg а- В этот момент тело будет иметь скорость г0, которую получим из закона изменения механической энергии тела: mgb cos а 2 - = mg sin а • х0-— = mg sin а • х0--, 2-,- . 2 9 sin2 а Vq = 2дх0 sin а — gbx^ cos а =----. b cos а При последующем движении тело сместится еще на величину х0 — «амплитудное» значение колебаний, что легко получить из «зако- на сохранения механической энергии». Частоту соответствующего ко- лебательного движения найдем из соотношения к/т = gb cos а = Таким образом, тело, пройдя после «положения равновесия» еще расстояние х0 — (l/d)tgoc, остановится. В этот момент возвращающая сила «пропадет», так как она есть не что иное, как сила трения сколь- жения. Как только тело остановится, сила трения скольжения изменит направление и превратится в силу трения покоя, равную mg sin а. Коэф- фициент трения между телом и наклонной плоскостью в месте оста- новки тела равен = b • 2х0 — 2 tg а, т. е. его заведомо хватит, чтобы тело оставалось в покое и дальше. С точки зрения колебательного подхода к описанию данного дви- жения полное время движения тела займет половину «периода колеба- ний». Таким образом, t — Т/2 = 2я/(2соо) = я/]/gb cos а. 1.30. Сила трения саней с грузом Frp(x) прямо пропорциональна длине х въехавшей на песок части саней; запишем уравнение движения саней при их торможении по песку в первом случае: та= — тд(х/1)\к, где т — масса, а — ускорение, I — длина, ц — коэффициент трения саней о песок. Как и в решении задачи 1.29, мы получили «уравнение колеба- ний». Поэтому торможение саней при въезде на песок соответствует движению груза на пружине (жесткости к = (тд/1) ц), которому сообщи- ли в положении равновесия скорость г0. Тогда зависимость от времени въехавшей на песок части саней х (t) и их скорость v (t) получим в виде х (0 = х0 sin v(0 = v0COSG)o^ причем *о = »o/mo> “о = 1А/™ = ]/(з/ОН- Нетрудно понять, что время до полной остановки саней равно четвер- ти «периода колебаний»; таким образом, h = л/(2ш0) = (я/2)0/(^). 77
Во втором случае (после резкого толчка) движение можно рас- сматривать, как если бы сани, въезжая на песок, имели скорость vi>v0 и, пройдя расстояние х0, затормозились до скорости г0 (с этого момен- та и наступает второй случай). При этом интересующее нас движение саней после толчка предстанет в виде части общего колебательного движения по закону x(r) = Xj sinG)or, cos®0t с момента времени r2, когда скорость саней стала равной г0. Как и прежде, хх = vJ(aQ. Кроме того, [п^/(2/)]цхо = m»i/2 - отсюда г, = хо<оо j/2. После толчка сани пройдут путь *1 - *0 = - цо/“о = (1/®о)(»1 - »о) = - !)• Следовательно, отношение путей торможения равно (Xj — х0)/х0 — |/2 — 1. Для нахождения времени движения саней после толчка мы дол- жны определить время движения саней от положения х0 до положения хь используя формулу x(t) = Xi sincOgL Для этого найдем t2 по формуле х0 = Xi sin<oor2. Так как Xj^|/2x0, то ©ог2 —я/4. Следовательно, t2 = я/(4<оо)= t*/2. Поскольку t3 ~ — г2, где t3 — время движения саней после толчка, по- лучим искомое соотношение: ^з/Ч = 1/2- 1.31. Сила тяжести груза тд = 60 Н значительно больше силы, с которой надо тянуть веревку, чтобы удержать груз. Это определя- ется существенными силами трения веревки о бревно. Сначала силы трения препятствуют соскальзыванию груза под действием силы тяжести. Полный расчет распределения сил трения, действующих на веревку, довольно сложен, поскольку сила натяжения веревки в местах ее соприкосновения с бревном меняется от Fj до тд. В свою очередь сила давления веревки на бревно также меняется, бу- дучи пропорциональной в каждой точке соответствующей локальной силе натяжения веревки. Соответственно и силы трения, действующие на веревку, определяются именно указанными силами давления. Одна- ко для решения задачи достаточно заметить, что полная сила трения Ftp (слагающие которой пропорциональны в каждой точке силе реак- ции бревна) будет с соответствующими коэффициентами пропорцио- нальна силам натяжения веревки на концах; в частности, с некоторым коэффициентом к она будет равна большей силе натяжения: FTp = ктд. 78
Это означает, что отношение большей силы натяжения к меньшей есть величина постоянная для данного расположения веревки и бревна: тд/7\ — 1/(1 — к], поскольку 7\ = mg — kmg. Когда мы хотим поднять груз, концы веревки как бы меняются местами. Сила трения теперь направлена против силы Т2 и уже не по- могает, а мешает. Отношение большей силы натяжения, равной теперь Т2, к меньшей — mg будет, очевидно, таким же, как и в первом случае: T2/Wsl/(l-k)sW/r1. Отсюда находим, что Т2 = (^)2/Т1=90 Н. 1.32. Рассмотрим, что происходит, когда водитель поворачивает передние колеса покоящегося автомобиля (речь будем вести про одну шину). В начальный момент колесо не деформировано (в смысле кру- чения), при этом участок шины площади S касается земли. Водитель при помощи рулевого управления деформирует покоящуюся шину до тех пор, пока момент сил приложенных к колесу (и стремящихся его повернуть), не станет больше максимально возможного момента сил трения покоя шины площади S. При этом силы трения перпенди- кулярны плоскости касания колеса. Пусть теперь автомобиль движется. Силы трения покоя прило- жены к тому же участку шины площади S, причем они достигают по- чти максимального значения и лежат в плоскости колеса. Приложения к колесу небольшого момента сил будет достаточно, чтобы повер- нуть колесо, так как противодействовать этому моменту будет теперь суммарный момент «косых» сил трения покоя, который значительно меньше, чем в случае автомобиля, находящегося в покое. По сути де- ла, составляющая силы трения покоя, создающая момент сил, препят- ствующих повороту колеса, при движущемся автомобиле аналогична силе жидкого трения, так как для движущегося колеса пропадает явле- ние застоя при поворотном движении. Таким образом, небольшой мо- мент сил способен легко повернуть движущееся колесо, причем, чем больше скорость движения (тем ближе к предельному значению силы трения покоя), тем легче повернуть колесо. 1.33. Выберем систему координат, как указано на рис. 144. Пусть вектор ОА является вектором начальной скорости v. Тогда вектор АВ есть изменение скорости за время Ат Поскольку сила, действующая на Рис. 144 79
тело, постоянна, то вектор ВС, равный вектору АВ есть изменение скорости за следующий интервал времени Al Поэтому спустя интер- вал времени 3At после начала действия силы направление скорости тела изобразится вектором OD , причем АВ = ВС = CD . Пусть проек- ции вектора АВ на оси х и у равны At\ и Avy, тогда получим два уравнения: (р 4- Avx)2 4- Av2 = v2/4, (v 4- 2Avx)2 4- (2Avy)2 = t?/16. Учитывая, что конечная скорость удовлетворяет соотношению гк — (” + 3At\)2 4- (3Avy)2, получим с использованием предыдущих уравнений, что »К = (1/7/4) v. 1.34. Поскольку движение происходит в горизонтальной плоско- сти, вертикальная составляющая силы, действующей на грузик, равна тд, а горизонтальная F2 — (mg)2 = тд]/а2 — 1, где а = 1,25 (рис. 145). Горизонтальное ускорение груза (и вагона) определяется этой горизон- тальной силой: та = mgij/a2 — 1; следовательно, а = д\/а2 — 1 = = (3/4) 0 = 7,5 м/с2. На первом участке движения вагон ускоряется до скорости v = = ati = (3/4)0^ = 30 м/с, проходя путь sx вперед по прямой: $1 = at2/2 — (3/8) grt2 = 60 м. Далее время, равное г2 = 3 с, он идет с постоянной скоростью v и про- ходит путь s2 = vt2 = 90 м. Таким образом, через 7 секунд после начала движения вагон оказы- вается впереди на расстоянии 4- s2 = 150 м от исходного положения. На третьем участке вагон движется по закруглению дороги впра- во. Так как скорость вагона, движущегося по рельсам, все время на- правлена вдоль вагона, то постоянное (в течение времени г3 = 25,12 с) поперечное ускорение а = (3/4)д является центростремительным, т. е. вагон движется по окружности с постоянной скоростью v: а = v2/R, ра- диус окружности R = v2/a = 120 м. Путь вагона вдоль окружности бу- дет равен s3 = Кф = vf3, откуда угол поворота вагона вокруг центра окружности равен ф = = vts/R = 6,28 = 2л рад, т. е. вагон описывает полную окружность. На последнем участке вагон тормозится и останавливается, по- скольку ускорение вдоль вагона равно первоначальному и действует столько же времени. Поэтому s4 = = 60 м. Вагон остановится на 80
mg Рис. 145 расстоянии s = 2Si + s2 = 210 м впе- реди по отношению к первоначаль- ному положению (рис. 146). 1.35. Пусть в результате приложения силы F шарнир сместится вниз на малую величину Ах, а стержни удлинятся на величину А/ (рис. 147). Тогда жесткость к системы стержней найдем из уравнения /сАх = 2/с0А/ cos а', где 2а' — угол между стержнями после смещения. Так как смещение малое, то а' % а, А/ « Ах cos а; отсюда получаем, что к % 2к0 cos2 а. 1.36. Предположим сначала, что си- ла сопротивления воздуха отсутствует. Тогда шарики встретятся, если верти- кальная составляющая начальной ско- рости 2-го шарика равна вертикальной составляющей скорости 1-го шарика: Vi = v2 sin а; отсюда sin а = Vi/v2 = 10/20 = 1/2, а = 30°. При этом время движения шариков до встречи равно t = s/(v2cosa) ® 0,6 с. Поскольку шарики тяжелые, то легко оценить роль силы сопро- тивления воздуха. Общий характер движения 1-го шарика существенно не изменится, поскольку ускорение, обусловленное силой сопротивле- ния, даже при массе шариков по 10 г и при максимальной скорости 1-го шарика vY = 10 м/с равно amax = 1 м/с2. Это ускорение не более чем на 1% изменяет общее время движения 1-го шарика. Поскольку сила сопротивления воздуха направлена против скорости шарика, то добиться того, чтобы 2-й шарик столкнулся с 1-м, можно, сообщив ему одинаковую с 1-м составляющую скорости по вертикали при усло- вии, что в последующие моменты времени вертикальные проекции ускорений шариков одинаковы в любой момент времени. Для этого угол а, образуемый скоростью 2-го шарика с горизонтом, в момент вылета должен быть равен 30°. 1.37. Запишем уравнение движения шарика в момент, когда пру- жина сжата на величину Ах: та = тд — /сАх. 81
Пока ускорение шарика положительно, его скорость возрастает. В мо- мент обращения ускорения в нуль скорость шарика достигнет макси- мального значения. Пружина при этом сожмется на величину А/, причем mg — fcA/ — 0; отсюда А/ “ тд/к. Таким образом, когда скорость шарика достигнет максимального зна- чения, шарик будет находиться от поверх- ности стола на высоте h — I — тд/к. 1.38* . Легко показать, что шарик достиг- нет положения равновесия при угле откло- нения а нити от вертикали, определяемом уравнением tg а = При колебательном движении шарика на него будет действовать постоянная большая сила F — ]/(тд)2 + (цг)2 и небольшая тормо- зящая сила (рис. 148). Следовательно, движе- ние шарика будет эквивалентно движению слабо затухающего мате- матического маятника с ускорением свободного падения д', равным 9 = д /И?)2 +(М2 = д 1А ! ( ИР? cos а тд у \тд J Период малых (но все-таки затухающих) колебаний шарика найдем из соотношения т= _______________= - ttVHrn2) |/(3//) |/1 +(nv/m^)2 - n7(4n?) 1.39. Запишем условие равновесия небольшого участка нити, ко- торый до подвешивания имел длину Ах и находился от точки подвеса на расстоянии х (рис. 149): т — &хд + Т (х + Ах) = Т (х), где L — длина резиновой нити в нерастянутом состоянии. Таким обра- зом, видно, что после подвешивания натяжение по длине нити будет равномерно падать от значения mg до нуля. В связи с этим относительные удлинения небольших участков ни- ти одинаковой длины в ненапряженном состоянии после подвешива- 82
ния также линейно будут спадать от максимального до нуля. Поэтому полусумма удлинений двух симметрично расположенных относительно середины нити участков будет равна удлинению центрального участка, который испытывает натяжение тд/2. Поэтому общее удлинение нити А/ будет таким, как если бы на нее действовала сила тд/2 в точке подвеса и в нижнем конце, а нить была бы не- ь весома; отсюда А/ «= mg/(2k). h 1.40. Считаем, что выполняется условие тг + т2 > > т3 + т4, иначе равновесие невозможно. Левая пружина — была растянута с силой Ть уравновешивающей силу тяжести груза т2д: 7\ — т2д. Условие равновесия груза т3 состояло в следующем: т3д + Т2 — FH = 0, Рис. 149 где Т2 — сила натяжения правой пружины, FH — сила натяжения верев- ки, переброшенной через блок (см. рис. 14). Эта веревка удерживает грузы массами mr и т2, откуда ^н = (^1 +т2)д. Получим выражение для силы натяжения Т2: Т2 = (Щ + т2-т3)д. После перерезания нижней нити уравнения движения всех грузов находим в виде следующей системы: т2а2 = т2д -7\, т3а3 = Т2 + т3д - Fn, = + - FB, -m4a4 = m^g - T2. Используя найденные ранее выражения для сил 7\, Т2 и FB, получим = а2 - а3 = 0, а4 = (m3 + m4 - - т2)д/т4. 1.41. Сразу после расторможения верхнего блока левый груз имеет скорость г, направленную вверх, а правый покоится. Ускорения грузов будут такими же, как если бы свободный конец веревки не дви- гался с постоянной скоростью, а был закреплен. Найдем их из следую- щих уравнений: тах — 1\— тд, та2 — Т2 — тд, 2Т\ — Т2, = — 2а2. где m — масса каждого груза, 7\, Т2 — силы натяжения веревок, дей- ствующие на левый и правый груз. Решая систему уравнений, получим «! = —(2/5)д, а2 — (1/5)#. Таким образом, ускорение левого груза на- 83
правлено вниз, правого — вверх. Время падения левого груза найдем из уравнения h - vt - 0,4^t2/2 = 0; отсюда t = 2,5 v/g + ]/6,25v2/g2 + 5h/g. Правый груз все это время будет двигаться вверх. Следовательно, первым на полу окажется левый груз. Рис. 150 Рис. 151 модули ускорений при движении вниз, вверх и вдоль горизонтальной направляющей будут соответственно равны (рис. 150): = \л.д cos а — д sjn а, а2 = цд cos а + д sin а, а = цд cos а. Здесь а — угол, образуемый наклонной плоскостью с горизонтом, ц — коэффициент трения. Отсюда получим соотношение а = (а1+ а2)/2. Расстояния, пройденные в указанных условиях задачи бруском при равнопеременном движении с начальной скоростью v до остановки, можно записать в виде G = г2/(2ах), /2 — г2/(2а2), I = v2/(2a). Учитывая связь ускорений а19 а2 и а, найдем расстояние /, пройденное бруском вдоль горизонтальной направляющей планки, / = 2/х/2/(/х+/2). 1.43. Запишем уравнения движения бруска в проекции на ось, про- веденную вдоль наклонной плоскости вниз. При движении бруска вверх, учитывая все дейсгвующие на него силы: силу тяжести тд, силу реакции опоры N и силу трения FTp (рис. 151), уравнение примет вид mg sin а + ртд cos а = таъ и соответственно при движении вниз mg sin а — ц mg cos а = та2. 84
Пусть при подъеме и спуске тело прошло расстояние s. Тогда вре- мя подъема tx и время спуска t2 определим из уравнений s = ajtj/2, s = a2t%/2. По условию задачи 2tx = t2; отсюда 4а2 = аг Следовательно, д sin а +41# cos а = 4 (д sin а — цд cos а), и окончательно ц = 0,6 tg а. 1.44. Если нижний шарик будет очень легким, он начнет подни- маться на подставку. Найдем его предельную массу т2, при которой он еще не начал подниматься, но уже перестал давить на правую на- клонную плоскость. При этом, поскольку подставка невесома, должны быть равны горизонталь- ные составляющие сил давления (равные по модулю силам реакции опоры), действующие на подставку со стороны шариков (рис. 152), иначе «подставка» приобрела бы бесконечно большое ускорение: Ni sin а = N2 sin а, = N2. Кроме того, так как нижний шарик не поднимается, то составляющие ускорений шариков на нормаль к правой наклонной плоскости дол- жны быть равны (в этом направлении у них нет относительного сме- щения). Как видно из рис. 152, угол между направлением силы реакции подставки N2 и правой наклонной плоскостью равен п/2 — 2а, и, следо- вательно, последнее условие можно записать в виде (mucosa — = (mucosa — N 2 cos 2a) /m2; откуда получим, что m2 = cos 2a. Таким образом, нижний шарик бу- дет «забираться» вверх, если выполнено условие т2 < т1 cos 2a. 1.45. До тех пор пока цилиндр не оторвется от опор, ось цилин- дра будет всегда находиться точно посередине между опорами. Следо- вательно, горизонтальная составляющая скорости оси цилиндра равна v/2. Поскольку все точки оси цилиндра движутся по окружности с центром в точке А, полная скорость и каждой точки оси направлена в любой момент времени перпендикулярно радиусу О А = г. Следова- тельно, все точки оси движутся с центростремительным ускорением Оц = U2/Г. 85
Запишем уравнение движения точки О в проекции на «центростре- мительную» ось: mg cos а — N — тац — mu2/r, (1) где N — сила реакции со стороны неподвижной опоры. Из условия, что расстояние между точками опор равно г j/2, следует отсутствие вклада силы реакции со стороны подвижной подставки на «центростремитель- ную» ось. По третьему закону Ньютона с такой же по модулю силой цилиндр давит на неподвижную опору. Из уравнения (1) находим N — mg cos а — ти2/г. В тот момент, когда расстояние между точками опор А и В (рис. 18) равно АВ = г|/2, cosa = rj/2/(2r) = l/j/2. Горизонтальная составляющая скорости точки О равна и cos a — г/2, откуда и — v |/2. Таким образом, при АВ = г ]/2 сила давления цилиндра равна N = т#/]/2 — mv2/(2г). Для того чтобы цилиндр не оторвался от опор до того, как АВ станет равным г]/2, должно выполняться условие g/^2>v2/(2r), т.е. v<]/gr]/2. 1.46. На цилиндр действуют: сила тяжести тхд, сила нормальной реакции N t со стороны левой наклонной плоскости и сила нормальной реакции N3 со стороны клина (сила N3 направлена горизонтально). За- пишем уравнение движения ци- линдра в проекции на ось хь на- правленную вдоль левой наклон- ной плоскости (рис. 153): sin a — N3 cos a, (1) где «1 — проекция ускорения ци- линдра на ось хг На клин действуют: сила тя- жести т2д, сила нормальной реакции N2 со стороны правой наклонной плоскости и сила нормальной реак- ции со стороны цилиндра, которая, согласно третьему закону Ньюто- на, равна — N3. Запишем уравнение движения клина в проекции на ось х2, направленную вдоль правой наклонной плоскости: т2а2 — — т2д sin a + N3 cos a. (2) В процессе движения клин соприкасается с цилиндром; поэтому, если перемещение клина вдоль оси х2 равно Ах, то центр цилиндра (вместе с вертикальной гранью клина) сместится по горизонтали на 86
Ax cos а. При этом вдоль левой наклонной плоскости (вдоль оси xj центр цилиндра сместится на Ах. Это означает, что в процессе движе- ния клина и цилиндра выполняется соотношение скорости груза, а ускорение тела Рис. 154 at = а2. (3) Решая совместно уравнения (1)—(3), найдем силу N = N3, с кото- рой клин давит на цилиндр: N3 = [2m1m2/(m1 4- m2)] tga. 1.47. До тех пор пока груз касается тела, скорость последнего рав- на горизонтальной составляющей равно горизонтальной состав- ляющей ускорения груза. Пусть а — полное ускорение груза, тогда можно написать: а = Of 4-Оц, где ац — центростре- мительное ускорение груза при его движении по окружности радиуса Z, т. е. ац = v2/l, где v — скорость груза (рис. 154). Горизонтальная составляющая ускорения груза равна аг — at sin a — (v2/Z)cos a. С таким ускорением движется и тело. Напишем уравнение движения тела: /V = Мог — Mat sin a - М (v2/l) cos a. где N — сила нормального давления на тело со стороны груза. В момент отрыва груза N — 0 и at sin a = (г2//) cos a. Ускорение at в момент отрыва сообщается грузу только силой тяжести: at = д cos a. Таким образом, скорость груза в момент отрыва равна v — ydl sin a> а скорость тела в тот же момент и = v sin a = sin a \/gl sin a. Согласно закону сохранения энергии, mgl — mgl sin a 4- mv2/2 4- Mv2 sin2 a/2. Подставив в это равенство найденное выражение для v в момент 87
отрыва и значение sin а = sin л/6 = 1/2, найдем отношение М/т\ М/т = (2 — 3 sin a) /sin3 а = 4. Скорость тела в момент отрыва груза равна и = v sin а = (1/2) Уgl/2. 1.48. На стержень действуют три силы: сила Т натяжения нити, сила тяжести тд и сила реакции со стороны стенки R = N 4- F^ (N — сила нормальной реакции стенки, — сила трения, |1N). Рис. 155 В состоянии равновесия стержня сумма мо- ментов этих сил относительно любой точки равна нулю. Чтобы это условие выполня- лось, линия действия силы R должна прохо- дить через точку пересечения линий действия сил Ти тд (моменты сил Тн тд относительно этой точки равны нулю). В зависимости от соотношения между углами аир точка пересечения линий дей- ствия сил Т и тд может лежать: 1) выше перпендикуляра АМ0 к стене (точка Mt на рис. 155); 2) ниже этого перпендикуляра (точ- ка М2); 3) на нем (точка Мо). Соответственно сила трения либо направлена вдоль АС вверх (FTpi), либо — вдоль АС вниз (FTp2), либо равна нулю. Рассмотрим каждый из этих случаев отдельно. 1) Условие равновесия стержня: Тcos а 4- FTpi — тд = О, N — Тsin а = 0 (1) — равенство нулю суммы проекций всех сил на оси у и х соответственно; а также равенство моментов сил отно- сительно точки А: mgdx = Td2, или (mgl/2) sin р = (77/3) sin (а 4- р), (2) где dx и d2 — плечи сил тд и Т соответственно. Из (1) и (2) имеем F^! 2 sin(a4-P) 1 1/2 1 \ и, > —— =-----------------------= — I------------); N 3 sin a sin р tga 3 у tg Р tga/ этот случай осуществляется при условии 2 tg а > tg р. 2) Написав условия равновесия, найдем 1 / 1 2 \ 3 \tgtx tg Р / Этому случаю соответствует условие 2 tg a < tg р. 88
3) В этом случае стержень находится в равновесии при любом значении ц3: 2 tg а = tg р. Таким образом, при произвольном соотношении между углами аир стержень находится в равновесии, если 1 2 tga tgp 1.49*. Рассмотрим, как будет двигаться малый диск сразу после соприкосновения с большим. Выберем два одинаковых небольших участка малого диска, распо- ложенных на одном диаметре симметрично относительно центра этого диска О'. На рис. 156 точки и А2 — центры масс таких участков. В момент касания дисков (когда малый диск еще покоится) скорости vr и v2 тех точек большого диска, которые соприкасаются с точками Л1 и Л2 малого диска, направлены так, как показано на рис. 156 (vj = = Q- ОАЪ v2 = Q- ОА2). Понятно, что вдоль скоростей vr и v2 будут на- правлены в момент касания силы трения FTpl и FTp2, действующие со стороны большого диска на центры масс выбранных участков Аг и А2 малого диска (FTpi = FTp2). Поскольку плечо Ц силы FTpi относительно оси малого диска меньше плеча /2 силы FTp2 (рис. 156), суммарный мо- мент пары сил FTpl и FTp2 будет закручивать малый диск в направле- нии вращения большого диска. Рассмотрев аналогичные пары участков малого диска, приходим к выводу, что сразу после касания малый диск начнет закручиваться в направлении вращения большого диска. Пусть в некоторый момент времени угловая скорость малого ди- ска стала равной со. Скорости участков с центрами масс в точках А{ и А2 будут равны v/ = v2 = cor, где г = О'Аг = О'А2 (рис. 157). Силы тре- ния F^pl и F^2, действующие на эти участки, будут направлены вдоль векторов — г/ (относительная скорость точки большого диска, ка- 89
сающейся точки и v2 — v2 (относительная скорость точки боль- шого диска, касающейся точки А2). Понятно, что момент пары сил F^pi и Ftp 2 будет раскручивать малый диск (т. е. угловая скорость диска бу- дет меняться), если v/= v2 < В1В2/2 = Qr (см. рис. 157; для удобства сравнения векторы, «относящиеся» к точке Л2, перенесены в точку ЛД Таким образом, пока со < О, имеется отличный от нуля момент сил трения, раскручивающих малый диск. При со = Q относительные скорости участков с центрами масс в точках At и А2 направлены пер- пендикулярно отрезку 00' (вдоль отрезка А^С на рис. 157) и момент сил трения относительно оси малого диска равен нулю. Следователь- но, и в дальнейшем малый диск будет вращаться с установившейся угловой скоростью (1 При со = Q все силы трения, действующие на аналогичные пары участков малого диска, будут равны по модулю и направлены одина- ково - перпендикулярно отрезку 00'. Согласно третьему закону Нью- тона, результирующая всех сил трения, действующих на большой диск, будет приложена к точке большого диска, касающейся центра О' мало- го диска, и равна цтд. Чтобы компенсировать тормозящий момент этой силы, к оси большого диска надо приложить момент сил, равный Л = pmgd. L50. После установления поступательного движения системы от- ношение сил трения FTpi и FTp2, действующих на первую и вторую па- лочки, будет равно отношению сил давления соответствующих участ- ков: FTpi/FTp2 = Ni/N2. Поскольку каждая из сил давления пропорцио- нальна массе: Nt = тхд, N2 = m2gy то отношение сил трения получим в виде ^тр1/^'тр2 = "’1/т2. (1) 90
С другой стороны, из равенства моментов этих сил относительно вер- шины прямого угла (рис. 158 — вид сверху) получим / FTP1 cos ф = I Ft₽2 sin ф, (2) где / — расстояние от вершины до центров масс палочек. Из уравнений (1) и (2) находим, что tg ср = где ф = «=₽ а — я/2. Следовательно, угол а « я/2 4- \ + arctg(m!/m2). а 1.51. Если нога футболиста при ударе движется со скоростью и, то в системе отсчета, связанной с ногой футболиста, скорость мяча равна v 4- и (ось движения расположена ' по направлению движения мяча). После аб- солютно упругого удара скорость мяча в ^тр2г этой же системе будет — (к 4- и), а его ско- рис рость относительно земли — (v 4- и) — и. Если после удара мяч остановился, то v 4- 2и ** 0; отсюда и == — г/2 — —5 м/с. Знак минус указывает, что нога футболиста должна двигаться в ту же сторону, что и мяч до удара, 1.52. Поскольку после попадания пули в тело уменьшается верти- кальная составляющая скорости системы тело - пуля (закон сохране- ния импульса), то время падения тела на землю увеличится. Чтобы определить время падения тела на землю, найдем время падения тела до попадания в него пули и время t2 движения тела вместе с пулей. Итак, пусть t0 - время свободного падения тела с вы- соты h, Тогда первую половину пути тело пройдет за время h « = В момент попадания в тело массой т пули массой М его импульс был направлен вертикально вниз и равен mv ~ Попавшая в тело горизонтально летящая пуля не изменит верти- кальной составляющей импульса образовавшейся системы, поэтому скорость системы тело — пуля в вертикальном направлении будет равна т т t0 U =-------р — -------д--— т + М т + М 1/2 Оставшуюся половину пути система тело — пуля пройдет за время i2, которое можно найти из уравнения Л/2 = ut2 + gtj/2. Откуда находим время t2: t0 j/m2 4* (m 4- M)2 — M *2 j/2 m 4- Af 91
Таким образом, общее время падения тела на землю будет равно (М » т) t0 |Ли2 + (т + М)2 + т 1/2 т + М 1.53. Для решения задачи воспользуемся законом сохранения им- пульса системы. Выберем УА систему координат так, как указано на рис. 159: ось х совпадает с направ- лением скорости Vi тела массой ть ось у направлена вдоль скорости v2 тела массой т2. После слипания Рис. 159 Рис. 160 тела полетят со скоростью и, причем = (Ш1 + Ux, m2V2 = (Ш1 + Wz) иу До соударения кинетическая энергия системы была равна РТк = mrv2/2 + m2v2/2. Кинетическая энергия системы после соударения (слипания) тел станет равной W + 2)_ 2 7 2^ +т2) Таким образом, в результате соударения выделится количество те- плоты, равное т.т7 , 7 Q = - рг; = 12 (v? + v22) * 4,3 Дж. 2(т1 + т2) 1.54. Так как трение отсутствует, то в горизонтальном направле- нии на нашу систему не действуют внешние силы (рис. 160). Для опре- деления скорости v левого клина и скорости и шайбы сразу после спу- ска можно воспользоваться законом сохранения энергии и законом сохранения импульса: Mv2/2 + ти2/2 = mgh, Mv = mu. 92
Поскольку в момент наивысшего подъема шайбы на правый клин на высоту hmax их скорости будут одинаковы, закон сохранения им- пульса запишем в виде mu = (М + т) V, где V — общая скорость шайбы и правого клина. Кроме того, восполь- зуемся законом сохранения энергии: mu2 М + т э ---V + титлах • Совместное решение вышенаписанных уравнений дает выражение для максимальной высоты подъема шайбы на правый клин: М2 ^max — z, , , Ч2 ’ (М + ту закона сохранения энергии: 1.55. До тех пор пока шайба не окажется в наинизшем положении, брусок будет касаться стены. К этому моменту времени шайба при- обретет скорость г, которую найдем из v2 = 2дг. При последующем движении системы правую ускоряя наконец сравняются, те ль но бруска начнет соскальзывать вниз, при этом, пока она опять не пройдет низшее положение, брусок все еще будет ускоряться. Таким образом, максимальная скорость бруска будет шайбой низшего положения при ее движении назад относительно бруска. Чтобы найти максимальную скорость бруска, запишем закон со- хранения импульса шайба будет «забираться» на половину бруска, все время его вправо (рис. 161), пока скорости шайбы и бруска не Далее шайба относи- v=v2gr Рис. 161 в моменты прохождения после того, как брусок «оторвется» от стены: и закон сохранения шего положения: т2 У^9Г — mlVl + m2V2’ энергии для моментов прохождения шайбой низ- т<у2 m2v2 m2gr = 2 2 Написанная система уравнений 1. =0, v2 = у2дг, 2т2 /— 2 2. v + т2 имеет два решения: — т. /— т1+т2 г2 = 93
Решение 1 отвечает моментам времени, когда шайба движется, а бру- сок находится в покое. Нас интересует решение 2, отвечающее тем мо- ментам времени, когда брусок имеет максимальную скорость: l-’lmax = 2m2 1.56. Перейдем в систему отсчета, связанную с коробкой. По- скольку удары шайбы о коробку абсолютно упругие, то скорость шайбы относительно коробки будет периодически менять свое напра- вление, оставаясь равной по модулю v. Нетрудно понять, что характер движения шайбы будет повторяться через время 2Дг, где Д/ = = (£) — 2r) /v — время пролета шайбы от одного соударения с коробкой до следующего (центр шайбы всякий раз проходит путь D — 2г со ско- ростью v). Возвращаясь в систему отсчета, связанную с землей, можно по- строить график зависимости скорости центра шайбы от времени vm(t). Располагая зависимостью иш(г), легко построить график зависимости перемещения центра шайбы от времени хщ(г) (рис. 162). t0 Ь 2tf t Рис. 162 1.57. Силы, действующие на систему обруч — шайба, — это сила тяжести и сила нормальной реакции со стороны плоскости. Обе эти силы направлены вдоль вертикали. Следовательно, центр масс си- стемы в горизонтальном направлении не перемещается. Поскольку трение между обручем и плоскостью отсутствует, обруч движется поступательно. Согласно закону сохранения импульса, в любой момент времени Ми + mvx = 0, (1) где и и vx — горизонтальные проекции скоростей центра обруча и шайбы. Так как vx периодически меняет знак, то и и «синхронно» ме- няет знак. Общий характер движения обруча таков: шайба на участках ВС и BE — центр обруча движется вправо; шайба на участках CD и DE — центр обруча движется влево (рис. 163). 94
Скорости шайбы v и обруча и связаны законом сохранения энергии: mgr (1 + cos q>) = nw2/2 + Ми2 /2. (2) Движение шайбы относительно неподвижного наблюдателя мож- но представить в любой момент времени как суперпозицию двух дви- жений: движения относительно центра обруча со скоростью rt, напра- вленной по касательной к обручу, и движения вместе с обручем со скоростью и, направленной горизонтально (рис, 164). К-ак видно из рисунка, vy/(Vx + i\) = tgq>. (3) Решая совместна уравнения Ц)-(З), найдем скорость центра обру- ча в тот момент, когда радиус-вектор точки, в которой находится шай- ба, составляет угол ф с вертикалью: и = т cos ф ]/2дг (1 -Ь cos ф)/ [(М + ш)(М + т sin ф)], 1.58. В момент обрыва правой нити на стержень действуют: сила натяжения Т левой нити и силы и N2 нормального давления грузов массами и т2 (рис. 165). Поскольку стержень невесом (его масса равна нулю), уравнения его поступательного и вращательного движе- ний примут вид -T + Ni-N2 = 0, NJ = 2N2l. Из второго уравнения (условие равенства нулю суммы моментов всех сил относительно точки О) следует Ni = 2N2. (1) С учетом этих соотношений получаем (см. рис. 165) T = N1-N2 = N2. (2) Ускорения грузов массами и т2 в первый момент после обры- ва правой нити будут направлены вертикально (точка О неподвижна, 95
стержень нерастяжим) и связаны соотношением = 2аР (3) Запишем уравнения движения грузов в этот момент: mrg — N/ = mvav, m2g + N2=m2a2, где N[ и N2 — силы реакции стержня, действующие на грузы массами тх и т2. Поскольку N[ = и N2 = N2, то тхд — 2N2 = т2д + N2 = 2m2av Отсюда находим N2i а следовательно (см. (2)), и силу натяжения нити Т: T=N2 = -----7~Л--- mr + 4m 2 1.59. Пусть за малый промежуток времени Аг после начала движе- ния системы колечко опустилось на расстояние Ах от точки А и при- обрело скорость v (рис. 166). Ско- рость поступательного движения обручей в этот момент должна быть равна и = v tg a (At настолько ма- ло, что угол а практически не изменился). Следовательно, такова же линейная скорость всех точек обручей. Согласно закону сохранения энергии, mg Ах = 2 Ми2 + mv2/2 = 2Mv2 tg2 а + т и2/2, где Ми2 — кинетическая энергия каждого обруча в данный момент. Из этого равенства находим v2 т 1 2Ах 4М tg2 а 4- т 1+4 (М/т) tg2 а При Ах -> 0 можно считать, что v2 = 2а Ах, где а — ускорение колечка в начальный момент времени. Следовательно, 1 1+4 (М/т) tg2 а 96
1.60. Пусть за малый промежуток времени Аг после начала движе- ния веревка переместилась на расстояние А/ и приобрела скорость v. Так как Аг мало, можно считать, что v2 = 2abl, (1) где а — ускорение всех точек веревки в начальный момент времени. Из закона сохранения энергии (трение отсутствует) следует, что Mv2/2 = АЙГП, (2) где М — масса всей веревки, AJVn — изменение потенциальной энергии веревки за время Аг. Очевидно, что АРГП соответствует перераспределе- нию массы веревки, в результате которого кусок веревки длиной А/ «переходит» из точки А в точку В (см. рис. 31). Таким образом,, &Wn=(M/l)ghti. (3) Из (1) —(3) находим условие движения веревки в начальный момент времени: а = gh/l. 1.61. Очевидно, в момент удара только крайние кубики соприка- саются с шайбой. Сила, действующая на каждый из этих кубиков, на- правлена перпендикулярно грани касания шайбы с соответствующим кубиком и проходит через его центр (диаметр шайбы равен ребру ку- бика!). Поэтому средний кубик в результате этого удара останется не- подвижным. Для крайних кубиков и шайбы можно написать закон со- хранения импульса по направлению скорости шайбы v: mv = 2ти • |/2/2 + mv', здесь т — масса каждого кубика и шайбы, v' — скорость шайбы после соударения, и — скорость каждого из крайних кубиков. Закон сохране- ния энергии дает уравнение v2 = 2и2 4- v'2. В результате получаем, что и — vj/2, v' = 0. Следовательно, после удара скорости крайних кубиков составляют углы 45° со скоростью v, шайба останавливается, средний кубик так и останется неподвижным. 1.62. В данном случае закон сохранения импульса удается приме- нить в несколько неожиданном виде. При взрыве шарика составляю- щая импульса шарика вдоль трубы остается равной нулю, так как тре- ние отсутствует и силы реакции направлены перпендикулярно скоро- стям частей. Неупругие соударения также не изменяют продольной составляющей импульса системы. Следовательно, окончательная ско- рость образовавшегося после всех соударений тела равна нулю. 1.63. Для того чтобы выделившееся количество теплоты было максимальным, необходимо выполнение следующих условий: 4 А. И. Буздин и др. 97
1) потенциальная энергия тел в начальный момент должна быть максимальной; 2) тела должны столкнуться одновременно в нижней точке чаши; 3) скорость тел сразу после столкновения должна быть равна нулю. При выполнении этих условий весь запас начальной потенциаль- ной энергии тел перейдет в теплоту. Следовательно, тела в начальный момент должны быть расположены на кромке чаши на высоте г -над ее нижней точкой. Расположение тел должно быть таким, чтобы их сум- марный импульс перед соударением был равен нулю (тогда после со- ударения слипшееся тело останется неподвижным в нижней точке ча- ши). Поскольку значения импульсов тел в любой момент времени относятся, как 3:4:5, расположение тел в начальный момент должно быть таким, как на рис. 167 (вид сверху). После предоставления тел са- мим себе количество теплоты, которое выделится в системе, макси- мально и равно Q — 4тдг. 1.64. Пусть вначале относительно неподвижной системы отсчета протон покоится, а а-частица имеет скорость v0. Процесс их упругого соударения описывается законом сохранения импульса: 4m©0 = mv1 + 4m©2, и законом сохранения энергии: 4mv§/2 = mvl/2 + 4mv2/2, где и v2 — скорости протона и а-частицы в неподвижной системе от- счета после соударения, т и 4т — массы протона и а-частицы. Рассмотрим процесс соударения этих частиц в системе центра масс, т. е. в инерциальной системе отсчета, движущейся относительно неподвижной системы отсчета со скоростью v' = 4mv0/(m 4- 4т) — (4/5) v0 (в числителе первой дроби стоит полный импульс системы, в знамена- 98
теле — ее полная масса). На рис. 168 изображены скорость v0 и скоро- сти а-частицы (вектор ОВ) и протона (вектор ОА ) в системе центра масс до соударения: ОВ = (1/5) г0, О А = (4/5) v0. Согласно закону сохра- —► —► нения импульса, после соударения векторы ОВ и О А скоростей а-ча- стицы и протона должны лежать на одной прямой и должно выпол- няться соотношение О В': О А' =1:4 (см. рис. 168); согласно закону со- хранения энергии, ОВ' = О В и О А' = О А (покажите!). Скорости а-частицы и протона в неподвижной системе отсчета —> —> —> —► изображены на рисунке векторами ОС2 = О В' + v' и OCi = О А' + v'. Чтобы ответить на вопрос задачи, нужно определить максимально возможную длину вектора ОСЪ т. е. в равнобедренном треугольнике ОА'Сг необходимо определить максимально возможную длину осно- вания при неизменных длинах боковых сторон. Очевидно, что макси- мум OCi равен 2 • (4/5) v0 — l,6v0. Эта ситуация отвечает центральному удару. 1.65. Шины автомобиля в песке проминают след. Чем больше да- вление на песок, тем больше глубина следа, тем больше вероятность машине застрять. Если шины значительно спущены, то площадь со- прикосновения шин с песком увеличивается. Давление на песок при этом уменьшается, следовательно, уменьшается и глубина следа. 1.66. Сложное движение тела внутри трубы в любой момент времени можно представить как суперпозицию двух независимых дви- жений — движения вдоль оси трубы и движения по окружности в пло- скости, перпендикулярной оси трубы (рис. 169). Отрыв тела от поверх- ности трубы скажется только на втором движении — тело не будет двигаться по окружности. Поэтому мы рассмотрим только это движение. Рис. 169 На тело, движущееся по окружности, действует сила N нормаль- ной реакции со стороны стенок трубы (вектор N лежит в плоскости, перпендикулярной оси трубы) и «сила тяжести» mg' = mg cos а. Напи- шем условие движения тела по окружности: mg' cos р + N = mv2/r, (1) где р — угол, который составляет радиус-вектор той точки окружности, 4* 99
в которой в данный момент находится тело, с «вертикалью» у' (рис. 170). Чтобы тело не отрывалось от поверхности трубы, должно выполняться условие N = mv2/r — mgr cos 0 > 0, откуда следует v2 д'г cos 0. (2) Связь скорости v, с которой в данный момент движется тело по окружности, с начальной скоростью v0 найдем, воспользовавшись за- коном сохранения энергии: при любом значении угла 0 должно выпол- няться соотношение nw2/2 4- mg'r cos 0 = т (v0 sin <p)2/2 4- mg'r; отсюда v2 = Vq sin2 <p 4- 2g'r — 2g'r cos 0. (3) Подставив (3) в (2), найдем, при каких значениях v0 тело не оторвется от трубы: Vq 3 д'г cos 0/sin2 ф — 2 g'r/sin2 ф. Поскольку это условие должно выполняться при любом значении том. Эта скорость находится 0е[О, 2я], окончательно имеем Vq g'r/sin2 ф = gr cos a/sin2 ф. 1.67* . Пусть в некоторый мо- мент времени колесо находится в одном из положений, когда его центр масс расположен над прутом и его скорость равна v. В момент уда- ра о следующий прут (рис. 171) центр масс колеса имеет некоторую скорость v', направленную перпендикулярно ли- нии, соединяющей его с первым пру- из закона сохранения энергии: mgh 4- mv2/2 = mv'2/2. Здесь h = г — ]/r2 — Z2/4 «l2/($r). Получаем v' = v ]/l 4- gl2/(4rv2). По условию задачи (движение без подскоков) удар колеса о прут абсо- лютно неупругий. Это означает, что при ударе гасится проекция им- пульса колеса на прямую, соединяющую центр колеса с прутом. Тем самым при каждом ударе происходит потеря (переход в теплоту) энер- гии, равной ДРТ = m(v' sin a)2/2, 100
где sin а »l/r. Чтобы скорость v оставалась постоянной, нужно, чтобы работа силы натяжения Т веревки на пути I компенсировала эту поте- рю энергии. Таким образом, mv2 ( gl2 \ I2 г/==-Н1+-^Ч- — 2 \ 4rv2 / г2 Отсюда mv2l ( ql2 \ mv2l Т =--------1 1 4- —— «---------- 2r2 Г W / 2г2 ’ 1.68. Поскольку проскальзывания нет, то при перекатывании через границу раздела плоскостей ось колесной пары вращается вокруг точ- ки О (рис. 172). В момент отрыва сила давления колесной пары на плоскость и I сила трения равны нулю, поэтому угол 0, /* при котором происходит отрыв, нахо- ( 1 ) дится из условия mg cos 0 — mu2/г. Из закона сохранения энергии получаем nw2/2 = mv2/2 — mgr (1 — cos 0). Рис. 172 Отрыва не происходит, если опреде- ляемый из этих уравнений угол 0 не меньше а и, следовательно, cos 0 < cos а. Отсюда вытекает, что условие v < ]/gr(3 cos а — 2) есть условие перекатывания колесной пары через границу раздела без отрыва. Если 3 cos а — 2 < 0, т. е. а > arccos (2/3), то отрыв произойдет при любой скорости V. 1.69. Потенциальная энергия системы в начальный момент складывается из потенциальной энергии обода, равной mg (г + Л), и по- тенциальной энергии части ленты, лежащей на наклонной плоскости, pgrZi2/(2 sin ос). Полная конечная энергия системы также будет чисто по- тенциальной и равной ввиду отсутствия трения начальной энергии. Ко- нечная энергия складывается из энергии обода mgr и энергии намотавшейся на него ленты. Центр масс последней будем считать совпадающим с центром масс обода. Это предположение верно, если длина намотавшейся ленты много больше длины окружности обода. Тогда потенциальная энергия намотавшейся ленты есть p(/i/sinoc + s)gr, причем длина ленты равна h/ sin а + s, где s — искомое расстояние, 101
пройденное ободом от основания наклонной плоскости до точки остановки. Из закона сохранения энергии получаем Л2 ( h \ m#(r + h) + рд —---= mgr + pgrI —---+ $ ; 2 sin а \ sin а / отсюда mgi + р(Л/ sin а) (г - h/2) s ---------------------. рг 1.70. Установившимся движением системы в воздухе будет верти- кальное падение шариков с постоянной скоростью. Силы сопротивле- ния воздуха. F, действующие на нижний (более тяжелый) и верхний шарики, одинаковы, поскольку одинаковы скорости и размеры шари- ков. Поэтому уравнения движения шариков запишутся в виде / mig ~ Т~~ F = 0, тг9 + Т— F = 0. -J Решая эту систему, получим силу натяжения нити 'д? T=(mi-m2)g/2. у 1.71*. В каждый момент времени через точку касания нити с цилиндром проходит мгновенная ось вращения шарика. Это озна- чает, что сила натяжения нити перпенди- кулярна, скорости шарика, откуда следует, что эта сила не совершает работы. Тем самым кинетическая энергия шарика не меняется, и его скорость остается по модулю равной v. Чтобы найти зависимость / от г, мысленно разобьем размотав- шуюся к моменту t часть нити на очень большое число N малых равных кусочков длины AZ = 1/N каждый. Пусть разматывание n-го ку- сочка продолжалось время Аг„. За это время конец нити переместился на расстояние vAtn, а сама нить повернулась на угол Афп = v&tn/(n AZ) (рис. 173). На этот же угол повернулся и радиус, проведенный в точку касания нити с цилиндром, т. е. Афи = Аф = AZ/r; отсюда ^t„=n(M)2l(vr). Тогда t = Aft + At2 4- ... 4- AtN = _ 1(AZ)2 2 (AZ)2 N(AZ)2 _ (AZ)2 N(N + 1) vr vr vr vr 2 102
Поскольку N велико, t = (M)2N2/(2vr) = l2/(2vr), I = ]/2wt 1.72. За время T голубой шарик пройдет расстояние ®(//]/3)Т = = 2л///3 (рис. 174; со = 2л/Т — частота вращения). За это же время центр масс зеленого и белого шариков переместится на расстоя- ние со(//2 j/З) Т = л//]/з. Одновременно стержень, соединяющий зе- леный и белый шарики, повернете» на угол 2л, так как период обраще- ния этих шариков вокруг их центра масс совпадает с периодом Т. Поэтому искомое расстояние равно Ь=/|/з/4 + (л|/з + 1/2)2 или при другом расположении шариков (белый и зеленый на рисунке нужно поменять местами) L = / |/з/4 + (л ]/з — 1/2)2. 1.73. На центр масс системы, состоящей из брусков и нити, дей- ствует в горизонтальном направлении только сила со стороны блока. Ясно, что горизонтальная составляющая этой силы, равная Т(1 — coscp), где Г—сила натяже- ния нити, всегда направлена вправо (рис. 175). Поскольку в начальный момент центр масс нахо- дится над блоком и покоится, в процессе движе- ния он будет смещаться по горизонтали вправо. Отсюда следует, что левый брусок достигнет бло- ка раньше, чем правый ударится о стол, поскольку в противном случае центр масс оказался бы в момент удара слева от блока. 1.74. Благодаря начальным условиям (левый груз неподвижен, правому сообщена скорость г) левый груз будет двигаться по прямой, правый помимо движения по прямой будет совершать колебания. В некоторый момент времени на левый груз по вертикали действует сила тд — Т9 на правый mg — Тcos ф (рис. 176; вертикальная ось направлена вниз). Здесь Т — сила натяжения нити. Отсюда следует, 103
что разность вертикальных составляющих ускорений правого (at) и левого (а2) грузов, равная / т \ / т \ т а, — а2 = [д--cos ср I — \д-I = —(1— cos ср), \ m / \ m / m всегда неотрицательна. Поскольку относительное расстояние и относи- Рис. 177 тельная скорость грузов по верти- кали в начальный момент равны нулю, то с течением времени раз- ность ординат правого и левого грузов будет увеличиваться, т. е. в любой момент времени правый груз находится ниже левого. 1.75. Пусть правая нить откло- нилась на угол 0 от вертикали,, а левая — на угол а (рис. 177). Чтобы стержень не отклонялся от верти- кального положения, необходимо выполнение равенства Ti sin a = Т2 sin 0, (1) где 7\, Т2 — силы натяжения соответствующих нитей. Запишем уравнения движения обоих тел в вертикальном и гори- зонтальном направлениях: 7\ sin a = sin a, 7\ cos a = T2 sin 0 = m2G)1 2l2 sin 0, T2 cos 0 = m2g. Решив полученную систему уравнений с учетом (1), найдем (0 = д1/2 «14 рад/с. 1.76. Обозначим через и 12 длины пружин, соединяющих ось вращения с первым шариком и шарики между собой. Так как шарики движутся по окружности, то уравнения их движения примут вид mw2/, = k(lt - /0) - к(12 - 10), т<о2(/1 +/2) = к(/2 —/0); отсюда 1 — Зта2/к + (та2/к)2 ’ (1 — та2/к) 10 1 - Зта2/к + (та2/к)2 104
Решение имеет физический смысл, если выполняются неравенства 1 — Зто2/к 4- (то2/к) >0, 1 — то2/к 0. Пусть то2/к = х. Так как то2/к > 0, из второго условия следует 0 < х < 1. Первое условие дает х2 - Зх 4-1 > 0, откуда либо х > (3 4- j/5) /2 » 2,6, либо х < (3 —1/5) /2 » 0,4. Следова- тельно, область допустимых значений х есть 0 < х < (3 — j/5)/2, откуда /з -1/5 к /—г—• f 2 т 1.77. Изменение кинетической энергии тела WK при небольшом перемещении As записывается в виде AWK = FAs, где F — сила, действующая на тело. Поэтому на графике зависимости кинетической энергии от перемещения при прямолинейном движении сила в некоторой точке траектории движения определяется как тангенс Рис. 178 угла наклона касательной в соответствующей точке графика. Исполь- зуя график, приведенный в условии задачи, построением (рис. 178) на- ходим Fc « — 1 Н, FB « — 3 Н. 1.78. Количество выделившейся теплоты сбудет максимальным при условии, что брусок пройдет максимальное расстояние относи- тельно транспортера. Для этого необходимо, чтобы около ролика А скорость бруска относительно земли стала равной нулю (см. рис. 41). Начальная скорость относительно земли, которую при этом 105
должен иметь брусок, определяется условиями — vQ + at = О, I = vQt — at2 /2, где а = цд — ускорение, сообщаемое бруску силой трения. Отсюда p0 = j/W- Время движения бруска по ленте транспортера до ролика А равно t = j/2Z/(W). До остановки брусок пройдет по ленте путь, равный St = I + vt = I + v |/2//(ц^). Далее брусок начнет двигаться равноускоренно вправо. Интервал времени, спустя который прекратится проскальзывание, равен т = = v/a = v/pg. За это время относительно земли брусок переместится на расстояние 5 = т2/2 = Р2/(2рв)- Так как по условию задачи v < |/2цд/, то за время т брусок не со- скользнет с транспортера, т. е. s < I. Путь, который за это время брусок пройдет относительно ленты транспортера, равен s2 = | v2/2a - tn I = u7(2p0). Полный путь бруска относительно ленты равен S = S1 + s2 = I + v ]/21/(^д) + р2/(2цд) = (v + |/2цд02/(2|ад). Количество теплоты, выделившееся за счет работы силы трения, равно Q = \xvngs = т (v + ]/2\^gl)2/2. 1.79. В первом случае (движение трубы без проскальзывания) пер- воначальный запас потенциальной энергии в поле силы тяжести пере- ходит в кинетическую энергию движения трубы, которая делится по- ровну между энергией вращения и энергией поступательного движе- ния. Во втором случае (движение с проскальзыванием) не вся потенциальная энергия перейдет в энергию движения в конце горки из- за работы силы трения. А поскольку эта энергия также распределится поровну между энергиями поступательного и вращательного движе- ния, то во втором случае скорость трубы в конце горки будет меньше. 1.80. После того как пружину отпустили, устанавливается ее рав- номерное растяжение. При этом возникают очень быстрые колебания самой пружины, которые также быстро затухают. За это время груз практически не успевает сместиться, т. е. если, совершая работу А, се- редину пружины сместили на расстояние х, то теперь вся пружина рас- 106
тянута на х. Значит, ее потенциальная энергия, которая при последую- щем колебательном движении груза равна максимальной кинетиче- ской, равна РУК = &х2/2; где к — жесткость всей пружины. При оттягивании пружины вниз за середину удлинялась лишь ее верхняя половина, жесткость которой составляет 2k, и работа, равная потен- циальной энергии растяжения верхней части пружины, А = 2к (х2/2) = = кх2. Отсюда можно заключить, что максимальная кинетическая энергия груза при последующем движении равна = А/2. 1.81. Ввиду замкнутости системы звезды будут вращаться вокруг своего центра масс по концентрическим окружностям. Уравнения дви- жения звезд имеют вид mi®2/i=F, = (1) здесь ®t, ®2 — угловые скорости вращения звезд; /ь /2 — радиусы их орбит; F — сила взаимодействия звезд, равная Gm1m2/l2, где / — рас- стояние между звездами, G — гравитационная постоянная. Из опреде- ления центра масс следует, что = т212, /1 + 12 = /. (2) Решая совместно (1) и (2), получаем ®i = ®2 = У&(т1 + Jw2)//3 = 1 + ml)IU а искомый период обращения этих звезд равен Т = 2л/ ]/l/[G(m1 + т2)]. 1.82. Пусть t>t — скорость станции до столкновения, v2 — скорость станции и метеорита сразу после столкновения, т — масса метеорита, 10m — масса станции. До столкновения станция двигалась вокруг планеты по круговой орбите радиуса R, поэтому скорость станции t>t находится из уравнения 10mv?/K = G • 10mM/R2. Отсюда Vi = yGM/R. Скорости u, t>t и v2, согласно закону сохранения импульса, связаны соотношением mu + lOmt?! = llmv2. Напишем закон сохранения импульса в проекциях на оси х и у (рис. 179): lOrnt^ = 11т1?2х, (1) mu=Umv2y. (2) После столкновения станция переходит на эллиптическую орбиту. Энергия станции с застрявшим в ней метеоритом при движении по 107
этой эллиптической орбите остается постоянной. Следовательно, ИтМ 11т ' ИтМ 11m -G-—+ —(dx + ^)=-G——+ —И2, (3) К Z K/Z Z где V — скорость станции в момент наибольшего сближения с плане- т той. Здесь мы используем формулу а для потенциальной энергии грави- тационного взаимодействия двух тел (wii и ги2): = — Gm^m^r. Со- ▼ У Рис. 179 гласно второму закону Кеплера, скорость V и скорость v2 станции сразу после столкновения связаны соотношением VR/2 = v2xR. (4) Решая совместно уравнения (1) — (4) и учитывая, что Vi = j/GM/R, на- ходим скорость метеорита перед столкновением: u=]/58GM/R. 1.83. Для тела массой т, покоя- щегося на экваторе планеты, кото- рая имеет радиус R и вращается с угловой скоростью со, уравнение движения имеет вид mo2R = тд' — N, где N — сила реакции поверхности планеты, д' = 0,01g — ускорение сво- бодного падения на планете. По условию задачи тела на экваторе не- весомы, т. е. N = 0. Учитывая, что со = 2п/Т, где Т — период обращения планеты вокруг оси, равный продолжительности земных суток, по- лучим R = (Т2/4п2) д'. Подставляя значения Т=8,6 104 с, д' «0,1 м/с2, получим R «1,8107 м= 18000 км. 1.84. Запишем уравнения движения Нептуна и Земли вокруг Солн- ца (для простоты рассуждений предположим, что орбиты круговые): wihcoh^h = CMm^/Rlb И13СО3К3 = GMm^/R^. Здесь тн, пг^, сон, С03, Кн, &з~ массы, угловые скорости, радиусы ор- бит Нептуна и Земли соответственно, а М — масса Солнца. Учтем те- 108
перь связь между угловой скоростью и периодом обращения вокруг Солнца: ©и = 2 л/Тн, ®з = 2 л/Т3. Здесь Гн — период обращения Нептуна, а 1$ — период обращения Земли вокруг Солнца. В результате находим, что период обращения Нептуна вокруг Солнца равен Тн = » 165 лет. Аналогичный результат получится и для эллиптических орбит из 3-го закона Кеплера. 1.85. Рассмотрим два пути решения задачи. 1. Условия равновесия грузов имеют вид (рис. 180) Т1=т1д, Т2 = т2д, Мд = 7\ sin осi + Т2 sin а2, 1\ cos 0ц = Т2 cos а2. Из этих соотношений можно найти углы, соответствующие положе- нию равновесия системы: М2 - m2 + mj М2 - ml + m2 sin а. =------------, sin а2 =--------------. 1 2Mmr 2 2Mm2 Очевидно, что равновесие осуществимо не всегда. Действительно, для равновесия необходимо выполнение условий 0 < оц < л/2 и О < а2 < л/2, т. е. М2 — ml + ml М2 — ml+ml О <-------------- <1, 0 <------------- < 1. 2Mmr 2Мт2 Из этих неравенств следует, что вся система будет находиться в равно- весии лишь при выполнении усло- вий М < mr + т2, М2 > | ml — ml |. 2. Рассмотрим равновесие уз- ла Л. В этом случае на него дейст- вуют три силы: Ti = "НЯ, Т2 = т2д, Т3 = Мд. Для равновесия узла необходимо, чтобы три силы Т15 Т2, Т3 образова- ли треугольник. Поскольку в тре- угольнике сумма длин двух сторон не меньше третьей, получим соот- ношение между массами m19 т2, М, обеспечивающее равновесие узла А : mr + т2> М, М + mr > т2, М + т2> т 109
/cosа, а плечо силы 1.86. Рассмотрим условия равновесия стержня в тот момент, когда он наклонен к горизонту под углом а. Силы, дейст- вующие на стержень, изображены на рис. 181. При решении задачи удобнее всего воспользоваться равенством нулю суммы моментов сил относительно точки пересечения линий действия силы тяжести тд и приложенной человеком перпендикулярно стержню силы F (точ- ка О) — при этом моменты данных сил равны нулю. Пусть длина стержня равна 2/, тогда плечо силы реакции опоры N составляет трения — //sin а + / sin а, и условие равновесия записывается в виде ап г / 1 . • \ - 1 1 +sin2« Nl cos а = гтр/1 —----Ь sin а I = г тр/------; у sin а J sin а отсюда cos а sin а cos а sin а 1 l+sin2a 2 sin2 а + cos2 a 2tga + ctga С другой стороны, сила трения не может превышать значения \iN — силы трения скольжения, поэтому 1 LL • 2 tg a + ctg a Это неравенство должно выполняться при всех значениях угла а. Сле- довательно, для нахождения минимального коэффициента трения необходимо найти максимум функции (2х2 + 1/х2)"1, где x2 = tga. Из тождества 2х2 + 1/х2 = (j/2x — 1/х)2 + 21/2 следует, что макси- мальное значение l/(2tga + ctg а) равно 1/(2|/2) = j/2/4 и достигается при х2 = tg a = j/2/2. Таким образом, искомый минимальный коэффи- циент трения равен Minin = |/2/4. 1.87. Поскольку шарнир С находится в равновесии, сумма сил, действующих на него, равна нулю. Записывая силы, действующие на шарнир С, в проекции на ось, перпендикулярную АС, получим (т + тш) д sin a = Тcos a, (1) где тш — масса шарнира. Аналогично, из условия равновесия шарнира D и условия горизонтальности среднего стержня следует, что Тcos a = F cos 0 + тшд sin a. (2) 110
Из (1) и (2) получаем Тcos а — тша sin а mg sin а F —---------------------=----------> mg sin а. COS Р COS р Таким образом, минимальная сила Fmjn, при которой средний стер- жень сохраняет горизонтальное положение, равна fmin = mg sin a = mg/2 и направлена перпендикулярно стержню BD. 1.88. Из условия задачи следует, что коэффициент трения скольже- ния карандаша по наклонной плоскости удовлетворяет условию р tg а. Действительно, карандаш, положенный перпендикулярно обра- зующей, находится в равновесии, а эго означает, что m^sina = F^, где mg — сила тяжести, Гтр — сила трения. Но F^ ц mg cos а. Следова- тельно, mg sin a \img cos a, откуда |O tg a. Таким образом, ни при каких значениях угла ф карандаш по на- клонной плоскости скользить не будет. Скатывание карандаша может начаться при таком угле ф0, когда вектор силы тяжести «выйдет» за пределы участка соприкосновения карандаша и наклонной плоскости (на рис. 183 этот участок заштрихо- ван). Для нахождения этого угла спроецируем на наклонную плоскость центр масс карандаша (точку А) и отметим точку пересечения вертика- ли, проходящей через его центр масс, с наклонной плоскостью (точку В). Очевидно, при различной ориентации карандаша, если его центр масс остается на месте, точки А и В будут неподвижны, причем если сторона шестиугольного поперечного сечения карандаша равна 2/, то АВ = 21 cos 30° tg а (причем 21 cos 30° — это радиус вписанной в шести- угольное сечение окружности). Пока точка В лежит в заштрихованной области, карандаш не бу- дет скатываться. Напишем условие начала скатывания карандаша: AD/ cos ф0 = АВ, или I/ cos ф0 = I ]/з tg a. Ill
Отсюда Фо = arccos (1/|/з tg а). Таким образом, если угол ф удовлетворяет условию arccos (l/|/3tga)sS<psS7t/2, карандаш находится в равновесии. Выражение для угла ф0 имеет смысл при условии tga> 1/|/з. Тот факт, что карандаш, положенный параллельно образующей, скатывается, означает, что tg a > 1/]/з (пока- жите это самостоятельно). 1.89. Пусть сечение поверхности описывается функцией у(х), изображенной на рис. 184. Поскольку стержень должен в любом поло- жении находиться в равновесии, это равновесие может быть только безразличным, т. е. центр масс стержня должен находиться на одном уровне при любом положении стержня. Если конец стержня, опираю- щийся на поверхность, имеет абсциссу х, то ордината у0 Другого его конца, касающегося вертикальной стенки, находится из условия '2 = [yW-y0]2 + x2, y0 = y(.x)±Vl2-x2- Поскольку стержень однороден, его центр масс находится в середине стержня. Полагая для определенности ординату центра масс равной нулю, получим [Уо + УМ]/2 = 0, отсюда у(х)= ±]/12-х2/2. Физический смысл имеет лишь решение со знаком минус. Итак, сече- ние поверхности описывается в общем случае функцией у(х) = а - ]//2 -х2/2, где а — произвольная постоянная. 1.90. В отсутствие стенки угол отклонения математического маят- ника меняется по гармоническому закону с периодом Т и угловой 112
амплитудой а. Такое же движение совершает проекция точки, обраща- ющейся по окружности радиуса а с угловой скоростью <в = 2п/Т. Абсо- лютно упругий удар жесткого стержня о стенку при угле отклонения Р соответствует мгновенному перескоку из точки В в точку С (рис. 185). Период уменьшается на At = 2у/со, где у = arccos (0/а). Следовательно, 7\ = 2л/со — 2у/<х>, а искомое отношение равно Л 1 ₽ = 1 arccos—. Т-------п---а 1.91* . Пусть шарик массой тп, падая с высоты h, упруго ударяется о неподвижную горизонтальную поверхность. Считая, что время со- ударения шарика с поверхностью мало по сравнению с интервалом времени At между двумя последовательными соударениями, найдем At = 2]/2^. За каждое соударение импульс шарика меняется на величину Ар = = 2mv = 2m]/2gh. Поэтому за одно соударение с горизонтальной по- верхностью получаем тот же импульс Ар = 2т ]/2gh. Чтобы найти среднюю силу, действующую со стороны шарика на горизонтальную поверхность, рассмотрим интервал времени т» At. За время т импульс, передаваемый горизонтальной поверхности, равен т 1/2^Лт АР = Ар-— = 2т —-77= тдт. Следовательно, усредненную по интервалу времени т силу, действую- щую на горизонтальную поверхность со стороны прыгающего шари- ка, найдем из соотношения Fcp = АР/т = тд. По условию задачи масса М чашки весов намного больше массы т шарика. Поэтому на медленное колебательное движение чашки ве- сов будут накладываться почти периодические удары шарика. Средняя сила, действующая на чашку со стороны шарика, будет равна Fcp = тд. Следовательно, искомое смещение Ах положения равновесия весов бу- дет равно Ах = тд/к. 1.92. Сила, действующая на бусинку, находящуюся в какой-либо точке А, в направлении, касательном к проволоке, равна F = mg cos а, где а — угол между касательной в точке А и осью ординат (рис. 186). Для того чтобы длина участка проволоки от начала координат до бу- синки менялась по гармоническому закону, необходимо, чтобы сила 113
F в точке А была пропорциональна длине /4. Но F < тд, а /4 неограни- ченно возрастает. Следовательно, должна существовать точка В, в ко- торой условие пропорциональности нарушается. Это означает, что ко- лебания с амплитудой 1В уже не могут быть гармоническими. 1.93. Из уравнений движения брусков = Fynp, 2та2 — ^упр» где Fynp — сила упругости пружины, следует, что их ускорения в каждый момент времени связаны соотношением а2 = -ар- Отсюда вы- текает, что в инерциальной системе отсчета, связанной с центром масс брусков, последние совершают колебания в противофазе, причем относительные смещения брусков относительно их положений равно- весия связаны тем же соотношением, что и ускорения: Дх2 = — Дхх/2. Тогда Fynp = —(3/2) к Дхх = Зк Дх2. Следовательно, период малых продольных колебаний равен Т=2тг|/2т/(3/с). 1.94. Отметим горизонтальный диаметр АВ = 2г бревна в момент, когда бревно проходит положение равновесия. Рассмотрим теперь бревно в момент времени, когда нити подвеса отклонены от вертикали на какой-либо малый угол а (рис. 187). Если ника с длиной подвеса проскальзывания нитей нет, то нетрудно из геометрических соображений получить, что отмеченный диаметр АВ всегда в процессе колебания бревна остается горизонтальным. Действительно, если EF1DK, то FK = = 2r tg а » 2га. Но BD » FK/2 « га. Следо- вательно, как и указывалось, л. BOD = а. Так как диаметр АВ с течением времени остается горизонтальным, то бревно дви- жется плоскопараллельно, т. е. скорости всех его точек одинаковы в каждый данный мо- мент времени. Поэтому движение бревна синхронно колебанию математического маят- /. Таким образом, период малых колебаний бревна равен Т = 2л///0. 114
1.95. Период колебания маятника в направлении, перпендикуляр- ном рельсам, равен (/ — длина невесомой нерастяжимой нити), так как груз М при этом покоится (рис. 188). Период колебаний в плоскости, параллельной рельсам («парал- лельные» колебания), найдем из условия неподвижности центра масс системы. Положение. центра масс системы определяется уравнением тЦ = М (/ — lY): Таким образом, шарик совершает колебания при неподвижной точке О, находясь от нее на расстоянии = М1/(М + т). Отсюда пе- риод «параллельных» колебаний маятника будет равен Т2=2п\/М1Д_(т + М)д]. Следовательно, Tj/Ti = ]/МЦт + М). 1.96. Со стороны стержней на груз действует сила = 2FH cos а, на пружину — F2 = 2FH sin а (см. рис. 48). Согласно закону Гука, F2 = — (1,5/ — 21 sin а) к, где к — жесткость пружины. В результате Ft = l,5lk ctg а — 2lk cos а. Чтобы найти период малых колебаний, нужно определить силу АГ, действующую на груз при малом изменении высоты груза А/1 относительно положения равновесия h0 = 21 cos а0. Получим АГ = 1,5//с A (ctg а) — 2lk A (cos а), где A (ctg а) = ( - а | Аа =-----------, A (cos а) = — sin а0 • Аа. \ da Ja = aQ sin2 а0 Следовательно, поскольку A/i = — 2/sina0Aa, то АГ = — 1,5/с ——F 2kl sin а0 Аа — — 5kl Аа = — 5к &h, sin2 а0 так как sin a0 = 1/2. Период малых колебаний груза найдем по формуле Т = = 2л ]/т/(5к), где т — масса груза, определяемая из условия равнове- сия: l,5kl ctg a0 — 2.1k cos a0 = mg, m = (j/3/2)(W/3). 115
Таким образом, Т= 2л|/)/з//(10з). 1.97. Кинетическая энергия обруча в каждый момент времени складывается из кинетической энергии движения центра масс обруча и кинетической энергии вращения обруча вокруг центра масс. Так как скорость точки А обруча все время равна нулю, то обе части кинетиче- ской энергии равны (скорость центра масс равна линейной скорости вращения относительно центра масс). Поэтому полная кинетическая энергия обруча равна mv2 (m — его масса, v — скорость его центра масс). Согласно закону сохранения энергии,- mv2 = mg (г — hA), где hA — высота центра масс обруча над точкой А в каждый момент вре- мени. Следовательно, скорость центра масс обруча равна v — — ]/#(г — /ц). В то же время скорость маятника В в момент, когда он находится на высоте hA над осью вращения 4, равна v = ^2g(r — hA), т. е. в |/2 раз больше. Таким об- разом, маятник достигнет положе- ния равновесия в |/2 раз быстрее, чем обруч, т. е. за время t = t/j/2 « 035 с. 1.98. Заметим, что малые коле- бания груза происходят относи- тельно неподвижной оси АВ (рис. 189). Пусть DC LAB. Тогда малые колебания груза эквива- лентны колебаниям математического маятника той же массы, но с длиной подвеса Г = Lsin а = L _________.... 1//2 + (1/2)2 и ускорением свободного падения Ь/2 д = д cos а = д ]/l2 + W2)2 где L = AD. Таким образом, искомый период малых колебаний системы будет равен Т = 2л ]/Т/д' = 2п ]/2Цд. 1.99. Для решения задачи достаточно заметить, что движение ка- челей есть вращение вокруг оси, проходящей через точки крепления ве- ревок, т. е. система представляет собой «наклонный математический маятник» (рис. 190). Составляющая силы тяжести тд, направленная вдоль оси вращения, не влияет на колебания, а перпендикулярная со- ставляющая тд sin а является фактически возвращающей силой. В ре- 116
зультате, используя формулу для периода математического маятни- ка, найдем Т = 2л |/h/(g sin а), где h — ]//? + Р2 \/а2 + Ь2 ' поэтому период малых качаний равен Т=2л]/М2/(од). 1.100. Период математического маятника обратно пропорциона- лен корню квадратному из ускорения свободного падения: т~1/]4 Пусть модуль ускорения лифта равен а. Тогда период маятника при движении лифта вверх с ускорением а будет пропорционален тв ~ i/p7T7 при движении лифта вниз с ускорением а ти ~ i/i/Г7^ Ясно, что время, которое отсчитывают маятниковые часы при движе- нии вверх с ускорением а, пропорционально отношению времени tB равноускоренного движения вверх к периоду Тв: tB = (в/Тв Время, которое отсчитывают маятниковые часы при движении вниз с ускорением а, равно tH = /К ~ tn g — а. По условию задачи времена равноускоренного движения вниз и вверх равны: tH = = ^1/2, где Ц — полное время движения лифта с ускоре- нием. В итоге время, которое отсчитывают маятниковые часы за рабо- чий день, равно t' «(t,/2) \]/(g + a)/g + +10. Здесь t0 — время движения лифта без ускорения. Неподвижные маятни- ковые часы показали бы время t ~ ti + t0. 117
Легко убедиться, что выполняется неравенство |/д + а + \/д — а < 2 |/д. Действительно, /j/g + o + l/g-д \2 _ д + |/д2 - а1 <} \ 2]Гд Отсюда вытекает, что маятниковые часы в лифте в среднем идут мед- леннее: t'<t, и, следовательно, лифтер переработает. 1.101. Как следует из закона Паскаля, в сообщающихся сосудах давления газа на одинаковой высоте совпадают. Поскольку трубы ма- нометра сообщаются с атмосферой, давление внутри них будет ме- няться с высотой по тому же закону, что и давление атмосферного воздуха. Это означает, что давление воздуха на жидкость, находящую- ся в различных коленах манометра, одинаково и равно давлению ат- мосферного воздуха на той высоте, где находится манометр. Таким образом, показания манометра нулевые, поскольку нет разности давлений. 1.102. Выберем нулевой уровень отсчета потенциальной энергии совпадающим с дном внешней трубки. Тогда в начальный момент по- тенциальная энергия ртути равна 1Ун = 25/рртз(//2)=рртд5/2. Потенциальная энергия ртути в конечный момент (момент отрыва внутренней трубки, рис. 191) равна (по условию I > h) WK = 25хРртд(х/2) + ShppTg(x + h/2), где х — уровень ртути во внешней трубке в момент отрыва; этот уро- вень находится из условия постоянства объема ртути: 2Sx + Sh = 2SI, x = l- h/2. Разность потенциальных энергий равна сумме искомой работы А внешних сил и работы силы атмосферного давления, действовавше- го на поверхность ртути во внешней трубке и на верхний (запаянный) конец внутренней трубкй. Смещение уровня ртути во внешней трубке равно I — х, соответствующая работа силы атмосферного давления равна pQS (l-x)= pprgSh (I - х). Смещение запаянного конца внутренней трубки равно I — (/ + х), соответствующая работа равна — p0Sx = — PpTgShx. В итоге искомая работа внешних сил равна A=WK-WH- PprgSh (I - 2х) = p^gSh (I - 3h/4). 1.103. Давление у дна «вертикального» цилиндра равно р = pQ + + Рв^> где Ро — атмосферное давление, рв — плотность воды, д — уско- рение свободного падения. По закону Паскаля то же давление дей- ствует на нижний край поршня, находящегося в «горизонтальном» ци- линдре. Вообще давление воды на часть поршня, отстоящую от нижнего края на расстояние х по вертикали, равно р — рдх (рис. 192). 118
Рассмотрим части поршня, представляющие собой узкие (шири- ной Ах) горизонтальные полоски, отстоящие на равные расстояния а от его центров. Сила давления воды на верхнюю полоску равна [р- РвЗ ('• + <’)] Д$, а сила давления на нижнюю [р - РвР (»• - о)] Д5> где AS — площадь полоски. Сумма этих сил пропорциональна площа- ди полоски, причем коэффициент пропорциональности, равный 2(р — — рдг), не зависит от а. Отсюда следует, что полная сила давления во- ды на поршень равна (Р - Mr)nr2 = [ро + M(h - г)] пг2. Чтобы поршень находился в равновесии, необходимо равенство этой силы силе атмосферного давления, действующей на поршень слева и равной ропг2. Отсюда h = г, т. е. поршень находится в равновесии, если уровень воды в вертикаль- ном цилиндре равен радиусу горизонтального. Из рассмотрения вид- но, что равновесие устойчивое. Рис. 191 Рис. 192 1.104. Условием полного погружения тела является М>РвР, где М — масса тела, V — его объем. В нашем случае получим Л/ = шПр + Шдд, V = шПр / Рпр + тИдд / Рдд. Отсюда следует, что минимальная масса проволоки равна ™ал = Рал ( Рв — Рпр) “ \ шпр (Рал — Рв) Рпр ® 1,6тпр. 119
1.105. Ясно, что в положении равновесия шар находится на неко- торой высоте h над дном водоема, а цепь частично лежит на дне, ча- стично висит в воде между дном и шаром, причем висящий участок цепи вертикален (рис. 193). Данные задачи таковы, что можно заранее утверждать, что шар целиком находится под водой (в противном слу- чае почти вся цепь оказалась бы висящей, что невозможно ввиду боль- шой плотности железа). Высота h в таком случае получится из условия равенства суммарной силы тя- жести шара и висящего участка цепи h h Рис. 194 и действующей на них выталкивающей силы: h - D/2 М 4- т---------- ( т h - D/2 \ Iff = Рв И +------------i---- »• \ Рж I / Отсюда PbV-M -------------<= 1,6 м. W(1 - Рв/Рж) Глубина, на которой плавает шар, равна Н — h = 1,4 м. 1.106. -Условием равновесия рычага является равенство моментов сил (рис. 194). В воздухе этими силами являются силы тяжести тел тгд и т2д. Поскольку они различны, различны и соответствующие плечи Zx, /2, так как midli = "h-Qli- При погружении рычага в воду на тела дополнительно к силе тя- жести начинает действовать выталкивающая сила со стороны воды, которая пропорциональна объему тела. В силу условий задачи = F2. В таком случае #(m2£-F2)Z2. Поэтому равновесие рычага нарушится. 1.107. Объем погруженной в воду части каждой коробочки ме- няется на одну и ту же величину А / = m/рь, где т — масса предмета, рв — плотность воды. Поскольку изменение уровня воды в каждом из сосудов определяется только величиной А/ и сосуды одинаковы, уров- ни воды в них Изменятся на одну и ту же величину. 120
1.108. Пусть объем стального шарика равен 7, а объем его части, погруженной в ртуть, равен 70 до наливания воды и Vr после того, как вода полностью закроет шарик. Величина Vo находится из условия Рст У =. Ррт Ио, где Рст и Ррт “ плотности стали и ртути. Поскольку давление воды передается через ртуть на нижнюю часть шарика, выталкивающая сила, действующая на него со стороны воды, равна рв(У— VJg, где ft, — плотность воды, а выталкивающая сила со стороны ртути — p^THi^. Условие плавания шарика теперь примет вид РстИ^РртИх + Рв^-ИД откуда JZ1 — ^ст у Ррт ~' Рв Таким образом, отношение объемов погруженной в ртуть части шари- ка в первом и втором случаях равно Ио _ Рст Ррт “ Рв _ 1 “ Рв / Ррт Ррт Рст “ Рв 1 “ Рв / Рст Поскольку ррТ > рст, то Vo > V19 т. е. объем погруженной в ртуть части шарика при наливании воды уменьшится. 1.109. Известно, что при таянии льда, плавающего в сосуде с во- дой, уровень воды не меняется. В нашем случае будем считать уровень воды в начальный момент (отсчитываемый от дна сосуда) равным h0, уровень поверхности масла h (рис. 195). Если бы в соЬуде находилась только вода, то ее уровень при условии не- изменного положения куска льда относительно сосуда подчинялся бы условиям h0 <ht <h. При таянии вода, образующаяся из льда, имеет объем, равный объему заштрихованной области на рисунке. Поскольку часть этого объема выходит за пре- делы поверхности воды > h0), то при таянии льда уровень воды повышается. С другой стороны, поскольку < h, масло заполняет образующуюся «лунку», т. е. общий уровень жидкости в сосуде понижается. 1.110. Пусть х — длина части палочки, свешивающейся внутрь ста- кана, а у — длина ее наружной части. Тогда длина палочки равна х + у. Центр палочки находится на расстоянии (х + у) /2 от ее концов и на расстоянии (у —х)/2 от наружного края. Условия равновесия получим из равенства нулю суммы моментов сил относительно края стакана: (Гш-ЕА)х = М0(у-х)/2, 121
где Гм = тшд = Рал^У - сила тяжести шарика, а ГА = Рв^И/2 - выталкивающая сила (сила Архимеда), причем V = (4/3) пг3 — объем шарика. Искомое отношение равно у/х=1+2(Гш-ГА)/(М^)«1,5. 1.111. При свободном падении сила давления атмосферного воз- духа больше не компенсируется весом столба ртути независимо от вы- соты столба. В результате ртуть заполнит трубку барометра целиком, т. е. до деления 1050 мм. 1.112. В сосуде с жидкостью, движущемся b горизонтальном на- правлении с ускорением а, поверхность жидкости становится наклон- Рис* 196 ной плоскостью. Угол наклона ср определяется теми условиями, что сумма силы давления F и силы тяжести тд, действующих на эле- мент поверхности, равна та и сила давления нормальна к поверх- ности. Отсюда tg <р = а/д. В силу закона сообщающихся сосу- дов поверхности жидкости в коленах манометра принадлежат упо- мянутой выше наклонной плоскости (рис. 196). Из геометрических соображений очевидно, что tg9 = (ft2-ft1)/(ft2 + ft1); отсюда а = з(Л2-Л1)/(й2 + Л1). 1.113. На слой воздуха, находящийся на расстоянии х от начала салона и имеющий толщину Дх, действует сила давления, равная [р(х +Дх)-р(х)]5, где S — площадь поперечного сечения салона. Поскольку относительно салона воздух покоится, уравнение движения рассматриваемой массы воздуха имеет вид pS Аха = [р(х + Дх) — р(х)] S. Устремляя Дх к нулю, получаем dp/dx = ра; отсюда р(х) = р1 + рах. 122
Поскольку среднее давление в салоне остается неизменным и равным атмосферному давлению р0, постоянная pt находится из условия Ро = Pi + М/2, где I — длина салона. Таким образом, в середине салона давление рав- но атмосферному, а в начале и в конце салона давление соответствен- но меньше и больше атмосферного на величину Ар = ра//2 « 0,03 Па. 1.114. Рассмотрим условия равновесия относительно трубки массы воды, находящейся между поперечными сечениями, отстоящими на х и х + Ах от оси вращения. Эта часть жидкости, масса которой равна PbSAx, равномерно вращается с угловой скоростью со под дей- ствием сил давления, действующих на ее боковые поверхности. Обо- значая давление в сечении х как р(х), находим [р (х 4- Ах) — р (х)] 5 = р5 Ахсо2 (х 4- Ах/2). Устремляя Ах к нулю, получаем уравнение dp/dx = РвШ2х; отсюда р(х)= ft,w2x2/2+ р0. Используя данные задачи Pi = Р(п) = РвИ2г?/2 + р0, Р2 = Р Ог) = Рв w2 rf/2 + р0, находим угловую скорость вращения трубки: 1.115. Пусть показатель степени а таков, что тело, имеющее на- чальную скорость v, проходит в среде конечное расстояние s(v). По- скольку скорость тела при его движении в среде монотонно умень- шается, ясно, что s(v)>s(Vi) при v>vv Очевидно также, что s(v) стремится к нулю при v->0. Условием остановки тела является равен- ство работы А силы сопротивления начальной кинетической энергии тела: mv2/2 = А. (1) Поскольку сила сопротивления монотонно убывает вместе со ско- ростью тела в процессе его движения, то А pv’s (v). (2) 123
Подставляя (2) в (1), получаем откуда следует, что при а>2 нарушается условие lims(r) = 0. Таким t>-*0 образом, при а > 2 тело не может затормозиться на конечном участке пути. 1.116. Сила притяжения тела массой т к Марсу на его поверхно- Ммт сти в соответствии с законом всемирного тяготения равна G , где Мм — масса Марса, а ~ его радиус. Это значит, что ускорение свободного падения на* поверхности Марса равно дм = GMм /Rm- Если масса марсианской атмосферы равна тм, то она притягивается к по- верхности с силой тмЯм* которая и представляет собой силу давления атмосферы, т. е. давление на поверхности Марса равно рм = = Аналогично для соответствующих параметров на Зе- мле рч — тздз/(4пКз). Отношение масс атмосфер Марса и Земли равно тм _ Рм ' 4тс/?м0з . т3 Рз*4лКз0м ’ учитывая, что Мм = (4/3) uRm Рм (и аналогично для Земли), получаем, подставляя данные из условия, тм Рм Рз ^3 44q-3 ™з Рз Кз Рм ~ ’ Отметим, что при решении задачи мы фактически считали атмосферу расположенной вблизи поверхности планеты. Это действительно так: высота атмосферы много меньше радиуса планет (например, на Земле на высоте 10 км уже нечем дышать, а радиус Земли равен R3 » 6400 км!).
2. ТЕПЛОТА И МОЛЕКУЛЯРНАЯ ФИЗИКА
2.1. Так как вертикальные цилиндры представляют собой сооб- щающиеся сосуды, то после увеличения массы 1-го поршня равновесие наступит лишь после того, как он «ляжет» на дно своего цилиндра, а весь газ перейдет во 2-й цилиндр. Поскольку давление газа и его температура остаются неизменными, то и полный объем, занимаемый газом, должен остаться неизменным. Отсюда заключаем, что S\/i0 + + S2h0 = S2h, где S1 и S2 — поперечные сечения поршней 1-го и 2-го цилиндров, a h — высота, на которой будет находиться 2-й поршень, т. е. как раз искомая разность высот (ведь 1-й поршень лежит на дне). Вначале давления, создаваемые поршнями, были одинаковы; таким образом, = m2g/S2, SiIS2 = mllm2, а, значит, h = h0(m1/m2 + 1) = 0,3 м. Рис. 198 2.2. Если температура стенок сосуда Тс совпадает с температурой газа Т, то молекула, ударяясь о стенку, меняет нормальную компо- ненту импульса рх на — рх. Значит, суммарное изменение импульса равно 2рх. Когда температура стенок Тс > Т, газ нагревается. Это оз- начает, что молекулы газа от- скакивают от стенки с большей скоростью, чем налетают, а сле- довательно, и с большим им- пульсом. В результате измене- ние импульса будет больше, чем 2рх (рис. 197). Если же Тс < Т, то газ охлаж- дается, т. е. молекулы газа отска- кивают от стенки с меньшим импульсом, чем налетают на в этом случае будет меньше, чем 2рх (рис. 198). Так как в соответствии со вторым законом Ньютона изменение импульса пропорционально средней силе, то давление газа на стенки больше, когда стенки теплее газа (Тс > Т). 2.3. Работа А газа за цикл определяется площадью на рКдиа- грамме, ограниченной циклом, т. е. площадью трапеции (см. рис. 57): А = (р2-р1)(К3-Г2 + Г4-П)/2. Нетрудно выразить все величины через давление и объем в точке 1: р1 и Действительно, из закона Гей-Люссака следует, что V3 = = ^2^3/T2 = ^±Т3/Т2 и V^V.TJT^V.T./T,, а из закона Шарля — р2 = PiТ2/1\. Подставляя эти значения в выражение для нее. Ясно, что изменение 126
работы, находим Т, - Т. ( Г, Т3 Л А = р V.—---- —+ — -2 • F1 1 Т, \Т2 Т2 J Уравнение Менделеева — Клапейрона для п молей идеального газа позволяет написать p1V1 = nRT^ и в результате окончательно получим А = nR(T2 - Т\)(Т2/Т\ + Т3/Т2 - 2). 2.4. Из рис, 58 видно, что на участках 1—2 и 3 — 4 реализуется прямая пропорциональная зависимость давления от температуры, т. е., как следует из закона Менделеева — Клапейрона, объем газа при этом не меняется, а значит, и работы газ не совершает. Необходимо, таким образом, найти работу газа лишь при изобарических процессах 2 — 3 и 4 — 1. На участке 2 — 3 совершенная работа будет равна А23 = = Рг (*з - *2), а на участке 4-1 А41 = рх (Ух - У4). Полная работа А газа за цикл равна A = P2(V3-V2) + P1(V1-y4). Уравнение Менделеева — Клапейрона для 3 молей идеального га- за записывается в виде pV = 3RT, и, следовательно, p1V1 = 3RT1, P1V4 = 3RT4, p2V2 = 3RT2, p2V3=p3V3 = 3RT3. Подставляя эти значения в выражение для работы, окончательно получаем А = 3R(T1 + Т3-Т2- Т4) = 2-104 Дж = 20 кДж. 2.5. Выполнение цикла фактически эквивалентно выполнению двух простых циклов 1 ->0->2-+1 и 0->4-> 3 ->0 (см. рис. 59). Работа газа определяется площадью соответствующего цикла на рУ-диаграмме. Однако если в первом цикле она положительна, то во втором отрицательна (работа совершается над газом). Нетрудно найти работу Лх, совершенную в первом цикле: А = (Ро-Р1)(^-^)/2. Что касается цикла 0->4-> 3 ->0, то соответствующий ему на рУ-диа- грамме треугольник подобен треугольнику, отвечающему первому циклу. В результате работа А2 во втором цикле будет Л2 = - Al (р2 - Ро)7(Ро ~ Pi)2- (Площади подобных треугольников относятся, как квадраты длин со- ответственных элементов, в данном случае — высот.) Полная работа А за цикл будет, таким образом, равна А = Ai Е1 - (Р2 - Ро)2/(Ро ~ Pi)2] » 250 Дж. 127
2.6. Согласно 1-му закону термодинамики, при переходе газа из состояния 1 (pOi Vo) в состояние 2 (р15 (рис. 199) количество теплоты AQi, полученное газом, равно Д21 = A^i •+ а19 где ДИ! — изменение его внутрен- ней энергии, a — работа, совер- шенная газом, причем ^41 =(Ро ~ Ио)/2. При переходе газа из состояния 1 в состояние 3 (p2i V2) (точки 2 и 3 лежат на одной изотерме) выпол- няются соотношения Д02 = 2 4- А2, ^2=(Po+P2)(H2-Fo)/2. Поскольку конечная температура газа в состоянии 2 и в состоянии 3 одна и та же, то Д17\ = A172- Чтобы выяснить, в каком из процессов газу сообщается большее количество теплоты, надо сравнить работы А1 и А2. Сделаем это: А - А2 = (р0 + P1)(FX - Vo)/2 -(pQ + p2)(V2 - Vo)/2 = = [(Po^i - P(A2) + (P2^o " Pi^o)]/2 < 0, так как p0V1 < p0V2i p2V0 < PiV0. Следовательно, A2 > А1 и &Q2 > AQ15 т. e. в процессе 1 3 газу сообщается большее количество теплоты. 2.7. Поскольку водород диффундирует через все перегородки, то он равномерно распределится по всему сосуду, и давление водорода во всех трех частях сосуда будет равно тн2 RT Рн2 = -- ,7- Цн2 V (ведь если газ проходит через перегородку, то в равновесии его давле- ние по обе стороны перегородки должно быть одинаковым). Азот может диффундировать лишь через правую перегородку, по- этому он заполнит среднюю и правую части сосуда (см. рис. 61) об- щим объемом (2/3) V; его давление будет 3RT Кислород не диффундирует через перегородки, и давление кислорода в средней части сосуда равно wq2 3RT 128
Согласно закону Дальтона, давление во всех трех частях сосуда будет равно сумме парциальных давлений находящихся там газов, т. е. Pi = Рн2 ~ 1,3 • Ю9 Па = 1,3 ГПа, Р2 = Рн2 + Ро2 + Pn2 « 4,5 • 10’ Па = 4,5 ГПа, Рз = Рн2 + Pn2 ~ 2,0-10’ Па = 2,0 ГПа. 2.8* . Определим сначала скорость спускаемого аппарата. Для это- го заметим, что изменение давления Ар с изменением высоты Ah свя- зано соотношением &p=-pg&h, (1) где р — плотность газа. Из закона Менделеева — Клапейрона следует, что р = (р/ц)КТ(здесь Т — температура газа именно в той точке, вбли- зи которой мы интересуемся изменением давления). Учитывая, что Ah = — v Аг, где v — скорость спуска аппарата, Аг — время спуска, мож- но записать выражение (1) в виде Ар Аг Зная отношение Ap/At, т. е. тангенс угла наклона касательной в конеч- ной точке А графика, с помощью соотношения (2) найдем скорость v. (Отметим, что, поскольку в левой части (2) стоит отношение Ар/р, нам безразлично, каков масштаб графика по оси ординат.) Находя из гра- фика (Ар/Аг)р-1'и подставляя ц = 44 г/моль для СО2, получим, что скорость спуска аппарата космического корабля равна RT Ар 8,3 Дж/(К-моль)-7-102 К _ , V =---------=-------z-------;--------------- ~ 11,5 М/С. др. p&t 10 м/с2-44-’10 3 кг/моль-1150 с Перейдем теперь к решению второй части задачи. Учитывая, что скорость аппарата равна v = 11,5 м/с, на высоте h = 15 км над поверх- ностью планеты он был за 1300 с до посадки, т. е. этому соответствует время t = 2350 с. С помощью соотношения (Ар/Аг)р-1 в этой точке графика из выражения (2) найдем искомую температуру Т^ : ^w/рД^Х к К \ Др / 2.9. Так как поршень, когда на него положили груз, переместился на расстояние h, то это означает, что объем газа уменьшился на вели- чину hS и стал равным V — hS. Давление газа под поршнем равно ат- мосферному давлению р0 плюс давление Mg/S, создаваемое грузом, т. е. р0 4- Mg/S. В результате можно записать уравнения Менделеева — Клапейрона для газа до того, как на поршень положили груз, и после 5 А. И. Буздин и др. 129
этого: р0Г = иЯТн, (1) (р0 + Mg/S)(V — hS) = nRTK. (2) Здесь Тн и Тк — начальная и конечная температуры газа. Поскольку по условию задачи газ теплоизолирован, то, как сле- дует из 1-го закона термодинамики, вся совершенная над ним работа А пойдет на изменение внутренней энергии газа, т. е. Л = (3/2)иК(Тк — — Тн) (внутренняя энергия 1 моля идеального газа U = (3/2)RT). Не- трудно сообразить, что работа равна А = Mgh, значит, Mgh = (3/2) nR(TK—Tn). (3) Вычитая почленно из (2) уравнение (1) и используя для Тк — Тн выра- жение (3), получим уравнение для определения h: MgV/S - Mgh - pohS = (2/3) Mgh. (4) Отсюда найдем, что MgV S(p0S + Mg/3)' и, подставляя h в уравнение (2), определим конечную температуру газа: т (p0S + Mg)(3p0S-2Mg)V к (3p0S + Mg)SnR 2.10. В соответствии с 1-м законом термодинамики подведенное к газу количество теплоты Q идет на изменение его внутренней энер- гии A U и совершение газом работы А : Q = AU+'A. Внутреннюю энергию 1 моля идеального газа U можно записать в ви- де U = СуТ= (3/2) КТ, т. е. А17 = (3/2) К АТ. Работа, совершаемая газом при постоянном давлении р, равна А = рАУ = pSAx, где Ах — смеще- ние поршня. Давление газа равно Р = Ро + WS, т. е. представляет собой сумму атмосферного давления и давления, создаваемого поршнем. Наконец, из закона Менделеева — Клапейрона pV= КТ можно найти связь между изменением объема АУ и измене- нием температуры АТ при постоянном давлении: рАУ = КАТ. Подставляя выражения для АСУ и Л в 1-й закон термодинамики и учитывая связь между А У и АТ, получим Q = р А У + (3/2) р АУ = (5/2) pS Ах. (1) 130
Поскольку за единицу времени нагреватель выделяет количество те- плоты q, то Q = q At, где At — соответствующий промежуток времени. Скорость движения поршня равна v — Лх/Лг, и из выражения (1) находим 2 q v =-----------. 5 p0S + Mg 2.11*. При очень сильном сжатии газа начинает сказываться от- талкивание между его молекулами — необходимо учитывать конеч- ность их размеров. Это означает, что при прочих равных условиях давление реального газа превосходит давление идеального газа тем больше, чем сильнее сжат газ. В результате, если при постоянной тем- пературе произведение pV идеального газа постоянно, то для реальных газов оно будет расти по мере уменьшения объема газа. 2.12*. Рассмотрим промежуточное положение поршня, когда оц сместился на величину у от своего первоначального положения. Пусть при этом давление газа в правой части сосуда равно р2, а в левой Поскольку поршень при этом находится в равновесии, то сумма сил, действующих на поршень, равна нулю: (р2 — Pi)S— 2ку = 0, (1) где 5 — площадь поршня. При последующем небольшом перемещении поршня Ау полная работа газа ЛА равна ЛА = ААХ + АА2, где АА2 — работа газа правой части, ЛАГ — работа газа левой части, причем AAj + АА2 — р2 Ау5 - рг Ау5 = (р2 - рг)Лу5 = 2ку Лу. (2) Таким образом, к моменту смещения поршня на величину х = 1/2 полная работа газа будет равна сумме потенциальных энергий, запа- сенных в пружинах: к / I \2 4 = 2— - . 2 \ 2 J (3) Если к газу в правой части подвели количество теплоты 2, а газ в левой части передал количество теплоты Q термостату, то полное сообщенное системе количество теплоты будет Q — Q\ и можно напи- сать (1-й закон термодинамики) к ( I \2 Q-Q' = 2 — +А(7, (4) где Ли — изменение внутренней энергии газа. Поскольку поршень не- теплопроводящий, то температура газа слева не меняется и все изме- нение внутренней энергии газа A U обусловлено нагревом газа справа на АТ, причем для п молей идеального газа A U = п (3/2) R АТ 5* 131
Повышение температуры АТ найдем из условия равновесия в конце процесса. Давление газа р в правой части сосуда в соответствии с законом Менделеева — Клапейрона равно р = nR(T + AT)/[S(/ + Z/2)], с другой стороны, оно должно быть равно сумме давления газа слева р' = = nRT/[S(l —1/2)] и давления, создаваемого пружинами, p" = 2kl/(2S), т. е. 2nR(T + AT)/(3S/) = 2nRT/(Sl) + kl/S. Отсюда находим, что АТ = 2Т 4- 3kl2/(2nR), и окончательно с по- мощью (4) получаем Q' = Q-3nRT-(5/2)kl2. 2.13. Пусть 7\ — начальная температура газа под поршнем, Т2 — температура газа после того, как системе сообщено количество теплоты AQ. Поскольку трение отсутствует и сосуд теплоизолирован, вся теплота AQ идет на изменение АРИ внутренней энергии системы, т. е. AQ = AW Изменение внутренней энергии системы складывается из изменения внутренней энергии газа и изменения потенциальной энергии сжатой пружины (так как теплоемкостью сосуда, поршня и пружины пренебре- гаем). Внутренняя энергия 1 моля идеального одноатомного газа при на- гревании от температуры 7\ до температуры Т2 увеличивается на ДЖ,=(3/2)К(Т2-Т,). (1) Потенциальная энергия сжатой пружины изменяется на величину к ЛИ'г = -(х22-х21), (2) где к — жесткость пружины, а хх и х2 — значения абсолютного смеще- ния левого конца (деформации) пружины при температурах газа 7\ и Т2 соответственно. Найдем связь между параметрами газа под порш- нем и деформацией пружины. Из условия равновесия поршня следует, что р = F/S = kx/S, х = pS/k, (3) где р — давление газа, 5 — площадь поршня. Согласно закону Менде- леева — Клапейрона, для 1 моля идеального газа pV = RT. При дефор- мации х пружины объем газа под поршнем равен V = xS и р = = RT/(xS). Подставляя это выражение для р в (3), находим x2 = RT/k. (4) 132
Таким образом, изменение потенциальной энергии сжатой пру- жины при нагревании системы равно Д1У2 = (Я/2)(Т2-Т1). Полное изменение внутренней энергии системы при нагревании от температуры 7\ до температуры Т2 равно Д W = Д + Д W2 = 2R (Т2 - ТД и теплоемкость системы составит С = &QJ&T = Д1//(Т2 “ Л) = 2К- 2.14. Проанализируем работу тепловой машины, использующей цикл, который состоит из двух изотерм и двух изохор (рис. 200). Пусть температура холодильника Г. (соответствующая нижней изотерме), а нагревателя Т2 (соответствующая верхней изотерме). На изохориче- ском участке 1—2 объем газа не меняется, т. е. работы он не совер- шает, однако его температура уве- личивается от 7] до Т2, и, значит, к газу подводится некоторое коли- чество теплоты Qi. На изотерми- ческом участке 2 — 3 внутренняя энергия газа постоянна и вся подво- димая теплота Q2 идет на совер- шение работы: Q2 = А2. На изохорическом участке 3—4 Рис. 200 температура газа возвращается к своему начальному значению 7j, т. е. от газа отбирается коли- чество теплоты Qi. При изотермическом процессе 4 — 1 совершаемая газом работа отрицательна и, следовательно, от него также отбирается теплота. Таким образом, полная теплота, подведенная за один цикл к газу, составляет Qi + А2. Работа газа за цикл, как видно из рис. 200, складывается из положительной работы А2 на участке 2 — 3 и отрица- тельной Л4 на участке 4 — 1. Сравним давления в точках, соответствующих одинаковым объе- мам, на участках 4 — 1 и 2 — 3. Из закона Шарля следует, что отноше- ние этих давлений равно Тх/Т2, а, значит, работа газа Л4 = = — (TJT2)A2. Полная работа за цикл будет а к. п. д. А = А2 + А4 = (1-Т1/Т2)А2, А 1 - TJT2 = // <1~Г^ 61+Л2 \+qja2 133
т. е. к. п. д. тепловой машины, использующей цикл, состоящий из двух изотерм и двух изохор, меньше к. п. д. 1 — 1\/Т2 идеальной тепловой машины Карно. 2.15* . Найдем вначале ускорение свободного падения дп на по- верхности планеты. С одной стороны, известно, что сила притяжения тела массы m к планете равна тдп, с другой стороны, из закона все- мирного тяготения следует, что она равна GmM/r2, где G — гравита- ционная постоянная. Отсюда получаем дп = GM/r2. Давление р, созда- ваемое атмосферным столбом высоты h на поверхности планеты, будет равно Р=Р9Л (1) где р — плотность атмосферы. Здесь при определении давления атмос- ферного столба ускорение свободного падения считаем не зависящим от высоты. Это предположение оправданно, поскольку по условию за- дачи высота атмосферы намного меньше радиуса планеты г (h г). Написав уравнение Менделеева — Клапейрона для массы М газа, занимающей объем К, в виде pV = (M/p)RT и учитывая, что р = М/К, находим р = рц/(КТ)- Подставляя это выражение для р в (1) и сокращая на р, определяем температуру Т атмосферы на поверхности планеты: T = wnh/R = iiGMh/(Rr2). 2.16. Здесь необходимо учитывать, что передаваемая в единицу времени теплота пропорциональна разности температур. Введем обо- значения: Ту1, Ту2 и Тк1, Тк2 — температуры на улице и в комнате в первом и втором случаях. Тепловая мощность, рассеиваемая бата- реей в комнате, равна к1(Т— Тк), где кг — некоторый коэффициент. Тепловая мощность, рассеиваемая из комнаты на улицу, будет к2(ТК — — Ту); здесь к2 — некоторый другой коэффициент. В условиях теплово- го равновесия рассеиваемая батареей мощность равна мощности, рас- сеиваемой из комнаты на улицу. В результате можно написать /с1(Т-Тк1) = /с2(Тк1-Ту1), и аналогично во втором случае ^1 (^ — ^к2) ~ ^2 (^к2 ~’ ^у2)- Поделив одно уравнение на другое, находим (Т— ТК1)/(Т- Тк2) = (Тк1 - Ту1)/(Тк2- Ту2). Отсюда определяем Т: Т = (Тк2Ту1 - ТК1 Ту^/СТ^ + Ту1 - Ту2 - Тк1) = 60°С. 134
2.17. Полное количество теплоты q, выделяемое космическим объектом в единицу времени, пропорционально его объему, т. е. q = = аК3, где а — некоторый коэффициент. Поскольку количество те- плоты, отданное единицей поверхности, пропорционально Т4 и в равновесии вся выделяемая теплота рассеивается в пространство, то можно записать q = £К2Т4 (площадь поверхности пропорциональна R2 и Р — некоторый коэффициент). Приравняв два выражения для q, найдем Г4 = (<*/₽) К. Следовательно, 4-я степень темцературы объекта пропорциональна его радиусу, и, значит, при уменьшении радиуса в два раза температура уменьшится лишь в j/2 « 1,19 раза. 2.18* . Для определенности будем считать, что жидкость, текущая по внутренней трубе 2, уменьшает свою температуру, т. е. ТН2 > ТК2,.а, значит, THi < TKi. Поскольку площадь сечения потока жидкости, теку- щей по внешней трубе 1 (2S — S = S), равна площади сечения потока жидкости, текущей по внутренней трубе 2 (S), и их скорости совпа- дают, то уменьшение температуры жидкости, текущей по трубе 2, по мере удаления от входного конца равно уменьшению температуры жидкости, текущей по трубе 1. Другими словами, разность температур жидкостей остается постоянной по всей длине теплообменника, и, следовательно, ^н2 — ТК1 = Тк2 — Тн1. (1) Постоянство разности температур ведет к тому, что скорость передачи теплоты постоянна вдоль теплообменника. Количество теп- лоты 2, переданное от жидкости, текущей по трубе 2, к жидкости, те- кущей по трубе 1, за время Г, равно С = ^бок^(^н2 — TKi); (2) здесь S6oK — площадь боковой поверхности внутренней трубы, 5бок = = 2лг/, где г — радиус внутренней трубы, т. е. лг2 = S, г = j/s/л. Тепло- та Q идет на нагревание жидкости, текущей по трубе 1: за время t по внешней трубе 1 проходит масса жидкости m = pvtS, а ее температура повышается от THi до TKi; значит, Q — pvtSc(Tyl - Тя1). (3) Приравнивая выражения (2) и (3), получаем 2л J/S/л №(Тн2 - Тк1) = pvSc(TK1 - Гн1). Отсюда можем найти Ткь ас помощью (1) и ТК2- В результате будем иметь Гк1 = Тн2 + (Ти1 - Тн2)[1 + 2]/^Slk/(pvc)]-1, Гй = ТН1 +(Ти2-Тн1)[1 +2l^/Slk/(pvc)]-1, 135
2.19* . Ясно, что после перераспределения давление газа макси- мально у задней (по отношению к направлению движения) стенки со- суда, так как именно сила давления со стороны этой стенки сообщает газу ускорение а. Обозначим его pmax. С другой стороны, ртах рнас. Учитывая, что рнас » р и, следовательно, силой давления со стороны передней стенки можно пренебречь, с помощью 2-го закона Ньютона получим Ртах — MfCl/S Рнас, где тг — масса вещества, находящегося в сосуде в газообразном со- стоянии. Следовательно, при а рНас$/М конденсации происходить не будет, а при а > рНаС$/М масса газа станет равной т = p^S/a и в сосу- де окажется масса жидкости, равная т% = М — т = М — pn3CS/a. 2.20. Кипение воды представляет собой процесс интенсивного образования пузырьков пара. В пузырьках находится насыщенный во- дяной пар, и их образование возможно, если давление насыщенного водяного пара достигает атмосферного давления (760 мм рт. ст., или 105 Па). Как известно, это условие выполняется при температуре, рав- ной температуре кипения воды: Тк = 100°C (или 373 К). По условию задачи давление насыщенного водяного пара на планете равно р0 = = 760 мм рт. ст., а, значит, температура на планете Т = Тк = 373 К. Из уравнения Менделеева — Клапейрона вытекает, что Р = РоМЛТД где ц — молярная масса воды, а р0 — атмосферное давление. Под- ставляя числовые значения, находим р = 0,58 кг/м3. 2.21. Если в холодную погоду мы выдыхаем изо рта воздух, то он резко охлаждается. Как известно, при уменьшении температуры давле- ние насыщенных паров падает. Водяной пар в выдыхаемом воздухе при охлаждении становится насыщенным и превращается в мельчай- шие капельки воды — «пар». Если приоткрыть дверь в теплую избу в морозный день, то прони- кающий с улицы холодный воздух охлаждает пары воды в воздухе из- бы. Это опять-таки приводит к тому, что пар становится насы- щенным, и мы видим «пар» — капельки сконденсировавшейся воды. 2.22* . Задачу проще всего решать графически. Полное давление р в сосуде складывается из давления насыщенных паров воды рнас и давления водорода рн2- Согласно уравнению Менделеева — Клапей- рона, давление водорода равно _ _ mH, RT_ 210’3 кг-8,3 Дж/(моль К) 3 2.10- кг/моль-2.10- м3 Т = 4’1510 Т’ где Рн2 измеряется в паскалях. Зависимость рн2(Т) линейная; поэтому, 136
вычислив рн2(Т) при двух значениях температур, например при 7^=373 К, Рн2 = 15,5-105 Па, Т2 = 453 К, Рн2 = 18,8-105 Па, построим график рн2(Т)- Пользуясь указанием в условии задачи, строим график функции Рнас (Л- «Складывая» графики рн (Т) и рнас(^)> получаем график зави- симости полного давления в сосуде от температуры р(Т) (рис. 201). Из полученного графика р(Т) по заданным в условии задачи начальному и конечному давлениям находим начальную и конечную температуры в сосуде: Рн = 17-105 Па, рк = 26-105 Па, 7\ = Тн * 380 К, Т2 = Тк « 440 К. Найдем теперь массу испарившейся воды. Считая пары воды идеальным газом, определим начальное pHi и конечное рП2 давления паров воды в сосуде. Для этого воспользуемся полученными графика- ми. При 7\ = 380 К давление водорода равно Рн2 ~ 15,5-105 Па и Рп! = Ри - Рн2 * 1,5 • 105 Па. При Т2 = 440 К рн2 « 18 • Ю5 Па и Рп2 = Рк-Рн2 »8-105 Па. Напишем уравнение состояния для паров воды при рп1, и Рп2> ^2: АИП1 тп2 Рп1 ^ = — Pn2V=—RT2f Нп Нп 137
где mnl, mn2 — начальная и конечная массы паров в сосуде. Отсюда на- ходим массу испарившейся воды: . HnV / Рп2 Рп1 \ Дт = тп2-тп1 =-—I —----------— = К \ 1 2 1 1 / 18-10—3 кг/моль-2-10-3 м3 / 8 Па 1,5 Па \ 5 = 8,3 Дж/(К-моль) \440 К ” 380 К / 1 = 6-10"3 кг = 6 г. 2.23. Если h — высота столбика воды в капилляре, то температура капилляра и, следовательно, воды на этой высоте равна Th = T3h/l. Вода в капилляре удерживается силами поверхностного натяжения. Если п/, — поверхностное натяжение при температуре Т/,, то h = 2ch/(p3gr)) где рв — плотность воды. Отсюда находим oh=pgrh/2 = (pgrl/2)(Th/T3). Пользуясь указанием в условии задачи, строим график функции <j(T). Температура Th на уровне максимального поднятия воды опре- деляется точкой пересечения графиков зависимости (pgrl/2)T/T3 и за- висимости ст(Т). Из рис. 202 видно, что Т/,*80оС. Следовательно, h = lTh/T3 *6,4 см. Задачу можно решить и аналитически, если заметить, что зависи- мость п(Т) является практически линейной. 2.24. Условие равновесия мыльной пленки пузыря заключается в том, что давление воздуха в нем pni равно сумме внешнего давления 138
Pi и избыточного давления 4ст/г, создаваемого поверхностным натяже- нием. Обратим внимание, что в мыльном пузыре имеются две гра- ницы раздела воздух — мыльная пленка, каждая из которых создает давление 2ст/г. Именно поэтому полное избыточное давление будет равно 2 • 2ст/г = 4а/г. Итак, сначала можно записать условие равновесия в виде Рп! = Р1 + 4с/г. После того как радиус пузыря уменьшился вдвое, поверхностное натяжение будет создавать давление 8ст/г. По условию задачи темпера- тура поддерживается постоянной, а значит, в соответствии с законом Бойля — Мариотта при уменьшении объема пузыря в 8 раз (радиус уменьшился в 2 раза) давление воздуха в нем будет 8рпь и можно написать 8pni = Pi + ^/г- Подставляя сюда из первого уравнения, окончательно находим р2 = %Р1 + 24ст/г. 2.25. В печи возможны большие перепады температуры. Если кир- пичи и раствор сделаны из разных материалов, т. е. из материалов с разными температурными коэффициентами расширения, то печь растрескается. 2.26. Пусть в результате смешивания жидкостей, начальные тем-, пературы которых были 7\ и Т2, температура смеси в сосуде стала равной Т. Так как сосуд теплоизолирован (AQ = 0), то clm1(T-Tl) + c2m2(T-T2) = 0; отсюда mi/m2 = (с2/с1)(Т — -Г)’1. Согласно условию задачи, 2(7\ — Т) = 7\ — Т2, а, значит, Т — Т2 = = 7\ — Т, и отношение (Т — Т2)/(7\ — Т) = 1. Таким образом, mi/m2 = С2/СЪ т. е. отношение масс жидкостей равно обратному отношению их теплоемкостей. 2.27. В первом случае нагревание воды в пробирке в основном бу- дет происходить за счет конвекции, так как более теплая вода легче, чем холодная. Во втором случае остывание будет происходить только за счет теплообмена между слоями воды в пробирке. Так как условия теплообмена между пробиркой и внешней водой остаются одинаковы- ми, то t i < 12. Заметим, что если поменять параметры задачи: 20°C-*0°C и 80°C-*4°C, то ответ будет обратный. Вода в пробирке будет нагре- ваться дольше, чем остывать. Причина здесь в аномалии воды — в ин- тервале 0 — 4 °C более холодная вода легче, чем более теплая. 139
2.28. Поток теплоты в единицу времени q = AQ/At через поверх- ность контакта исследуемой системы (сосуд — вода, сосуд — вода — шарик) с окружающей средой зависит от разности температур: AQ/At = aF(Tc — Т); здесь t — время, Тс — температура сосуда, Т — температура окружаю- щей среды, F — некоторая функция температуры. Коэффициент а опре- деляется условиями контакта исследуемых систем с окружающей сре- дой. В нашем случае для обоих сосудов условия контакта одинаковы, поэтому коэффициент а для обоих случаев одинаков. Потеря сосудом некоторого количества теплоты AQ приведет к снижению температуры сосуда на АТС. Для сосуда с водой получим AQ1 = (Мвсв 4- тссс) АТС, где Мв и св — масса и удельная теплоемкость воды, тс и сс — аналогичные величины для сосуда. Для сосуда с водой и шариком находим AQ2 = (МВСВ 4- tn^cB + ШщСщ) AFC, где тш и сш — масса и удельная теплоемкость шарика. По условию за- дачи тс «МВ1 nfa = Мв. Кроме того, сс «св. Поэтому можем написать AQ1 = МвсвАТс, А{?2 = Мв(св “Ь ^ш) Нетрудно видеть, что изменение температуры на АТС в обоих сосудах проходит за разные времена Att и At2, причем АТС a АТС a F(TC — T) Маса ° F(TC — T) Ма(са + сш) отсюда получим Ati/At2 = св/ (св + сш). Поэтому и для общих времен охлаждения сосудов tx и t2 будет выпол- нено соотношение t2/h = (св + сш)/св = к; отсюда сш/св = к - 1. 2.29. Если уровень заполнения калориметра после установления теплового равновесия повысился, это означает, что часть воды замерз- ла (при замерзании объем воды увеличивается). Между тем можно утверждать, что не вся вода замерзла — в противном случае ее объем увеличился бы в Рв/рл « 1,1 раза, а уровень заполнения калориметра увеличился бы на (h/3)(l,l — 1) »2,5 см, тогда как по условию А/г = = 0,5 см. Итак, приходим к выводу, что установившаяся температура в калориметре равна 0°С. 140
Используя это условие, запишем уравнение теплового баланса: свтв(Тв - 0°С) = -ХАт + слтл (0°С - TJ, (1) где Тл — начальная температура льда, а Ат — масса замерзшей воды. Как уже отмечалось, при замерзании объем увеличивается в Рв/Рл раз, значит, А^ = (рв/рл-1)Ат/рв, (2) здесь 5 — площадь поперечного сечения калориметра. Подставляя Ат из (2) в (1) и используя соотношения тв = (h/З) р^5, тл = (h/З) рл5, получаем CBS~ РЛ = -XSAh -----------слРл5Т А. 3 Рв - Рл 3 Отсюда к 3 A/i рв св рв Тл=------------------------ -- Тв « -54°C. сл Рв ~ Рл сл Рл 2.30* . 1) Считая воду и лед несжимаемыми, найдем уменьшение температуры смеси в результате увеличения внешнего давления: AT = (pi/p)-l К « 0,18 К. Столь малое изменение температуры указывает лишь небольшая масса льда, т. е. Ат « тл. Запишем закон сохранения энергии: на то, что растает А = k Ат — (сл + св) т&Т. Оценим работу А внешней силы. Изменение объема смеси в ре- зультате таяния льда массы Ат равно Рл Ат Рв — Рл т . = Ат « 0,1 — ~ 10" 4 м5. Рв----------Рв Рл-Рл Мы учли, что плотность воды при замерзании уменьшается примерно на 10%, т. е. (рв — рл)/рв «0,1. В результате имеем оценку 4<р1АК=0,25 кДж. Количество теплоты AQ, необходимое для нагревания массы m льда и массы m воды на температуру АТ, равно AQ = (сл + cB)m АТ « 1,1 кДж. Видно, что 4«AQ; поэтому можно считать, что XAm = AQ, откуда Am = AQA = 3,2 г. 141
Изменение объема за счет таяния этой массы льда равно =Am fe~ Рл »3,5-10~7 м3. Рв Рл Учитывая, что при медленном увеличении давления изменение объема AKj ~ Ар, найдем работу внешней силы: Л = Pi AKi/2 » 0,44 Дж. 2) Примем во внимание сжимаемость льда и воды. Изменение объема льда и воды будет равно ДГ = —10’2ИОВ + —10~2ИОЛ «2-10"6 м3, Р W где Уов = 10“ 3 м3 и Кол — 1,1 • 10“ 3 м3 — начальные объемы воды и льда. Работа А' внешней силы, затраченная на сжатие смеси, равна А' = pi АГ/2 « 2,5 Дж. Полная работа внешней силы равна ^полн А + А ~ 3 Дж. Ясно, что, поскольку опять Аполн « AQ, масса растаявшего льда будет такой же, как в случае 1. 2.31. Учитывая, что плотность газа равна р = М/V, можем запи- сать уравнение состояния водяных паров в виде p = (p/p)RT, где р, ц — плотность и молярная масса водяного пара. Кипение происходит, когда давление насыщенных паров становится равным атмосферному. венно в твердое тело. Из 1-го Если температура кипения подсо- ленной воды повысилась, то при не- изменном атмосферном давлении это означает, что плотность насы- щенных водяных паров должна уменьшиться. 2.32. Рассмотрим цикл, окру- жающий тройную точку: -► С А (рис. 203); при этом по оче- реди происходят следующие прев- ращения фаз: плавление испаре- ние -► превращение газа непосредст- начала термодинамики при условии, что цикл неограниченно сжимается в тройную точку, получим для массы т вещества mk + mq — mv = 0, так как работа системы за цикл равна нулю, притока тепла извне нет, 142
а, значит, полное изменение внутренней энергии также равно нулю (правая часть уравнения). Отсюда находим удельную теплоту сублима- ции воды в тройной точке: v = X -ь q = 2,82 • 106 Дж/кг. 2.33. Концентрация налитого на горизонтальную поверхность рас- твора сахара практически не меняется. После установления равновесия концентрация раствора в сосуде будет равна с = ct hjh (изменение концентрации происходит за счет испарения из раствора молекул воды — увеличение концентрации, либо за счет конденсации молекул пара в сосуд — уменьшение концентрации). Давление насы- щенного пара над раствором в цилиндрическом сосуде меньше, чем над раствором на дне, на Ар = 0,05рнас(с — с2\ Эта разность давлений уравновешивается давлением столба пара высоты h: Pnff/i = О,О5рнас(с — с2). Отсюда находим h = О,О5рнас(с - с2)/(Рпй). Плотность рп пара при температуре Т = 293 К найдем из уравне- ния газового состояния: Рп = — Рнас М-/ ТУ Таким образом, высота столба пара h, удовлетворяет квадратному уравнению h = (0,05с2КТ/^)(2Ь1/Л - 1). Подставляя приведенные в условии числовые данные и решая квадрат- ное уравнение, находим h «16,4 см. Интересно отметить, что, как следует из рассмотренной задачи, ес- ли под колпаком будут находиться два одинаковых сосуда с раствора- ми разных концентраций, то испарение будет происходить из раствора с меньшей концентрацией. И наоборот, водяной пар будет концентри- роваться в раствор с большей концентрацией. Таким образом, концен- трации будут стремиться выравняться. Именно с этим физическим явлением связано намокание сахара, соли и т. п. в атмосфере влажного воздуха. 2.34. Поскольку нижний конец трубы поддерживается при темпе- ратуре Т1У большей температуры плавления,’ чугун внизу будет распла- влен. Температура границы между расплавленным и твердым чугуном, естественно, равна температуре плавления Тпл. 143
Так как температуры верхнего и нижнего концов трубы поддержи- ваются постоянными, количество теплоты, проходящей за единицу времени через поперечное сечение трубы, в любом месте трубы дол- жно быть одним и тем же. Другими словами, поток теплоты через рас- плавленный и твердый чугун должен быть одним и тем же (кирпичная труба плохо проводит тепло и теплообменом через ее стенки можно пренебречь). Поток теплоты пропорционален теплопроводности, площади по- перечного сечения и разности температур, приходящейся на единицу длины. Пусть /t — длина нижней части трубы, где чугун расплавлен, а 12 — длина верхней части, где чугун находится в* твердой фазе. Условие постоянства потока теплоты запишется тогда в виде (сечение трубы постоянно) Ш ~ ^iui)/G = ^тв(^пл ^2)/^2’ где Хж и Хтв — теплопроводность жидкого и твердого чугуна. Учиты- вая, что %ж = /сХтв, находим Полная длина трубы равна 1Г + /2. Значит, расплавленным металлом занята часть трубы, определяемая отношением к(7\ - Тил) Zi +12 ~ Гпл) + (Тдл — т2) 2.35* . Полное количество теплоты Q, излучаемой в пространство в единицу времени, останется прежним, так как оно определяется энер- говыделением при работе аппаратуры станции. Поскольку в простран- ство излучает лишь внешняя поверхность экрана и это излучение опре- деляется только его температурой, то температура экрана должна быть равна начальной температуре станции Т = 500 К. Экран, однако, излучает точно такое же количество теплоты Q и вовнутрь. Это излу- чение попадает на оболочку станции и поглощается ею. Таким обра- зом, полный подвод теплоты в единицу времени к станции склады- вается из выделяемой при работе приборов теплоты Q и поглощаемой от внутренней поверхности экрана теплоты £), т. е. равен 2Q. Из усло- вия теплового баланса оно же должно и излучаться; значит, Q/(2Q)= T*/Tt, где Тх — искомая температура оболочки станции. Окончательно нахо- дим Тх = 600 К. 2.36. Как видно из графика, первые 50 минут температура смеси не менялась и оставалась равной 0°С. Все это время теплота, получае- мая смесью из комнаты, шла на таяние льда. Через 50 минут весь лед 144
растаял и температура воды начала повышаться. За 10 минут (от -q = = 50 до т2 = 60 мин) температура повысилась на АТ = 2 °C. Теплота, поступившая к воде из комнаты за это время, равна q — АТ = = 84 кДж. Значит, за первые 50 минут к смеси из комнаты поступило количество теплоты Q = 5q = 420 кДж. Эта теплота и пошла на таяние массы тл льда: Q = Хтл. Таким образом, масса льда в ведре, внесен- ном в комнату, равна гил = 2А ~ 1,2 кг. 2.37* . Обозначим коэффициент пропорциональности между рас- сеиваемой на резисторе мощностью и разностью температур резисто- ра и окружающего воздуха через а. Тогда, поскольку при Т3 = 80°С сопротивление резистора равно а напряжение на нем С\, то рассеи- ваемая мощность равна Uj/Rlf и мы можем записать, что Ul/Rt = <z(T3 — То). ~ (1) При повышении приложенного напряжения температура резисто- ра растет, так как увеличивается выделяемая при прохождении тока теплота. Когда температура достигнет величины 7\ — 100 °C, сопро- тивление резистора скачком увеличится в два раза. Выделяемая на нем теплота уменьшится, и, если напряжение не очень велико, отвод теп- лоты окажется более быстрым, чем ее выделение. Это приведет к паде- нию температуры до Т2 = 99 °C, но тогда сопротивление скачком уменьшится до прежней величины и процесс начнет повторяться. Та- ким образом в цепи возникнут колебания тока, связанные со скачко- образной зависимостью сопротивления резистора от температуры. Температура резистора во время этих колебаний почти постоянна (она изменяется в пределах от Т2 = 99 °C до 7\ = 100 °C), так что мож- но считать теплоотвод постоянным, и отводимая мощность составляет а(7\ — То). Тогда, вводя — время нагрева (от 99 до 100 °C), t2 — время охлаждения и 4-12 = Т — период колебаний, можно напи- сать уравнения теплового баланса: UjtJR, = а(Т, - То) t, + С(Т2- Т2), (2) l/ft2/K2 = а(Т, - Т0)С2 - C(Tt - Т2). Используя найденное из (1) значение ос, находим f =____________С(Т, - Т2)____________ 1 u2/r2 - U2 (Т2 - То)/[К2 (Тэ - То)] ’ f =____________С(Л - т2)_____________ 2 и2 (Т2 - То)/[«1 (Гз - То)] - U2/R2 • Подставляя числовые значения величин, получаем, что tr = t2 = = 3/32 с 0,1 с, а Т » 0,2 с. 145
Максимальное и минимальное значения тока легко находим, так как в процессе колебаний сопротивление скачком изменяется от Rr = = 50 до R2 = 100 Ом; следовательно, /max = U2/R! = 1,6 A, /min = U2/R2 = 0,8 А. Хочется обратить внимание на тот факт, что описываемая в зада- че ситуация отвечает фазовому переходу 1-го рода в материале, из #о- торого сделан резистор. При нагревании при температуре 7\ = 100 °C происходит переход материала в новую фазу (этот переход, например, может быть связан с перестройкой кристаллической решетки материа- ла резистора). Обратный переход происходит при более низкой темпе- ратуре Т2 = 99 °C — это явление так называемого гистерезиса, свой- ственного фазовым переходам 1-го рода. 2.38. Капли дождя, попав на кирпич, в первый момент образуют пленку на его поверхности (рис. 204). Кирпич имеет пористую структу- ру. Поры представляют собой капилляры. За счет сил поверхностного Рис. 204 натяжения вода втягивается в поры-капилляры. Капилляры соединены между собой. Их размеры различны: узких капилляров больше, чем широких. В широких капиллярах втягивающая сила поверхностного натяжения меньше, чем в узких. Поэтому в широких капиллярах водя- ная пленка должна вздуться и лопнуть. Это и является причиной шипения. 2.39. Совершаемая газом работа А складывается из двух частей: работы против силы атмосферного давления и работы А2 против силы тяжести. Граница раздела газ — ртуть до полного вытеснения ртути перемещается на 214-1/2 = (5/2) /, и, значит, А1=(5/2)р^1. Работа А2 против силы тяжести равна изменению потенциальной энергии ртути при ее вытеснении. Вся ртуть в результате вытеснения поднимается на высоту I относительно горизонтального участка; это и надо считать конечной высотой центра масс ртути. Начальное поло- жение центра масс ртути, как нетрудно видеть, равно h0 = //8. Отсюда можно заключить, что Л2 = Л1^(/—//8) = (7/8)М^/, 146
где М = 2/SppT — масса ртути. Окончательно находим 5 7 А = Ai + А2 = —p0Sl + у PprgSl2 « 7,7 Дж. 2.40. Работа 4, которую совершила внешняя сила в результате на- ложения и последующего снятия нагрузки, определяется площадью фигуры ABCD (см. рис. 69). В соответствии с 1-м законом термодина- мики изменение внутренней энергии стержня (стержень теплоизолиро- ван) и будет равно этой работе, т. е. АЖ = А = кх0 (х — х0). С другой стороны, АЖ = САГ, где АТ — изменение температуры стержня, и в результате находим АТ = АЖ/С = кх0(х - х0)/С. 2.41. Пусть цилиндр заполнился водой до уровня х от нижнего ос- нования. Изменение выталкивающей силы равно увеличению силы тя- жести, действующей на цилиндр с водой. Это позволяет сделать вы- вод, что Ah = х. Из условия равновесия цилиндра можем написать р2$ = PQS 4- тд, где р2 — давление газа в цилиндре после того, как в него вошла вода, т. е. р2 = р0 + пгд/S- Используя закон Бойля — Мариотта, напишем, что p2(h — A/i) = рр где Pi — начальное давление газа. Окончательно нахо- дим Pi = (Ро + - Лй/й). 2.42. Выберем начало отсчета вдоль оси х так, как показано на рис. 205. Тогда сила, действующая на клин, зависит лишь от коорди- наты х фронта ударной волны. Го- ризонтальная составляющая этой силы равна Fx = Росх tg а = Pocxa/b = p^caut/b, где х = vt — координата фронта волны к моменту времени t от начала прохождения волны через клин. Ускорение, сообщаемое клину в данный момент времени, равно at = Fx/m = pQcavt/ (bm). В тот момент времени t0, когда фронт волны дойдет до дальней грани клина, т. е. координата фронта будет равна b = vt0, ускорение 147
клина составит «(„ = Роса/т. Поскольку ускорение клина линейно зависит от времени, для вычисле- ния скорости и клина к моменту времени г0 воспользуемся средним значением ускорения = роса/(2т): w = Ocptg = poabc/(2mv). Когда весь клин окажется в области повышенного давления, результи- рующая сила, действующая на клин, будет равна нулю. Как видно из ответа задачи, условие u«v означает, что р0 <<2mv2/(abc).
3. ЭЛЕКТРИЧЕСТВО И МАГНЕТИЗМ
3.1. Из-за поляризации диэлектрического стержня АВ на точечный заряд + qr помимо точечного заряда — q2 будут действовать поляри- А________________В © I- ?| 9/ •Рис. 206 зационные заряды, образовавшиеся на концах стержня (рис. 206). Сила при- © тяжения со стороны отрицательного ^2 заряда, возникшего на конце 4, бу- дет больше силы отталкивания поло- жительного заряда, возникшего на конце В. Таким образом, общая сила, действующая на заряд +q^ увеличится. 3.2. В непосредственной близости от каждого из точечных зарядов вклад в общую напряженность поля от другого заряда пренебрежимо мал, поэтому линии напряженности выходят (входят) равномерным пространственным пучком, общее их число пропорционально модулю заряда. Внутрь конуса с углом 2а при вершине вблизи заряда + qr по- падает только часть линий. Отношение их числа к общему числу выхо- дящих из заряда + qr линий напряженности равно отношению площа- дей соответствующих сферических сегментов: 2nRR (1 — cos а) 1 ---------=----— —(1 — cos а). 4лК2 2 v 7 Поскольку линии напряженности связывают между собой равные по модулю заряды, то число линий, выходящих из заряда + qx внутри угла 2а, равно числу линий, входящих в заряд — q2 под-углом 2р. Следовательно, l«i1(1 - cosa) = |921(1 - cos Р); отсюда sin(P/2) = sin(a/2)]/|<J11/|^ I- Если ]/| | /1 1 sin (ос/2) > 1, то линия напряженности не войдет в заряд -q2. 3.3. Прежде чем перейти к решению задачи 3.3, сформулируем теорему, полезную для решения и более сложных задач. Ниже будет дано доказательство этой теоремы применительное к конкретному случаю, рассмотренному в задаче 3.3. Если заряд с постоянной плотностью о распределен по участку сферической поверхности радиуса R, то проекция создаваемой этим за- рядом напряженности электрического поля в центре сферической по- верхности на произвольное направление а будет равна 1 о =--------г- S_Le, 4ле0 R2 где 5j_e — площадь проекции этого участка поверхности на плоскость, перпендикулярную направлению а. 150
Рассмотрим некоторый участок сферической поверхности — «доль- ку» и сориентируем ее так, как указано на рис. 207 — плоскость сим- метрии «дольки» совместим с осями Oz и Ох. Из симметрии располо- жения заряда очевидно, что общая напряженность в центре системы координат (точка О) будет направлена против оси Oz (если а > 0), а со- ставляющие напряженности по осям Ох и Оу будут равны нулю. Рис. 207 Рассмотрим небольшой участок поверхности «дольки» AS. Верти- кальную составляющую напряженности в точке О, создаваемую участ- ком AS, определим по формуле 1 а АЕ =---------— AS cos (p, 4тсе0 R2 где ф — угол между нормалью к площадке и вертикалью. Но AS cos ф — это площадь проекции участка AS на горизонтальную плоскость. От- сюда полная напряженность в точке О находится по формуле 1 aS £' =_________ 4тсе0 R2 ’ где S - площадь заштрихованной фигуры на рис. 207 - проекции «дольки» на горизонтальную плоскость хОу. Так как площадь лю- бой узкой полоски этой области (черного участка на рис. 207) в sin (а/2) раз меньше площади соответствующей полоски большого круга, то и вся площадь заштрихованной области в sin (а/2) раз будет меньше площади большого круга. Отсюда aS о £' —_______—________ 4я£0К2 4я£0Е2 _ а а sin (а/2) tiR sin — =-------------- 2 4е0 151
В случае полусферы а = п и £ = а/(480). 3.4. Из соображений симметрии легко понять, что вектор напря- женности электрического поля «дольки», угол раствора которой равен а, лежит в плоскостях продольной и поперечной симметрии «дольки». Пусть модуль этого вектора равен Е. Воспользуемся принципом супер- позиции и дополним «дольку» до полусферы, заряженной с той же по- верхностной плотностью. Тогда приложим к исходной «дольке» еще одну с углом раствора п — а. Пусть модуль вектора напряженности по- ля, создаваемого добавочной «долькой» в центре сферы, равен Е'. Лег- ко понять, что векторы Е и Е взаимно перпендикулярны, а их вектор- ная сумма представляет собой вектор напряженности поля полусферы в ее центре. По условию задачи эта сумма равна Ео. Поскольку угол между векторами Е и Ео равен л/2 — а/2, то Е = Ео sin (а/2). 3.5* . Рассмотрим случай, когда конденсаторы ориентированы друг на друга одноименно заряженными пластинами (рис. 208). Поле, создаваемое первым конденсатором на оси на расстоянии х от поло- жительно заряженной пластины, равно £(Х)=Л1П—1_ 4л80 L * (х + О2 2д,1 4тС80Х3 Сила, действующая на второй конденсатор, находящийся на расстоя- нии d от первого, равна F = q2 [£(</)-E(d + 0] = ^[4 " Тгкт 4л80 L “ (d + О 2tc80J4 Значит, в этом случае конденсаторы будут отталкиваться. Аналогичное рассмотрение можно провести и для случая, когда конденсаторы ориентированы друг на друга разноименно заряженны- - _ми пластинами. При этом конденса- • ^^7 торы будут притягиваться с той же силой + £ Рис. 208 г _ з д2д212 2 nzod* ’ 3.6. Выберем два небольших произвольных участка площадью A5j и AS2, принадлежащих первой и второй полусферам. Пусть рас- стояние между этими участками равно г12. Силу взаимодействия этих участков АГ12 определим из закона Кулона: . _ 11 л л AS, AS, AF12 — — 2 ^1 AS1CF2 AS2 — —- 2 ^1^2’ 4tC80 rf2 47C80ri2 152
При нахождении общей силы взаимодействия полусфер мы дол- жны, исходя из принципа суперпозиции, просуммировать силы AF12 для всех взаимодействующих пар участков, так что результирующая сила взаимодействия полусфер F примет вид F = /сстхст2, где коэффициент к определяется лишь геометрией расположения заря- дов и выбором системы единиц. Если бы полусферы были заряжены с одинаковой поверхностной плотностью заряда, ст, соответствующая сила взаимодействия полусфер F была бы равна F - ко2, где коэффициент к тот же, что и в предыдущей формуле. Найдем силу F. Для этого рассмотрим «верхнюю» полусферу. Не- большой ее участок площадью AS несет заряд Ад = ст AS и находится под действием электрического поля, напряженность которого Е' соста- вляет половину напряженности сферы радиуса R, равномерно заряжен- ной по поверхности с плотностью заряда ст. (Мы должны исключить из напряженности ту ее часть, которая создается самим зарядом Ад.) На заряд Ад действует сила 1 4лК2 ст2 AF — AqE' - ст AS----ст —-— =-----AS, 4Л£О R2 -2 2е0 которая направлена перпендикулярно поверхности участка. Чтобы найти силу, действующую на верхнюю полусферу, заметим, что из вы- ражения для силы AF следует, что полусфера как бы находится под действием эффективного давления р = ст2/(2е0). Отсюда общая сила F, действующая на верхнюю полусферу, равна F - pitR2 = nR2 2е0 (хотя в выражение AF для силы, действующей на участок AS, дает вклад не только «нижняя» полусфера, но и все остальные участки «верхней» полусферы, в общем выражении для силы взаимодействия полусфер, найденном выше, силы взаимодействия участков верхней по- лусферы между собой взаимно сократятся.) Поскольку F = /ест2, получим следующее выражение для силы взаи- модействия полусфер в случае их разной поверхностной плотности зарядов: itR2 F = ^^2 = ——ст1ст2. Z£o 3.7. Плотность зарядов, наведенных на сфере, пропорциональ- на напряженности электрического поля: ст ~ Е. Сила, действующая 153
на половинки сферы, пропорциональна напряженности: F ~ aSE ~ R2E2, где 5 — площадь половинки сферы, R — ее радиус. При изменении ра- диуса сферы в п раз, а напряженности поля в к раз, сила F изменится в к2п2 раз. Поскольку толщина стенок сферы остается постоянной, сила, разрывающая сферу и приходящаяся на единицу длины, должна быть прежней, т. е. к2п2/п=1, к = 1/]/й = 1/J/2. Следовательно, минималь- ная напряженность поля, которое разорвет проводящую оболочку вдвое большего радиуса, равна = £0/1/2- 3.8. Пусть расстояние от большого шара до каждого небольшого шарика равно /, расстояние между шариками J, радиус каждого шари- ка г. При соединении большого шара с первым шариком их потен- циалы сравниваются: ~— (~т “*—)= 4л£0 \ I г ) (1) где Q — заряд большого шара, <р — его потенциал. При соединении большого шара со вторым шариком получим аналогичное уравнение, отвечающее равенству потенциалов большого шара и второго шарика: 4л£0 \ I d г ) (2) (Мы предположили, что при каждом заряде шариков заряд и потен- циал большого шара меняются незначительно.) При соединении боль- шого шара с третьим шариком, когда первый и второй заряжены, со- ответствующее уравнение равенства потенциалов примет вид 1 4яе0 б.,А+ + I d d r) <p. (3) Заряд q3 найдем, решая уравнения (1) —(3): Чз = ч1/Ч1- 3.9. Заряд qr шара найдем по формуле 41 «ДяеоФ^р После соединения шара и оболочки весь заряд qY перетечет с шара на оболочку и распределится равномерно по поверхности оболочки. Ее потенциал <р2 (совпадающий с новым значением потенциала шара) бу- дет равен 4>2=~A-----= Ф1—• 4л£0г2 г2 154
3.10. Запишем условие равенства потенциала сферы, а значит, и любой точки пространства внутри нее (в частности, центра) нулю к моменту времени t, причем выделим три интервала времени: 1) t<a/v, 2) a/v <b/v, 3) t^b/v. Обозначим заряд сферы q(t). Для момента времени t внутри первого интервала 1 получим qi/a + q2/b + q(t)/(vt) = 0;_ отсюда q W = - v (qja + q2/b) t, Ii (t) = -v (qja + q2/b). Для момента времени t внутри интервала 2 получим, что поля внутри сферы и вне нее независимы; отсюда q(t) + q2 q2 , q2 ----------------= 7~,-----I2(t) = -v—. vt------------------------b b Наконец, как только сфера поглотит внутри себя оба точечных заряда qY и q2, протекание тока по «заземляющему» ее проводнику прекратит- ся, и /3(О = о. Итак, ’ ~v(qi/a + q2/b), = < vq2/b, . 0, t < a/v, a/v^t < b/v, t b/v. 3.11. Учитывая связь между емкостью конденсатора, напряжением и зарядом на нем, можно записать уравнения для трех конденсаторов: Фл — Фо = Фв — Фо = 41/^2» Фд“Фо = 4з/^ где С2 и С3 — емкости соответствующих конденсаторов, a qY, q2 и q3 — заряды на их обкладках. Согласно закону сохранения заряда, 41+42 + 4з = 0; поэтому потенциал общей точки О равен Фо — (Фл^1 + Фв^2 + Фд^з)/(^1 + ^2 + Сз)- 3.12* . Так как листок металлический, заряды должны перераспре- делиться по его поверхности так, чтобы поле внутри листка отсутство- вало. В первом приближении можно считать это распределение равно- мерным как по верхней, так и по нижней поверхности листка с плотностью —пип соответственно. Из принципа суперпозиции по- лучим условие отсутствия поля внутри листка: <?/(4ле012) - а/е0 = 0. 155
Учтем теперь неоднородность поля точечного заряда, так как именно вследствие нее возникнет сила взаимодействия F. Верхняя по- верхность листка должна притягиваться с силой ст5^/(4ле0/2), а ниж- няя — отталкиваться с силой ст5д/[4ле0(/ 4- J)2]- Следовательно, сила притяжения F листка к заряду равна &Sq Г 1 q2Sd 4л80/2 |_ (14- d/l)2 _ 8л2е0/5 3.13. Нетрудно понять, что схема, предложенная в задаче, пред- ставляет собой «правильный» тетраэдр, в ребра которого «включены» шесть одинаковых конденсаторов. Поэтому* из соображений симмет- рии ясно, что, к какой бы паре точек мы ни подключили источник, всегда най- дется конденсатор, который не будет заряжен (конденсатор ребра, скрещен- ного с ребром подключения источни- ка). Например, на рис. 209 при под- ключении источника к точкам А и В конденсатор, соединяющий точки С и D, не будет заряжен, поскольку потен- циалы точек С и D равны. 3.14. Емкость нелинейного кон- денсатора равна С = 8С0 = а 17 Со, где Со — емкость конденсатора без диэлектрика. На нелинейном конденсаторе заряд будет равен qH = = CU — aC0U2, а на обычном соответственно q0 = C0U. Из закона сохранения заряда Qu + Qo — получаем искомое напряжение V = (|/4аи0 + 1 - 1)/(2а) = 12 В. 3.15. Перейдем в инерциальную систему отсчета, связанную с дви- жущимся центром нити. Тогда в начальный момент времени шарики имеют одинаковую скорость v. Первоначальный запас энергии в систе- ме равен Wi = q2/(4mQ^2l) + 2mv2/2. В момент наибольшего сближения энергия системы равна ^2 = ^2/(4Я804 Из закона сохранения энергии получим ответ: d = 2lq2/(q2 4- 8itsomv2l). 156
3.16. Пусть vr и v2 будут соответственно скорости 1-го и 2-го ша- риков после выключения однородного электрического поля. По усло- вию задачи угол между скоростью и начальной скоростью и равен 60°. Поэтому изменение импульса Арх 1-го шарика равно Api = q^EAt = v sin 60°. Здесь мы использовали условие = г/2, из которого следует, что на- правление изменения импульса 1-го шарика Арх перпендикулярно на- правлению его скорости Так как Е || Арх и направление изменения импульса 2-го шарика параллельно направлению Арх, то для скорости 2-го шарика получим (легко догадаться, кстати, что знаки зарядов шариков совпадают) v2 — vtg30° — г/]/з. Соответствующее изменение импульса 2-го шарика равно Ар2 = q2E At = m2v/cos 30°. Отсюда получим, что qt m1sin60° q2 4 q{ 4^ q2 m2/cos30°’ m2 3 т/ 3 1 3.17. Кинетическую энергию 1-го Освобожденного шарика на бесконечности (спустя большой промежуток времени) найдем из зако- на сохранения энергии: * А I ’ ’ • • • ’ I 9 2 47C6q \ Л2 Clfj_। J где аъ а2, ..., a^-i — расстояния от 1-го шарика до того момента, как он был освобожден, до остальных шариков по кругу, аг и ajv-i — расстояния до ближайших соседей, т. е. a1=a^_i=a (N = = 1977). При движении 2-го шарика влиянием 1-го освобожденного шарика пренебрегаем; тогда mv2 q2 ( 1 1 1 \ —-— = —----(----1-----F ... 4- --), 2 4ле0 а2 aN_2/ т. е. в скобках не хватает одного ближайшего соседа. Таким образом, 9 9 9 к = mu2 _ ти2 = д 2 2 4л£оа * или q = у^леоКа. 3.18. Согласно закону сохранения импульса, ту = (т + М) и, 157
где т — масса ускоренной частицы, М — масса атома, а и — их ско- рость сразу после соударения. Обозначив через энергию ионизации, закон сохранения энер- гии можно записать в виде mv2/2 = + (m + М) и2 /2. Исключив из этих уравнений скорость и, получим mv2 /2 = WH (1 + т/М). Если т — масса электрона, то т/М « 1 и необходимая для ионизации кинетическая энергия равна mv2/2 « №И. В случае столкновения атома с ионом, масса которого т «М, mv2/2 «2^, т. е. энергия иона, необходимая для ионизации, должна быть в два раза больше, чем энергия электрона. 3.19. Из равенства электрической и упругой сил, действующих на незакрепленный шарик, <?2/(4тсе0 • 4Z2) = /с/, получается следующее соотношение для длины нерастянутой пру- жины /: = q2/(16neofc), где к — жесткость пружины, q — заряды шариков. Пусть незакрепленный шарик отклонили на малое по сравнению с I расстояние х относительно положения равновесия. Потенциальная энергия системы следующим • образом зависит от х: W = lfc(M2 + Я2 4л80 (21 — х) « — kl2 + кх2. 2 Здесь мы учли соотношение между qu к и /, полученное ранее, и в выражении 1 1 1 f (х \ f х \2 ] 21 —х = 2/(1—х/2/) [1 + \2Г/+\2Г/ + " J оставили слагаемые с точностью до [х/(2/)]2. Таким образом, рас- тянутая пружина обладает как бы вдвое большей жесткостью, и отно- шение частот гармонических колебаний системы равно v2/Vl=|/2fc/|/fc = |/2. 3.20. Рассмотрим два заряженных шарика как единую меха- ническую. систему. Кулоновское взаимодействие между шариками является внутренним, поэтому оно не влияет на движение центра масс. Единственная внешняя сила, действующая на систему, — сила тяжести, 158
только она и будет определять движение центра масс системы. Так как массы шариков равны, то начальное положение центра масс находится на высоте (hx + h2)/2 и его начальная скорость v направлена горизон- тально. В дальнейшем центр масс будет двигаться по параболе, харак- теризуемой уравнением h = (hi+h2)/2-to/2)(x/v)2, (1) где х — координата центра масс по горизонтали, h — по вертикали. Когда 1-й шарик упал на землю на расстоянии х = /, высота Н центра масс системы, как это следует из выражения (1), была равна H = (hl+h2)/2-(g/2)(l/v)2. Поскольку массы шариков одинаковы, то 2-й шарик должен находить- ся в этот момент на высоте Н2 = 2Н. Таким образом, H2 = h1+h2-g(l/v)2. 3.21. Пусть сопротивление половины витка равно R. Тогда в пер- вом случае мы имеем параллельное соединение пятнадцати сопроти- влений величины R каждое; общее сопротивление равно R/15. Во втором случае мы получим последовательное соединение этих же пятнадцати сопротивлений; общее сопротивление будет 15К. Таким образом, при разматывании мотка сопротивление проволоки чится в 225 раз. 3.22. Из соображений симметрии нетрудно заметить, что циалы точек А и С (рис. 210) в любой момент времени будут ковыми, поэтому замыкание ключа К не приведет к изменению ра- боты схемы, а сама спираль АС не будет накалена. 3.23. После добавления двух проводников цепь примет вид, изображенный на рис. 211. В силу увели- потен- одина- Рис. 211 симметрии образовавшейся схемы центральный проводник не будет участвовать в процессах переноса электрического заряда. Поэтому, если исходное сопротивление цепи было равно Rt = 5г, где г — сопротивление одного проводника, то после изменения схемы новое ее сопротивление R2 станет равным R2 = 2г 4- 2г/2 = Зг. 159
Таким образом, Л2/Л1=3/5. 3.24. Общее сопротивление RAb каркаса легко вычисляется, учиты- вая, что по ребру CD из соображений симметрии ток не течет: RAB — = R/2, где R — сопротивление каж- дого из ребер, поэтому I = 2U/R, где U — поданное напряжение. Изменить общий ток можно двумя способами: 1) убрать одно из ребер: AD, АС, ВС или BD — изменение тока при этом будет оди- наковым; 2) убрать ребро АВ. В случае 1 изменение тока будет равно AI = - (2/5) U/R = -//5, в случае 2 общее сопротивление рав- но R, поэтому = —U/R = = -//2 = 4/^. 3.25. Из соображений симметрии потенциалы точек С и D равны, поэтому данную схему можно заменить на эквивалентную (мы объеди- ним узлы Си/), рис, 212). Сопротивление между точками А и В схемы найдем по формулам сопротивления для параллельного и последова- тельного соединений проводников. Отсюда R/2(R/2 + R) 3 R 3 Rc(D)b- R/2 + r/2 + R ~ Т Т “ Т*’ R 3 7 Rac (D) в = у + у R = -т- К, Z Q О R(7/8)R 7 АВ R+(1/8)R 15 R’ Таким образом, ток I в подводящих проводах найдем по формуле U _ 15 U (7/15)2? ~~~R' 3.26* . Для решения задачи представим схему цепи в более сим- метричном виде (рис. 213). Полученную схему не удается упростить пу- тем разъединения или соединения узлов (или путем удаления каких-ни- будь проводников), чтобы упрощенная схема представляла собой лишь параллельные и последовательные соединения проводников. Однако, поскольку любая задача на постоянный ток имеет единственное реше- ние, попытаемся «угадать», используя симметрию схемы, сходство в силах токов, протекающих по цепи. 160
Подадим на нашу цепь напряжение U. Проставим во всех участ- ках цепи токи. В данном случае нам понадобится не девять величин (как было бы при произвольных значениях сопротивлений участков це- пи), а лишь пять величин: Ц, I2, Z3, Л, Л (рис. 214). При таких токах первое правило Кирхгофа, записанное для узла С: Ц = 13 + /5, и узла Z>: h + ^3 = ^4 + Л» автоматически выполняется для узлов Е и F (именно в этом проявило себя равенство сопротивлений всех резисторов схемы). Запишем теперь второе правило Кирхгофа, чтобы получить систему пяти независимых уравнений: (12 + Ц + IJR = и, (I3 + I4)R = ISR, (Л +/3)К = М, где R — сопротивление каждого резистора. Решая систему пяти уравне- ний, выразим все токи в цепи через ток I,: 6 1, 3 4 Л=уЛ, *э = у-, /« = у Л, /5 = у Л- Кроме того, и = (Ц +у/1+уЛ)Я. Следовательно, U/Ц = 3R. Учитывая, что сопротивление цепи RAB удовлетворяет уравнению Rab — U/(Ji + 12), находим, что R - U - U 5 U АВ~ Л + /2 ” Л+(б/5)Л "1Т7/ Отсюда с учетом полученного выше соотношения искомое сопротивле- ние равно Rab = (15/11) R. 6 А. И. Буздин и др. 161
3.27. Из соображений симметрии исходная схема может быть за- менена на эквивалентную (рис. 215); при этом мы заменим «внутрен- ний» треугольник из бесконечного числа звеньев резистором сопро- тивления RAB/2, причем сопротивление RAB таково, что RAB = Rx, RAB = = ар. После упрощения схема представляет собой систему последова- тельных и параллельных соединений проводников. Для нахождения величины Rx получим уравнение RRx/2 V1 R + Rx/2/ / RRX 2 \( RX = R[R +---- R + R + \ R + Rx/2/\ Решая его, находим Rab = Rx = R(]/7 - l)/3 = ap(j/7 - l)/3. 3.28. Из соображений симметрии, если из схемы убрать первое звено, то сопротивление оставшейся цепи между точками С и D будет равно Rcd = kRAB. Поэтому эк- вивалентная схема бесконечной Рис. 216 цепочки примет вид, изображенный на рис. 216. Отсюда, применяя формулы для расчета сопротивлений последовательного и параллель- ного соединения резисторов, получим Rab — Ri+ R2kRAB/(R2 + kRAB). Решая квадратное уравнение относительно величины RAB, найдем (в частности, при условии к = 1/2) Rab = («1 - R2 + VRl + Rl + M^RJ/l. 3.29. Реостат вместе с нагрузкой эквивалентен резистору сопро- тивлением R RR/2 5 К. = — +-------— = — R. 1 2 R + R/2 6 Отсюда общий ток в цепи равен 17 6 17 11 =--------=-------. 1 (5/6) К 5 R 162
Напряжение на нагрузке СЛнагр будет равно R 2 Ui^^U-Ц — ^-и. Если сопротивление нагрузки станет равным 2R, общий ток 12 будет равен U _ ю и 11 R (R/2)(2R) ~ V ~R Т + R/2 + 2R Напряжение на нагрузке станет равным С/гнагр- R 4 u2mrp=u-i2— = -u. Таким образом, напряжение на нагрузке изменится в к — ^2нагр/^1нагр раз: ^2нагр/^1нагр = Ю/9. 3.30. В первом случае выполняется условие Ц —12, причем Ц — = о^Ир 12 = а2и2, следовательно, = а2и2. Во втором случае I[Rt — = I2Rx, где Rx — сопротивление второго резистора. Кроме того, Ц = = о^и/, 12 = а2и2, поэтому ^1а1и1 = ^ха2и2- Окончательно получим R1n;/n1=Rxni/n2. Таким образом, = Rt п2п;/(п^). 3.31. Условием того, что проволока нагревается, а затем плавится, является превышение выделившегося джоулева тепла над количеством теплоты, уходящей в окружающую среду: I2R>kS(T-Tcp). По условию задачи для того, чтобы первая проволока расплави- лась, необходимо пропустить через нее ток, больший чем 10 А. Поэтому fc.4d1/(TnjI-Tcp) = /2R1, где I — длина проволок, Тпл — температура плавления материала про- волок, R i — ток и сопротивление первой проволоки. Сопротивление второй проволоки равно R2 = Ri/16. Таким обра- зом, чтобы вторая проволока расплавилась, необходимо через нее 6* 163
пропустить ток 12, удовлетворяющий соотношению ^R2>fc-4</2/(TnJI-Tcp). Окончательно получим 12 > 8Л = 80 А. 3.32. Пусть э. д. с. второго источника равна ^2. Тогда по условию задачи 7 = (^i + ^)/(K + Kx)=^2/R, где R — сопротивление переменного резистора при одинаковом токе. Отсюда получим ответ: ях = М- 3.33. Сила тока 119 протекающего через контур до замыкания ис- точника тока с э. д. с. &2, удовлетворяет условию А=(^ + ^)/(к + п + г2), где и г2 — внутренние сопротивления источников тока. После замы- кания накоротко источника тока с э. д. с. %2 силу тока 12 через рези- стор сопротивлением R найдем по формуле /2=М« + Г1). Очевидно, что если %i/(K + ri) > (^2 + %i)/(K + ri + гг)> то ответ на вопрос задачи будет утвердительным. Итак, если выполнено неравенство (R + rx + г2) > (8\ 4- %2)(R + 4-гД а значит, %{r2 > %2(R 4- rj, то ток в цепи увеличивается. Если, наоборот, ^tr2 < %2(R 4- г J, то замыкание источника тока приводит к уменьшению тока в цепи. 3.34* . Воспользуемся тем фактом, что любую схему «черного ящика» из резисторов можно привести к виду (рис. 217), где величины Рис. 217 Рис. 218 Rif R2i ..., R5 выражаются через сопротивления исходных резисторов схемы «черного ящика» (убедиться в этом можно, используя в исход- ной схеме преобразования типа звезда — треугольник и наоборот). По 164
условию задачи через резисторы сопротивлением Rr и R4, а также R3 и R5 каждый раз протекают равные по значению токи (либо вообще не текут, когда клеммы разомкнуты). Воспользовавшись этим обстоя- тельством, схему для простоты рассмотрения приведем к виду, изобра- женному на рис. 218. Тогда по условию задачи С2 м и2 1 = r; + r; ’ 2 = я; + r2r2kr2 + я3') ’ и2 V2 N —__________ tv =______________________ 3 r2 + r2’ . 4 r; + r;ri/(r; + r;)' Нетрудно убедиться, что NrN4 = N2N3. Следовательно, N4 = N2N3/Nr = 40 Вт. 3.35. В первый момент времени после замыкания ключа К кон- денсаторы не заряжены, поэтому напряжение между точками А и В равно нулю (см. рис. 93). Силу тока Ц в цепи в этот момент времени найдем из условия Л = Г/Я1. В установившемся режиме ток между точками А и В будет течь через сопротивления Rt и R3. Поэтому силу тока /2, спустя большой промежуток времени, найдем по формуле Z2 = ^/(R1+R3). 3.36. Рассмотрим установившийся режим, когда напряжение на конденсаторе практически не меняется и равно в среднем (7уст. При очередном замыкании ключа в положение 1 за небольшой ин- тервал времени At заряд конденсатора изменится на величину At(*\ - Uyei)/R2. При замыкании ключа в положение 2 заряд изменится на величину Дг(Г2 - иуст)/к2. Суммарное изменение заряда за цикл должно быть равно нулю: - UyCT)/R2 = 0. Отсюда напряжение 1/уст и заряд конденсатора qyCT в установившемся режиме найдем по формулам Uycr = (Ml + ^K2)/(«1+«2), Чует = CUycr = C^2Rt + M2)/(*i + R2). 165
3.37. Постоянный ток через конденсаторы емкостью и С2 не течет. Поэтому в установившемся режиме через источник тока будет течь ток, равный I=?/(r + R2). Так как конденсаторы соединены последовательно, то их заряды q бу- дут одинаковы, причем 4 - q/C2 = IR2. Следовательно, q = ^R2C.C2H(r + R2)(C. +'C2)]. Напряжения и U2 на конденсаторах соответственно найдем по формулам = q/C1 = ^К2С2/[(г + К2)(С1 + С2)], U2 = q/C2 = ^K2C1/[(r 4- К2) (Сх 4- С2)]. 3.38* . Включим мысленно между точками А и F последовательно два идеальных (без внутреннего сопротивления) источника тока с э. д. с., равными — Uo и Uo. Такое включение, очевидно, ничего не из- менит. Зависимость между силой тока, текущего через включенный ре- зистор сопротивлением R, и э. д. с. источников будет иметь вид Z = o^-pCr0 + pCr0, где — э. д. с. источника, содержащегося в схеме, а коэффициенты а и Р зависят от сопротивлений схемы. Если между точками А и F включить лишь один идеальный источ-. ник с э. д. с., равной — [70, то разность потенциалов между точками А и В станет равной нулю. Значит, два первых члена в уравнении для I компенсируются: I = |Ш0. Коэффициент р равен, очевидно, 1/(К + Кэф),гдеКэф — сопротивление между точками А и В при отключен- ном сопротивлении R. Эта формула справедлива и для случая R = О, что соответствует включению амперметра; при этом /о = ^о/-^эф- Следовательно, искомая сила тока равна I = U0I0/(RI0 + Uo). 3.39. Когда ключ К замкнут, напряжение на конденсаторе поддер- живается постоянным и равно — э. д. с. батареи. Пусть смещение пластины Б при достижении нового положения равновесия равно — хг Заряд на конденсаторе при этом будет равен qt = = 805^/ (d — хД где 5 — площадь пластин конденсатора. Напряженность поля в конден- саторе равна = %/(d — хД но она создается двумя пластинами; по- этому напряженность поля, создаваемая одной пластиной, равна EJ2, 166
и для силы, действующей на пластину Б, можем написать Eiqi/2 = е0^2/ [2(d - xt)2] = кхь (1) где к — жесткость пружины. Рассмотрим ситуацию, когда ключ К замкнули на короткое вре- мя. Конденсатор при этом получил заряд q2 = 80S#/d (пластины не ус- пели сдвинуться), который в дальнейшем остается постоянным. Пусть в новом положении равновесия смещение пластины Б составляет х2, тогда напряженность поля в конденсаторе равна Е2 = q2/ [С2 (d — х2)] и С2 = e0S/ (d — х2). Условие равновесия пластины Б запишется при этом так: E2q2/2 = q22/(2e0S) = E0S?2/(2d2) = кх2. (2) Поделив почленно уравнения (1) и (2), найдем х2 = хх [(J — xJ/J]2. Учитывая, что по условию задачи хг = 0,Id, получим ответ: х2 = 0,08d. 3.40. Представим центральное соединение проволок в виде двух узлов, соединенных проводом 5 — 6, как это показано на рис. 219. Тог- да из соображений симметрии следует, что ток по этому проводу не течет. Поэтому в исходной схеме центральное соединение можно убрать, и мы приходим к схеме, показанной на рис. 220. По условию задачи Е\2 = = R34 = R24 = г, ^15 = ^25 = ^36 = ^46 = Пусть напряжение между точками 1 и 2 равно U. Тогда в единицу времени на участке 1—2 выделится количество теплоты Q12=U2/r. Из закона Ома получаем силу тока, протекающего по участку 3 — 4: ^=^/['•(/2 + 3)]. 167
В единицу времени на участке 3 — 4 выделится количество теплоты б34 = 724г = 172/[г(|/2 + З)2]. Таким образом, искомое отношение равно Q12/e34=(|/2 + 3)2 = ll+6j/2. 3.41. Поскольку анодный ток равен I, то число носителей заряда (электронов) п, подлетающих к аноду в единицу времени, найдем по формуле где е — заряд одного носителя. Импульс Др, передаваемый носителями заряда аноду в единицу времени, равен Др = nmv, где т — масса носителя, v — его скорость. Этот импульс равен средней силе, действующей на анод со стороны оседающих на него носителей заряда. Так как mv2/2 = | е 117, где U — разность потенциалов анода и катода, то среднее давление рср электронов на анод найдем по формуле nmv I т 1 l2\e\U I 1 / 2m [7 Рср = —S-” $”|ёГ / т У |/ |е| ’ 3.42. Нетрудно сообразить, что за время от 0 до tQ напряжение на конденсаторе равно нулю, заряд его также равен нулю, ток через кон- денсатор не течет, а, значит, (JCD все это время равно нулю (рис. 221). i ! *о ! ! Рис. 221 За время от t0 до 2t0 напряжение на конденсаторе, а следовательно, и заряд на обкладках растут линейно, а, значит, в цепи течет по- стоянный ток, поэтому напряжение Ucd постоянно. В интервале вре- мени от 2г0 до 3t0 напряжение на конденсаторе не меняется; значит, ток не течет и Ucd равно нулю. Наконец, в интервале от 3t0 до 5t0 конденсатор разряжается, ток, текущий через резистор, отрицателен и постоянен, и его значение в два раза меньше тока в интервале вре- мени от t0 до 2t0. 3.43. До переключения ключа К заряд конденсатора равен Qi — 168
причем на нижней пластине окажется положительный заряд, а на верхней — отрицательный. После переключения ключа К в процессе перезарядки на верхней пластине окажется положительный, а на нижней отрицательный заряд, равный 32 = Таким образом, источник тока с э. д. с. % 2 совершит работу % 2 (<?i + 4- g2). Искомое количество теплоты, выделяющееся при этом на рези- сторе, равно <11 \ 2С / е=^2й1+«2)-(^- С(^ + ?2)2 2 3.44. Найдем силу тока /, текущего в цепи через источник тока: г У &(Rt+R2) г-Ь R1R2/(K1-Ь R2) r(Rt 4- R2) 4- RrR2 Напряжение на каждом из резисторов Rr и R2 определим по формуле r, * , U R = 6 —1г = -------------. rR1+R2(r4-R1) Мощность Nk2, выделяемую на резисторе R2, найдем по формуле U2r Nr=—^~ R2 R2 %RyR2 rRt 4- R2(RX + r) 1 *7 PR\ (rRtf/R-i + 2rRt (r + KJ + (r + RJ2R2 ’ Максимальной мощности будет соответствовать минимум знаменате- ля. Учитывая классическое неравенство а2 4- Ь2 > 2аЬ, получим, что при R2 = rRJ(r 4- RJ знаменатель дроби минимален и выделяемая на рези- сторе R2 мощность максимальна. 3.45. В момент, когда ток через резистор достигает значения /0, заряд конденсатора емкостью равен Qi — CiIqR' Энергия, запасенная в конденсаторе в этот момент, равна W1=^/(2C1). После размыкания ключа, в конце процесса перезарядки, общий заряд на конденсаторах равен q19 а напряжения на обкладках обоих конденсаторов равны. Запишем эти условия в виде двух уравнений: Q1 + ^2 = #1» ^1/^1 = ^1/^2» 169
где q[ и q2 — заряды конденсаторов после перезарядки. Отсюда получим 41 = 41^1/(^*1 С2), q2 = 410г/(^i Ог)- Полная энергия системы после перезарядки равна w2 = rf/pc,) + q'22/(2c2) = «5/[2 (Ci + с2)]. Количество теплоты, выделившейся в резисторе за это время, равно Q = Wt - W2 = (/0K)2C2/[2(Ct + С2)]. 3.46. До переключения ключа К ток через резистор сопротивле- нием R не течет, и заряд конденсатора емкостью С2 найдем по формуле q = ?C2. Энергия, запасенная в этом конденсаторе, определяется по формуле Wc2 = ?2C2/2. После переключения ключа К заряд q перераспределится между двумя конденсаторами так, что заряд qr на конденсаторе емкостью Ct и заряд q2 на конденсаторе емкостью С2 найдем по формулам 4i +42 = 4> 41/Ci =q2/C2. Полная энергия двух конденсаторов будет равна Wc1+c2 = 42/ [2 (Ct + С2)] = ^С^/[2(С1 + С2Д. Поэтому выделившееся на резисторе количество теплоты Q получится из соотношения „ ?2С2 %2С2 С2 %2СхС2 Q “ ~2 2~ (Ct + С2) “ 2(Cj +С2) 3.47. Пусть к моменту, когда напряжение на конденсаторе стало равно U, через источник тока протек заряд q. Очевидно, что q/U = С. Из закона сохранения энергии получим ^ = G + <?7(2C), где Q — количество теплоты, выделившееся на обоих резисторах. По- скольку резисторы соединены параллельно, то QJQ2 = K2/Ki; отсюда U2 \ CURi Q2 = C{?U---------=-------—(2Г- U). \ 2 Jr,+r2 2(Rt + R2y 3.48. При движении перемычки меняется поток магнитной индук- ции через контур, образуемый перемычкой, рельсами и резистором. В контуре возникает э. д. с. индукции и появляется ток. В результате действия магнитного поля на ток, текущий по перемычке, перемычка будет тормозиться. 170
Найдем тормозящую силу F. Пусть скорость движения перемычки в некоторый момент времени равна v. За малый промежуток времени At перемычка переместится вдоль рельсов на малое расстояние Ах = = v At. Изменение площади контура будет равно vd St, поток магнит- ной индукции за это время изменится на АФ = Bvd At, в контуре по- явится э. д. с., равная ? = - АФ/At = -Bvd, и по перемычке, согласно закону Ома, будет течь ток I = $ /R. Сила, действующая на перемычку со стороны магнитного поля, будет равна F = IBd= — B2d2v/R. Направлена сила F, согласно правилу Ленца, против скорости движе- ния перемычки v. Напишем теперь уравнение движения перемычки (на малом пере- мещении Ах): та = F = —B2d2v/R, или, учитывая, что а = Sv/St, v = Sx/St, mSv= — B2d2 Sx/R. Мы видим, что изменение скорости движения перемычки пропорцио- нально изменению ее координаты х (в начальный момент х0 = 0). Зна- чит, полное изменение скорости vK = v0 = 0 — v0 = — v0 связано с изме- нением координаты (с полным перемещением s) соотношением т (— v0) = — B2d2s/R. Отсюда находим путь s, пройденный перемычкой до полной останов- ки: s = mRv0/B2d2. В том случае, когда направление индукции В составляет угол а с нормалью к плоскости рельсов, s = mRvQ/(B2d2 cos2 а). Действительно, э. д. с. индукции и, следовательно, ток, текущий по перемычке, определяются потоком магнитной индукции через контур, а поток в этом случае определяется проекцией индукции В на нормаль к плоскости контура. 3.49. Линии индукции магнитного поля, создаваемого падающим заряженным шариком, лежат в горизонтальной плоскости. Поэтому поток магнитной индукции Фд через площадь, ограниченную конту- ром, в любой момент равен нулю, так что показания гальванометра будут нулевыми. 171
3.50. Выберем систему координат хОу, центр которой совпадает с мгновенным положением шарика (рис. 222). Ось Ох — «центростре- мительная», ось Оу направлена вертикально, как и индукция магнитно- го поля В. Система уравнений, описывающих движение шарика (считаем, что шарик движется по окружности против часовой стрелки), запишется в виде N sin а — qvB = nw2/(l sin а), N cos а = mg. Кроме того, 2itr/v = Т, г = I sin а. Решая полученную систему уравнений, находим I2 - (Т/2п)2 [2п/(дТ) ± qB/(mg)']2 Знак плюс, если шарик вращается против часовой стрелки, и знак минус, если по часовой стрелке (смотреть надо сверху). 3.51. При движении металлического шарика в магнитном поле из-за действия силы Лоренца на свободные электроны последние рас- пределятся по поверхности шарика таким образом, чтобы результи- рующее электрическое поле внутри шарика было однородным и своим действием компенсировало действие магнитного поля. После этого на- правленное движение электронов внутри металла прекратится. Таким образом, напряженность этого поля равна ЕрезЯ + 4 [® х В] = 0, отсюда Врез — [В X с]. Мы приходим к выводу, что внутри шарика появляется однородное электрическое поле величиной I Врез I = I В|I ®|sina. Максимальная разность потенциалов Афтах возникнет между точ- ками диаметра шарика, параллельного вектору Е^, причем Афтах = I Врез | • 2г = | В1111 sin ос • 2г. 3.52. Индукция магнитного поля соленоида направлена вдоль его оси, поэтому сила Лоренца,' действующая на электрон в любой момент времени, будет лежать в плоскости, перпендикулярной оси соленоида. 172
Так как в начальный момент скорость электрона направлена перпенди- кулярно оси соленоида, то и траектория движения электрона будет ле- жать в плоскости, перпендикулярной оси соленоида. Силу Лоренца найдем по закону F = evB. Траектория движения электрона внутри соленоида будет предста- влять дугу окружности, радиус которой найдем из соотношения evB = = mv2/r; отсюда г = тоЦеВ). Траектория движения электрона показана на рис. 223 (вид сверху), где Oi — центр дуги АС, описываемой электроном, v' — скорость, с кото- рой электрон покидает соленоид Отрезки О А и ОС касаются траектории электрона в точках А и С. Очевидно, что угол между v и v' равен (р = ^АО1С, поскольку ОАОХ = ОСОХ. Чтобы найти <р, рассмотрим прямо- угольный треугольник OAOi; катет О А = R, катет AOt = г. Отсюда tg (ф/2) = = R/r = eBR/(mv). Таким образом Ф = 2 arctg (eBR/mv). Очевидно, модуль скорости не меня- ется на всей траектории, поскольку сила Лоренца в любой момент времени пер- Рис. 223 пендикулярна скорости. Поэтому время движения электрона внутри соленоида найдем из соотношения г<Р v тф 2т (eBR ---— arctg I- еВ еВ--------------\ mv 3.53. При движении перемычки меняется магнитный поток, про- низывающий контур, «замыкаемый» перемычкой. В результате в кон- туре возникает э. д. с. индукции. В течение малого промежутка времени, когда скорость v перемыч- ки можно считать неизменной, мгновенное значение э. д. с. индукции равно ^ = — АФ/Аг = — vbB cos а. Сила тока, текущего через перемычку в это время, равна I = Ад/Аг, где Ад - заряд, накопившийся на конденсаторе за время At, т. е. Ад = CA^f = CbB Av cos а (поскольку сопротивление направляющих и перемычки отсутствует, 173
мгновенное значение напряжения на конденсаторе равно Итак, I = CbB(Av/Ar)cosa = CbBa cos a, где а — ускорение, с которым движется перемычка. На перемычку действуют сила тяжести и сила Ампера. Напишем уравнение движения перемычки: та = тд sin a — IbB cos a = mg sin a — Cb2B2a cos2 a. Отсюда найдем a = mg sin a/(m 4- Cb2B2 cos2 a). Время, за которое перемычка достигнет основания «горки», опре- делим из условия l = at2/2: t = Г)Т 1 / . 21 / — = /----------(т 4- Cb2B2 cos2 a). /а у rag sin a Скорость перемычки у основания равна vK = at = 2lmg sin a m 4- Cb2B2cos2a 3.54* . Магнитный поток, пронизывающий площадку, ограничен- ную сверхпроводящим контуром, постоянен. Действительно, ДФ/Дг = = #, но # = IR = 0 (поскольку R = 0), следовательно, Ф = const. Магнитный поток через площадку, ограниченную контуром, складывается из потока внешнего магнитного поля и потока магнит- ного поля, создаваемого током 7, текущим через контур. Таким обра- зом, магнитный поток, пронизывающий рамку, в любой момент вре- мени равен Ф = а2В0 4- a2az 4- LI. Так как в начальный момент (z = 0, I = 0) Ф = В0а2, в любой другой момент времени сила тока I будет определяться соотношением Ы = — aza2, I = — aza2/L. Результирующая сила, действующая со стороны магнитного поля на рамку с током I, равна сумме сил, действующих на те стороны рам- ки, которые параллельны оси у, т. е. F = 2а | ax 11 = а2а/, и направлена вдоль оси z. Таким образом, уравнение движения рамки имеет вид mz = —тд + a2 al = —тд — a* a2 z/L. Это уравнение аналогично уравнению колебаний тела массой т, 174
подвешенного на пружине жесткостью к = a*v.2/L’. mz = —тд — kz. Из этой аналогии ясно, что рамка будет совершать гармонические ко- лебания вдоль оси z около положения равновесия, определяемого из условия а4а2 mgL -z0=-mg, Zo=-^r- Частота этих колебаний рамки будет равна а2а со = _-. yLm Координата рамки через время t после начала движения равна 3.55. Площади поперечного сечения катушек равны = я!>2/4, S2 = nD2/4. Воспользуемся известной формулой для потока магнитной индукции Ф = LI = BSN. Отсюда получим В = LI/(SN). Поэтому В2 L2 S.N, 12 В, L, S2N2 Ц Но Ц—12, так как провод и источник тока остались прежними. Отно- шение числа витков найдем по формуле NJN2 = D2/Dv Тогда полу- чим В2/В. = L^D^L^D,) = LiDJtL'Dt). Следовательно, индукция магнитного поля внутри новой катушки равна B2=BlL2DJ(LlD2\ 3.56* . Пусть — число витков катушки с индуктивностью Lx, N2 — число витков катушки с индуктивностью L2. Заметим, что состав- ную катушку, индуктивность L которой мы ищем, можно рассматри- вать как катушку с числом витков N = + N2. Если знать связь между индуктивностью и числом витков, то тогда можно выразить L через Li и L2. Для данных геометрических размеров катушки такая связь действительно должна существовать, потому что индуктивность определяется только геометрическими размерами и числом витков ка- тушки (имеются в виду длинные цилиндрические катушки с равномер- ной намоткой). Найдем эту связь. Из принципа суперпозиции для магнитного поля следует, что маг- нитное поле, создаваемое током 1 в катушке данных размеров, про- 175
порционально числу витков в ней. Действительно, удвоение числа вит- ков в катушке можно рассматривать как замену каждого витка на два новых, близко расположенных. Эти два витка дадут вдвое большее по- ле, чем один виток, так как поля от двух витков складываются. Таким образом, поле в катушке при удвоении числа витков увеличивается вдвое. Итак В ~ N (В — магнитная индукция, ток фиксирован). Заме- тим, что магнитный поток, охватываемый витками катушки, равен Ф = BNS ~BN~ N2. Осталось учесть, что L = <b/I~N2. Таким образом, мы получили, что L = kN2 при данной геометрии. Дальше учтем, что = ]/LJk, N2 = ]/L2/k, поэтому L= k(Nt + N2)2. Следовательно, L — 4- L2 4- 21/LiL2. 3.57. Для двигателя с независимым возбуждением получим схему, изображенную на рис. 224. В первом случае, т. е. когда лебедка не на- гружена, 0 = = (# - £\)/г, где г — внутреннее _____«I __ J____ сопротивление двигателя, $t — наводимая э. д. с., ' ' причем % Y = avp Таким образом, = JT, откуда а=^7и1. Во втором случае расходуемая мото- ром мощность равна ?2/2 = - ^2) X2/r = mgv2. Рис. 224 а Наводимая э. д. с. индукции равна е 2 = av2. Таким образом, для внутреннего сопротивления двигателя находим r = (^ - av2)а/(тд). В случае выделения максимальной мощности можно написать Nmax = (? - Л Г/г = m'gv', причем максимальное значение выделяемой мощности, как нетрудно показать, 8" = av' достигается при условии 8' = ^/2 (максимум знаме- нателя). Отсюда v' = #7 (2a) = vJ2 = 2 м/с, 2 т' = mvj [2 (vt — v2)] = 10— « 6,7 кг. 3.58. Нарисуем графики зависимостей от времени внешнего на- пряжения l/BH(t), тока в цепи /с(г) (который протекает только в одном направлении, когда диод открыт), напряжения на конденсаторе С7с (О и напряжения на диоде UAK(t) (рис. 225). 176
Таким образом, напряжение между точками А и К меняется в пределах от 0 до — 2U0. 3.59. Так как ток и напряжение изменяются синфазно и амплитуда тока равна I = ^0/К (т. е. вклады С и L скомпенсировались), то 1/(соС) = coL. Из соотношений Uc = q/C, dq/dt = I получаем U с = sin ю^/(Л(оС). Поэтому амплитуда напряжения между обкладками конденсатора равна U о = ^QGiL/R. 3.60. При установившихся колебаниях работа внешнего источника тока должна равняться выделяемой на сопротивлении теплоте. Для этого должны быть равны амплитуды внешнего напряжения и напря- жения на сопротивлении: RIQ = Uo. Поскольку ток в цепи и заряд на 177
конденсаторе связаны уравнением I = dq/dt, амплитуды тока /0 и заряда q0 находят по формуле 'о ~ где резонансная частота соо равна соо = i/j/Ec. По условию задачи U0=U = qJC; отсюда q0 = (<jj/.R)]/Z/C = 10-8-Кл. 3.61. При 0 < £ < т в контуре будут происходить колебания заряда, причем q = (С U/2) cos соо£, соо = ]/2/(LC). В момент времени т пробитый конденсатор имел заряд (CU/2) cos юот и обладал энергией (CC72/8)cos2 соот. После пробоя эта энергия пере- шла в теплоту и была потеряна для системы. Оставшаяся энергия равна W = CU2/4 - (CH2/8)cos2 соот- Амплитуду колебаний заряда после пробоя найдем из условия W = = qo/(2C); отсюда q0 = (CU/2)|/2 — COS2 (Оот. 3.62. Достаточно зашунтировать сверхпроводящую обмотку рези- стором с малым сопротивлением, способным выдержать большую температуру. Тогда в рабочем состоянии ток будет идти по обмотке, сколь бы мало ни было сопротивление резистора. Если же участок об- мотки утратит сверхпроводящие свойства, т. е. у него будет большое сопротивление, то ток потечет через шунтирующее сопротивление. В этом случае теплота выделится на резисторе.
4. ОПТИКА
4.1. Лучи, однократно отраженные от зеркальной поверхности ко- нуса, распространяются так, как если бы они шли от совокупности то- чечных мнимых источников, расположенных по окружности. Каждый такой источник симметричен источнику S относительно соответствую- щей образующей конуса. Изображением этих источников на экране бу- дет кольцо. Существенно, что пучок лучей, попадающий от каждого мнимого источника на линзу, плоский: он проходит не через всю по- верхность линзы, а пересекает линзу вдоль соответствующего диа- метра. Поэтому степень ослабления такого пучка диафрагмой зависит от формы и ориентации диафрагмы. Симметричная кольцевая диафрагма (см. рис. 116) в равной мере ослабит пучки, идущие от всех мнимых источников. В этом случае ос- вещенность кольца на экране уменьшится равномерно. Диафрагма, изображенная на рис. 117, пропустит целиком пучки, плоскости ко- торых образуют с вертикалью углы а < а0. Следовательно, освещен- ность верхних и нижних частей кольца на экране не изменится. Прочие пучки будут срезаться диафрагмой, причем тем сильнее, чем ближе плоскость пучка к горизонтальной плоскости. Поэтому на боковых участках кольца по мере изменения угла а от а0 до л/2 освещенность будет уменьшаться. 4.2. Пренебрежем сначала размерами зрачка, считая его то- чечным. Ясно, что из прошедших через линзу лучей в глаз попадут те и только те, которые перед попаданием на линзу прошли через точку В (рис. 226), сопряженную с той точкой, в которой расположен зрачок. Расстояние b от линзы до точки В найдем, воспользовавшись формулой тонкой линзы: Теперь понятно, что экран должен совпадать с действительным изображением зрачка в плоскости S. Из рис. 226 видно, что радиус ми- нимального экрана равен b R = — г » 0,5 мм а и его надо расположить в плоскости S с центром в точке В. 4.3* . Найдем сначала распределение давления газа вблизи оси со- суда. Рассмотрим элемент объема газа Ar AS (рис. 227). Центростреми- тельное ускорение этого элемента а = ш2г обеспечивается разностью соответствующих давлений: [р(г 4- Аг) — р(г)] AS = р Ar ASo2r. Поэтому для изменения давления получим уравнение dp/dr = рсо2г. 180
Так как для идеального газа выполняется соотношение цр = pRT (R — универсальная газовая постоянная), то dp/dr = p[\i(i)2/RT)r. Согласно условию задачи, при г < гп р(г) — р0 « р0; следовательно, Р('-)*Ро(1+^7 Г2\ \ * / Соответственно для плотности газа при г < гп получим / цсо2 Л ц p(r)«p0 1+——^г2), Ро = Ро—, \ ZK1 / К1 а для показателя преломления имеем и (г) = п0 + кг2, n0 = 1 + ар0, ар0 ( ¥ 2 \Rt)' Найдем теперь угол преломления луча, проходящего через сосуд на расстоянии г от оси. Оптический путь, пройденный им в сосуде, ра- вен п (г)1. Оптическая разность хода 5ОПТ двух близких лучей после прохо- ждения сосуда должна равняться геометрической разности хода 5, вы- званной отклонением лучей от первоначального направления распро- странения. В этом случае интерферен- ция лучей приведет к их усилению (принцип Гюйгенса). Из рис. 228 сле- дует, что 5ОПТ = [и (г + Аг) — п (г)] I, 5 = Ar sin ф. Отсюда 5 [и (г + Аг) —и (г)] . sin ф = — = —--------L----= 2klr. Ar Ar Здесь можно сделать следующий вы- вод. Если рассматривать узкий пучок Рис. 228 181
света такой, что угол отклонения ф будет мал, то ф ~ г, т. е. вра- щающийся сосуд будет работать как рассеивающаяся линза с фо- кусным расстоянием F = (2/с/)-1- Итак, для максимального угла отклонения получим sin <pmax = 2klrn. Поэтому искомый радиус пятна на экране равен К = rn + Ltg(pmax. В приближении рассеивающей линзы получим К ~ Гп “Ь ^Фтах гп “Ь 2/с/гnL = гп 1 + (X,polL цсо у ~rtJ 4.4. Рассматриваемая в задаче подзорная труба — системы Кепле- ра. Угловое увеличение к = F/f, поэтому фокусное расстояние окуляра равно f = F/k = 2,5 см. При приближении рассматриваемого предмета из бесконечности до минимально возможного расстояния а изображе- ние предмета в объективе будет перемещаться из фокальной плоскости в сторону окуляра на расстояние х, которое можно найти по формуле линзы: 1 _ a-F F + х aF ’ 1 1 _ 1 a F + х F9 aF F2 х =--------F «----, a — F а т. к. а » F. Таким образом, надо найти х. Окуляр трубы — лупа; при рассматривании предметов через лупу ненапряженным, т. е. аккомоди- рованным на бесконечность, глазом предмет должен располагаться в фокальной плоскости лупы. Искомое расстояние х равно смещению фокальной плоскости окуляра при его настройке; при этом, очевидно, окуляр надо отодвигать от объектива. Рис. 229 Как видно из рис. 229, при рассматривании бесконечно удаленных предметов через сдвинутый окуляр пучок света на выходе из окуляра будет сходящимся. При падении на трубу параллельного пучка лучей такое положение окуляра необходимо для дальнозоркого глаза, опти- ческая сила которого в ненапряженном состоянии недостаточна для собирания параллельного пучка света на сетчатке. Максимальный сдвиг окуляра соответствует оптической силе D+ — 4-10 дптр. Очки 182
с такой оптической силой собирают параллельный пучок света на рас- стоянии b — l/D + . Это расстояние b и определяет расстояние х: 111 f2 5 -------1-= —, отсюда х =-------= — см. ь f b-f 6 Искомое расстояние равно F2 0,25-6 м2 а %----=----—;----= 30 м. х 5-10 2 м 4.5. Можно. Ответ следует, из рис. 230. Искомые области заштри- хованы. 4.6. Когда карандаш входит в воду, поверхность воды около ка- рандаша образует рассеивающую линзу — лучи от источника света от- клоняются от оси карандаша. Поэтому под карандашом образуется большое темное пятно (рис. 231). Когда карандаш вытаскивают из воды, поверхность воды около ка- рандаша образует собирающую лин- зу — лучи от источника света откло- няются к оси карандаша. Поэтому под карандашом образуется светлое пятно (рис. 232). 4.7. Ответ следует из рис. 233. Первое светлое кольцо образуют лучи, испытывающие одно отражение от стенок корпуса до выхода из него (пример такого луча обозначен цифрой 1). Второе светлое кольцо образуют лучи с двумя отраже- ниями (на рисунке этот луч обозначен цифрой 2) и т. д. Понятно, что, чем больше отражений, тем больше света поглощается корпусом 183
Рис. 233 шариковой ручки. Поэтому чем дальше светлое кольцо от центра, тем оно менее светлое. 4.8. Покажем, что все лучи, идущие от точечного источника в на- правлении к экрану, после преломления окажутся внутри цилиндра и после многократных отражений от его боковой поверхности в конце концов пройдут через отверстие в экране. Действительно, предельный луч от точечного источника света S, угол падения которого на левое основание цилиндра равен л/2, после преломления будет образовывать угол а с осью цилиндра, причем sin а = 1/и (по закону преломления). Угол падения ф этого луча на бо- ковую поверхность цилиндра удовлетворяет условию а + ф = л/2 (рис. 234). Так как sina= 1/п = 1/1,5 < )/2/2, то a < л/4, а ф > л/4, т. е. угол падения на боковую поверхность цилин- дра будет больше угла полного внутреннего отражения. Поэтому впоследствии этот луч не сможет выйти из цилиндра нигде, кроме как через правое основание. Любой другой луч, выходящий из источ- ника в сторону экрана с отверстием, после преломления в левом осно- вании цилиндра пойдет под мень- падет на боковую поверхность под шим углом к оси, а тем самым углом, заведомо большим угла полного внутреннего отражения. Прозрачный цилиндр, таким образом, «соберет» в отверстие лучи, попадающие в телесный угол 2л стерадиан. В отсутствие цилиндра в отверстие экрана попадал световой по- ток, сосредоточенный внутри телесного угла nd2/(4l)2. Таким образом, световой поток через отверстие в присутствии прозрачного цилиндра 184
увеличится в ——— = 8 • 104 раз. nJ2/(4Z)2 4.9. Толщину линзы объектива найдем из геометрических сообра- жений (рис. 235). Действительно, г\ == (2ЛХ - h)h » 2КХА1, h = г?/(2Кх), где — радиус кривизны объектива. Запишем равенство оптических путей ABF и CDF в случае, когда в трубу налита вода: (/1 - h)n3 + 2hnCT = ив^ + h. Здесь j\ — фокусное расстояние линзы объектива, когда налита вода. Подставляя значения h и = ]//2 + г2 » Д + г2/(2Л)> получим у.2 у,2 ~ 4d [(ИСТ ~ 1) + (ист — ив)]. 1 Отсюда у *1”в (ист — 1) + (ист — ив) В случае, когда в трубе нет воды, фокусное расстояние линзы объек- тива равно г(°) = 2(nCT-l)- Поэтому f = ЛО) 2(”ст--1)Ив 1 1 (ист - 1) + (ист - ив) ’ Аналогичный расчет для фокусных расстояний /2 (с водой) и /(20) (без воды) линзы окуляра дает следующий результат: f — f(o> 2(Ист 1)ив 12 72 (Ист — 1) + (пст — ”в) 185
Таким образом, ь = /1 + /2 = (Л0, + Л0)) 2 (мст — 1) п3 (ист -1)4- (ист - ив) ’ Так как /ЭД 4- /ЭД = Lo, то искомое расстояние между объективом и окуляром равно 2(ист 1)ив 2иоу ив 1 = 30 см. Рис. 236 4.10. Пусть паук находится в точке А (рис. 236), расположенной над верхней точкой D шара. Поверхность шара, соответствующая дуге окружности BDB', является видимой для паука. Точки В и В' — точки пере- сечения касательных, проведенных из точки А к поверхности шара. Луч АВ будет распространяться внутри шара по отрезку ВС. Угол а находится к из условия \ sin а = 1/ист, I где Псу — коэффициент преломления / стекла. Этот луч выйдет из шара по направлению СА'. Поэтому часть по- верхности шара, соответствующая дуге CD'C', будет также видимой (для при- мера показано прохождение луча AKLM). Невидимой, для паука будет по- верхность шарового пояса, которой соответствуют дуги ВС и В'С'. Угол у определяется из условия cos у — R/(R 4- й), где R — радиус шара, h — высота паука над поверхностью шара. Так как по условию R » й, то у « 0. Заметим теперь, что 0 — п — 2а, a sin а — 1/ист. Поэтому Р = п — 2 arcsin (1/пст) ~ л/2. Итак, видимой для паука является противоположная для него поло- вина поверхности шара; там и должна находиться муха. 4.11. Через боковую поверхность цилиндра не выйдет ни один луч, если для луча с углом падения у « л/2 (рис. 237) угол падения а на внутреннюю поверхность будет удовлетворять соотношению sin а > > 1/и. В этом случае на боковой поверхности луч испытает полное внутреннее отражение. 186
Из геометрических соображений следует, что sin а = j/1 — sin2 р, sin р — 1/п. Таким образом, 4.12. Из условия задачи следует, что фокусы обеих линз совмещены, т. е. расстоя- ние между линзами равно 3/, где f — фокус- ное расстояние более слабой линзы. В первом случае все лучи, попавшие в трубу, выйдут из нее и образуют круглое пятно радиуса г/2, где г — радиус трубы (рис. 238). Во втором случае из трубы выйдут только те лучи, которые до трубы шли на расстоя- нии, меньшем чем г/2, от оси трубы. А на экране эти лучи образуют Рис. 238 круглое пятно радиуса г (рис. 239). Таким образом, если J — сила света, падающего на трубу, то отношение освещенностей пятен до и после перевертывания трубы равно F J F J/4 A = J_ 1 л(г/2)2’ 2 яг2’ 16 4.13. В аппарат поступает свет, отраженный от поверхности фаса- да. Отражение света от штукатурки можно считать практически не за- висящим от угла отражения. В этом случае на объектив аппарата па- дает световая энергия от объекта, пропорциональная телесному углу, под которым виден из объектива фасад. При сокращении расстояния до объекта в два раза телесный угол увеличивается в четыре раза, и на объектив от той же площади объекта падает в четыре раза больше световой энергии. При таких больших расстояних до объекта расстояние от объек- тива до пленки в аппарате практически не изменяется при выставлении на объективе расстояния до объекта, т. е. при фокусировании последне- го, и равно фокусному расстоянию объектива. Телесный угол, внутри которого энергия от объектива падает на площадь изображения, зави- 187
сит линейно от телесного угла, под которым виден фасад, т. е. линейно зависит от расстояния до объекта. В этом случае освещенность площа- ди изображения (она по условию равномерно распределена по этой площади), которая и определяет выдержку, пропорциональна соответ- ствующей энергии, падающей на объектив от фасада, и обратно про- Рис. 240 порциональна площади изо- бражения. Так как это отно- шение практически не за- висит от расстояния до объекта при данных услови- ях, то выдержку менять было не нужно. 4.14*. Задача является аналогом оптической зада- чи, в которой рассматрива- ется преломление плоской волны в призме. При этом лучу света, идущему от точки А в точку В (рис. 240), согласно законам геометрической оптики, требуется наименьшее время по сравнению со всеми другими траекториями. Рыбак должен двигаться по траектории «луча света», т. е. подойти к точке Е залива под углом у, переплыть залив в лодке перпендику- лярно биссектрисе угла а и двигаться далее по суше в направлении к точке В. Угол у найдем* из закона преломления (и = 2): sin у = п sin (а/2). Расстояние а равно а = h tg у = h п sin (а/2) ]/1 - и2 sin2 (а/2) Расстояние b найдем из уравнения а + b = )//2 — h2. Отсюда . j/1 - n2 sin2 (a/2) n2 sin2 (a/2) Если b > 0, т. e. I —h > h -------=---=-----, то рыбак должен восполь- 1 — n2 sin2 (a/2) зоваться лодкой. Отдельные участки пути при этом будут равны „„ к • a (у/li—П l nsin(a/2) \ . a EK = р = b sin — = I 1/r — h— h — I sin —, 2 \ |/1 - и2 sin2 (a/2) / 2 h h AE = q =-- - ------------- cosy ]/f-и2 sin2 (a/2) 188
I2 — h2 n2 sin2 (a/2) Итак, если ———>---------------- - ---то искомое время равно h2 1-и2 sin2 (a/2) t = 2| —+ — \ v v/n t 1 2h/ 1 и]//2 — h2 sin (a/2) n2 sin2 (a/2) \ v \ }/l - «2 sin2 (a/2) h |/1 - и2 sin2 (a/2) / 2h(]f I/*2-*2 • a\ 2 h 2 J V _ I2 - h2 n2 sin2 (a/2) Если -------—<---------—, h2 1 — n2 sin2 (a/2) 4.15. В силу симметрии изображение точечного источника S будет опять на расстоянии b от шара, но 4.16. Наблюдатель внутри ко- рабля сможет увидеть лишь те лучи, для которых sin a < 1/ист (если sin a > 1/ист, то такой луч испытает 21 t = —. v с другой стороны (рис. 241). полное внутреннее отражение и не попадет к наблюдателю, рис. 242). Для угла р имеем соотношение «ст «в sin р = «ст sin a, sin р =-sin a, «в где ист — показатель преломления стекла. Так как | sin а | < 1/«ст, то | sin р | < 1/ив. Поэтому наблюдатель сможет видеть только те объекты, свет от которых попадает на иллюминатор с углом падения р < arcsin (1/«в). Из рис. 242 ясно, что радиус R круга на дне, доступного наблюдению, будет R « Л tg р, и искомая площадь (/itg р » D/2) равна S = nR2 « Tth2/{nl — 1) ~ 82 м2. 4.17* . Близорукие люди носят рассеивающие очки, уменьшающие оптическую силу их глаз, а дальнозоркие — собирающие. Нетрудно со- образить, что глаз за рассеивающей линзой будет казаться умень- шенным, а за собирающей — увеличенным. Однако если вы не виде- ли вашего собеседника без очков, то определить, увеличены или 189
уменьшены его глаза, очень трудно, особенно если очки не слиш- ком сильные. Проще всего определить, куда сдвигается видимый край лица за очками по отношению к соседним частям лица: если внутрь, то это рассеивающие линзы — у собеседника близорукость; если на- ружу, то собирающие линзы — дальнозоркость. Рис. 243 4.18. Разложим вектор скорости человека v на две составляю- щие - одну параллельную зеркалу v ц и другую - перпендикулярную зеркалу vj_: v = гц 4- vj_ (рис. 243). Очевидно, что скорость изображения будет равна v' = &ц — Поэтому скорость, с которой человек прибли- жается к своему изображению, находится как скорость человека относительно своего изображения по формуле ^отн = 2г 1 = 2v sin а. 4.19. Пусть О — центр сферической поверхности зеркала, АВС — луч, который падает на расстоянии BE от оси зеркала. ОВ =s R (рис. 244). Из прямоугольного треугольника ОВЕ найдем, что sin а = = h/R. Треугольник ОВС равнобедренный, так как z. ABO = z. ОВС по закону отражения, а z. ВОС = z. ABO как внутренние, накрест лежащие углы. Отсюда OD = DB = R/2. Из треугольника ODC находим х = R/(2 cos а) = ^/(2 ]/R2 - h2) (С — точка пересечения отраженного от зеркала луча с оптической осью). Для луча, проходящего на расстоянии h19 ввиду того что hl « R2, xt »К/2 с погрешностью около 0,5%. Для луча, проходящего на рас- стоянии h2, расстояние х2 = 3,125 см. Окончательно получим Ах = х2 — xt » 0,6 см # 0 (!) 190
4.20. Рассмотрим некоторую светящуюся точку А нити и про- извольный луч АВ, выходящий из нее. Проведем плоскость через этот луч и светящуюся нить. Из геометрии задачи очевидно, что данный луч при всевозможных отражениях всегда будет оставаться в по- строенной плоскости (рис. 245). После первого отражения от кониче- ской поверхности луч АВ пойдет так, как если бы он вышел из точки А — мнимого изображения точки А. А чтобы после первого отражения ни один выходящий из точки А луч больше ни разу не попал на зерка- ло, необходимо, чтобы точка А лежала не выше прямой ОС — второй образующей конуса, лежащей в плоскости луча (точка О — вершина ко- нической поверхности). Это будет иметь место, если A0D 4- z. AOD 4- АОС = 3 (а/2) > 180°. Следовательно, Omin 120 .
СОДЕРЖАНИЕ Предисловие................................................. 3 Задачи Решения 1. Механика......................................... 5 61 2. Теплота и молекулярная физика .*................. 30 125 3. Электричество и магнетизм........................ 41 149 4. Оптика........................................... 56 179 Александр Иванович Буздин Вячеслав Анатольевич Ильин Игорь Владимирович Кривченков Сергей Сергеевич Кротов Никита Алексеевич Свешников ЗАДАЧИ МОСКОВСКИХ ФИЗИЧЕСКИХ ОЛИМПИАД Под редакцией С. С. Кротова Серия «Библиотечка «Квант», выпуск 60 Редактор Н. А. Михалина Художник Д. А. Кры лов Художественный редактор Т. Н. Кольченко Технический редактор С. Я. Шкляр Корректор Н. Б. Румянцева ИБ № 12980 Сдано в набор 02.10 86. Подписано к печати 07.01.88. Т-04606 Формат 84 X 108/32. Бумага кн. жури. Гарнитура тайме. Печать офсетная. Усл. печ. л. 10,08. Усл. кр.-отт. 10,5 Уч -изд. л 10,56. Тираж 300 000 экз. Заказ №786. Цена 40 коп. Ордена Трудового Красного Знамени издательство «Наука» Главная редакция физико-математической литературы 117071, Москва, В-71, Ленинский проспект, 15. Диапозитивы изготовлены в Ордена Октябрьской революции, ордена Трудового Красного Знамени Ленинградском производственно-техни- ческом объединении «Печатный двор» имени А. М. Горького Союз- полиграфпрома при Государственном Комитете СССР по делам издательств, полиграфии и книжной торговли. 197136, Ленинград, П-136, Чкаловский пр., 15. Отпечатано с диапозитивов в типографии им. И. Е. . Котлякова издательства «Финансы и статистика» Государственного комитета СССР по делам издательств, полиграфии и книжной торговли. 195273, Ленинград, Руставели, 13.
40 коп.